267893536-EMQs-busyspr-pdf.pdf

April 25, 2018 | Author: Hiba | Category: Urinary Incontinence, Childbirth, Menstrual Cycle, Pregnancy, Cervical Cancer
Share Embed Donate


Short Description

Download 267893536-EMQs-busyspr-pdf.pdf...

Description

Options for Questions 1-2 A Prolactinoma

B Pre-mature ovarian failure

C Turner’s syndrome

D Ovarian hyper-stimulation syndrome

E Polycystic ovary syndrome

F Hypothyroidism

G Cushing’s syndrome

H Congenital adrenal hyperplasia

Hypothalamic amenorrhoea

J

Hyperthyroidism

K Androgen insensitivity syndrome

I

L

Pure gonadal dysgenesis

Instructions:For each of the case histories described below, choose the singlemost likely cause of menstrual abnormalities from the above list. Each option may be used once, more than once, or not at all.

Question 1

Question 2

A 35 year old woman and her 40 year old partner have been referred to the fertility clinic because of a failure to conceive after 3 years of unprotected intercourse. The semen analysis is normal. The woman has irregular periods every 3-6 months but no other symptoms and her BMI is 27. The woman’s results are as follows: FSH (day 3) = 20mIU/ml, LH (day 3) = 8mIU/ml, prolactin = 110 ng/ml, progesterone (day 21) = 1.5ng/ml, testosterone = 2.0pg/ml, DHEA, DHEA-sulphate and thyroid function tests were normal. A 35 year old woman with one previous normal pregnancy 3 years earlier attends the gynaecology clinic because of irregular periods every 4-6 months. On direct questioning, she complains of persistent headaches in the last 9 months which are worse first thing in the morning and a milky discharge from her right breast. Her BMI is 27 and there is a bi-temporal hemi-anopia on

Your answer: A Correct answer: B

Your answer: A Correct answer: A

clinical examination. | EXPLANATION | Options for Questions 3-4 A Routine recall in 3 years

B Routine recall in 5 years

C Hysteroscopy + D&C

D Laser vapourisation

E Refer for colposcopy

F Cold knife cone biopsy

G Repeat cervical smear as soon as possible

H Cryotherapy

I

J

No further cervical smears required

Repeat cervical smear in 6 months

Instructions:For each scenario described below, choose the singlemost appropriate management from the above list of options. Each option may be used once, more than once, or not at all. Your answer: A Question 3

Question 4

A smear 30 yearwhich old woman with as previously normal cervical has a is reported inadequate because thesmears specimen was poorly fixed. A 35 year old woman had a cervical smear 3 years ago which showed border-line nuclear abnormalities. She has since had two consecutive smears 12 months apart which have been reported as negative

| EXPLANATION | Options for Questions 5-6

Correct answer: G

Your answer: A Correct answer: A

A Continue iv oxytocin

B Deliver by caesarean section

C Evacuation of retained products of conception

D

E Bimanual compression of the uterus

F Venous access and resuscitation

G Examination under spinal anaesthesia

H Lovset’s manoeuvre

I

Intravenous ritodrine

K Sub-cutaneous terbutalline

No additional intervention at this stage

J

Manual replacement of the uterus

L

Administer uterotonic agent

Instructions:For each of the case histories described below, choose the singlemost appropriate initial management from the above list. Each option may be used once, more than once, or not at all.

Question 5

Question 6

A 16 year old primigravida attends for induction of labour following intra-uterine fetal death at 36 weeks gestation. Maternal BP is 120/80 with no proteinuria and all blood tests are normal. The fetus is in a breech presentation. Prostaglandin is administered and oxytocin commenced at 12:00 according to the unit protocol. At 00:00, the cervix remains 1cm long and the os is closed. There are 3 uterine contractions every 10 minutes. A 34 year old woman is bleeding heavily and collapses during the third stage of labour. Her pulse is 40bpm and systolic BP is 70mmHg. There is a mass at the introitus consistent with uterine inversion

Your answer: A Correct answer: A

Your answer: A Correct answer: F

| EXPLANATION | Options for Questions 7-8 A Failure to relieve symptoms

B Damage to bowel

C Removal of ovaries

D Irregular bleeding for 3-4 months

E Failure to identify disease

F

G Shoulder-tip pain

H Haemorrhage requiring return to theatre

I

J

Laparotomy

Failure to visualise uterine cavity Failure rate 1 in 200

Instructions:For each of the case histories described below, choose the singlemost relevant complication that you must discuss with the patient when taking consent prior to surgery from the above list of options. Each option may be used once, more than once, or not at all.

Question 7

Question 8

A 35 year old nulliparous woman with menorrhagia is having the levonorgestrel releasing intra-uterine system inserted under general anaesthesia

A 40 year old woman with a previous failed TVT for genuine stress incontinence is having a Burch colposuspension

| EXPLANATION | Options for Questions 9-10 A Stress incontinence

B Urinary retention with overflow

C Urinary retention

D Detrusor instability

E Detrusor hyper-reflexia

F Interstitial cystitis

G Genuine stress incontinence

H Mixed urinary incontinence

I

J

Bladder tumour

Urinary tract infection

Your answer: A Correct answer: D Your answer: A Correct answer: A

K Diabetes mellitus Instructions:For each of the case histories described below, choose the singlemost likely diagnosis from the above list of options. Each option may be used once, more than once, or not at all.

Question 9

Question 10

A 35 year old woman complains of urinary frequency, urgency, urge incontinence and bladder pain. Urine dipstix showed haematruria. Her symptoms have not responded to conservative and medical treatment. Cystoscopy showed petechial haemorrhages in the bladder mucosa A 40 year old woman has been investigated extensively for persistent microscopic haematuria which was initially detected during pregnancy. Over the last year, she has developed urinary urgency with bladder pain. Bladder biopsy showed a chronic inflammatory infiltrate in the detrusor muscle

Your answer: A Correct answer: F

Your answer: A Correct answer: F

| EXPLANATION | Options for Questions 11-12

A

Counsel and offer evacuation of retained products of conception

C Counsel and offer laparoscopy & salpingectomy

B Counsel and offer support group D Counsel and offer investigation for recurrent miscarriage

E Counsel and offer treatment with methotrexate

F Counsel and offer laparotomy & salpingectomy

G Counsel and offer termination of pregnancy

H Perform salpingectomy

I

J

Serial HCG assay

L

Repeat trans-vaginal scan 7 days later

Proceed to laparotomy

Counsel and refer to early pregnancy assessment K unit

Instructions:For each patient described below choose the singlemost appropriate initial management option from the list. Each option may be used once, more than once, or not at all.

Question 11

Question 12

A 25 year old woman had a transvaginal scan following IVF treatment during which 2 embryos were replaced and was found to have a 6 week singleton intra-uterine pregnancy. She presents 2 weeks later with abdominal pain and vaginal bleeding and is found to have an empty uterus with a small amount of free fluid in the pouch of Douglas. A 25 year old woman had a transvaginal scan following IVF treatment during which 2 embryos were replaced and is found to have a 6 week singleton intra-uterine pregnancy with a 5cm cystic structure adjacent to the right ovary with internal echoes consistent with an ectopic pregnancy.

Your answer: A Correct answer: B

Your answer: A Correct answer: C

| EXPLANATION | Options for Questions 13-14 A Stop heparin on the evening before induction

B Commence oral warfarin

C Await results of V/Q scan then commence treatment E Await results of D-dimers assay

D Await results of venogram then commence treatment F Commence therapeutic dose of heparin

G

Commence prophylactic dose of heparin postpartum

H Oral aspirin therapy

Antenatal prophylactic treatment with heparin

J

Antenatal treatment with warfarin

K Prophylactic heparin for 6 weeks post-partum

I

L

Stop heparin therapy

Instructions:For each scenario described below, choose the singlemost appropriate management from the above list of options. Each option may be used once, more than once, or not at all.

Question 13

Question 14

A 30 year old woman with a previous DVT presents for antenatal care at 15 weeks gestation. There is no family history of VTE and her BMI is 25. She is otherwise fit and well with a negative thrombophilia screen A 30 year old woman with recurrent first trimester miscarriages is known to have the anti-phospholipid antibody syndrome and has had a spontaneous vaginal delivery at 39 weeks gestation. Her BMI is 27 and she is otherwise fit and well with no family history of VTE.

Your answer: A Correct answer: K

Your answer: A Correct answer: G

| EXPLANATION | Options for Questions 15-16 A 85% 5 year survival

B 70% 5 year survival

C 60% 5 year survival

D 40% 5 year survival

E 25% 5 year survival

F Virtually 0% 5 year survival

G Almost 100% 5 year survival

H Overall life-time risk ~1%

Overall life-time risk ~2%

J

Overall life-time risk _1.5%

K Overall life-time risk ~3%

I

L

Overall life-time risk of 0.1%

Instructions:For each scenario described below, choose the singlemost appropriate information that should be given to the woman from the above list of options. Each option may be used once, more than once, or not at all.

Question 15

Question 16

A 68 year old woman attends the gynaecology clinic 4 weeks after TAH + BSO for endometrial carcinoma. The histology shows that the tumour had extended to involve the cervix and she has been advised to have radiotherapy. She enquires about the survival rate for women with similar tumours. A 32 year old teacher has been referred to the gynaecology clinic because of marked anxiety since her friend died suddenly from ovarian cancer at the age of 35 years. She has no family history of ovarian cancer and enquires about the likelihood of any woman developing the disease.

Your answer: A Correct answer: B

Your answer: A Correct answer: I

| EXPLANATION | Options for Questions 17-18 A Endocervical and urethral swabs for culture

B EUA + cystoscopy

C MRI scan

D Out-patient hysteroscopy and biopsy

E Hysteroscopy D&C

F Vulval biopsy

G Cervical smear

H Peritoneal fluid for cytology

I

J

Serum CA-125

Colposcopy

Instructions:For each scenario described below, choose the singlemost appropriate investigation from the

above list of options. Each option may be used once, more than once, or not at all.

Question 17

Question 18

A 56 year old woman with a 3 year history of vulval lichen sclerosis complains of a firm lump on the labia majora which has increased in size over the last 3 months and is bleeding intermittently A 34 year old woman complains of a 3 months history of postcoital bleeding. The cervix appears normal and microbiology swabs are negative. Cervical smear shows atypical glandular cells

Your answer: A Correct answer: F

Your answer: A Correct answer: J

| EXPLANATION | Options for Questions 19-20 A Congenital syphilis

B Congenital varicella syndrome

C Parvovirus B19 infection

D Turner’s syndrome

E Parder-Willy syndrome

F Fetal hydantoin syndrome

G Group B streptococcal infection

H Congenital rubella syndrome

I

Cri-du-chat syndrome

K Patau’s syndrome

J

Edward’s syndrome

L

Down’s syndrome

Instructions:For each of the case histories described below, choose the singlemost likely cause of fetal abnormality from the above list of options. Each option may be used once, more than once, or not at all.

Question 19

Question 20

A 42 year old woman is late booking for antenatal care. The estimated gestation age by ultrasound scan is 22 weeks and the fetus is found to have an atrio-ventricular septal defect and a double bubble sign in the upper abdomen A 42 year old woman had bleeding in early pregnancy and was found to have an 8 week viable pregnancy. Anomaly scan at 22 weeks showed a head and abdominal circumferences below the third centile , bilateral choroids plexus cysts and rockerbottom feet

Your answer: A Correct answer: L

Your answer: A Correct answer: J

| EXPLANATION | Options for Questions 21-22 A No additional intervention at this stage

B Deliver by caesarean section

C Evacuation of retained products of conception

D Artificial rupture of membranes

E Oxytocin induction of labour G Controlled artificial rupture of membranes in theatre

F Venous access and resuscitation H Intra-muscular analgesia

I

Prostaglandin induction of labour

K Sub-cutaneous terbutalline

J

Manual replacement of the uterus

L

Delay induction of labour

Instructions:For each of the case histories described below, choose the singlemost appropriate management from the above list. Each option may be used once, more than once, or not at all.

Question 21

A 34 year old nulliparous woman presents in spontaneous labour at 37 weeks gestation with the cervix 3cm dilated. Four hours later, the cervix is 8cm dilated and a frank breech presentation is diagnosed.

Your answer: A Correct answer: B

Question 22

A 24 year old primigravida has been admitted for induction at 38 weeks gestation with a dichorionic twin pregnancy because of maternal discomfort. The pregnancy has otherwise been uncomplicated and twin 1 is cephalic. The cervix is 2cm long, posterior, soft and the os is closed. The presenting part is 3cm above the spines. CTG is reactive.

Your answer: A Correct answer: I

| EXPLANATION |

Options for Questions 23-24 A Cystoscopy

B MRI scan

C Urodynamic studies

D Bladder re-training

E Urine for cytology

F Cystoscopy and biopsy

G Examination under anaesthesia

H MSU for culture and sensitivity

I

J

Urine electrolytes

Renal function tests

Instructions:For each of the case histories described below, choose the singlemost important investigation from the above list of options. Each option may be used once, more than once, or not at all.

Question 23

Question 24

A 60 year old woman developed cervical cancer 12 months earlier and was treated with combined chemotherapy and radiotherapy. She now complains of blood in her urine which occurs all the time A 45 year old woman complained initially of urinary frequency urgency and urge incontinence and also leaked urine on coughing or straining. She was treated medically initially and her symptoms of urinary frequency and urgency have largely resolved. Over the last 6 months, she is finding that urinary leakage on coughing and straining is increasingly affecting her social life.

Your answer: A Correct answer: F

Your answer: A Correct answer: C

| EXPLANATION | Options for Questions 25-26 A In-vitro fertilisation

B Intra-uterine insemination

C Laparoscopy and dye test

D Intra-cytoplasmic sperm injection

E Clomephene citrate

F

G Metformin

H Oocyte donation

I

Surrogacy

K Weight gain

Gonadotrophin induction of ovulation

J

Weight reduction

L

Measure serum androgen concentrations

Instructions:For each of the case histories described below, choose the singlemost appropriate initial management from the above list. Each option may be used once, more than once, or not at all.

Question 25

A 24 year old woman with her 35 year old partner has been referred to the infertility clinic because of 2 years of primary infertility. The woman has an irregular menstrual cycle every 2-4 months and her partner’s semen analysis is normal. The woman’s BMI is 26 and there is no significant past medical history. Investigations have confirmed a diagnosis of polycystic ovary. The couple have not achieved a pregnancy despite a 6 months course of clomephene citrate with evidence of ovulation based on day 21 progesterone concentration and ultrasound

Your answer: A Correct answer: C

scanning.

Question 26

A 34 year old woman with her 35 year old partner has been referred to the infertility clinic because of 2 years of primary infertility. The woman has a regular 28 day cycle and her periods are heavy with severe dysmenorrhoea. She also complains of deep dyspareunia and intermittent lower abdominal and pelvic pain. Her BMI is 27. Investigations have shown normal LH, FHS, Thyroid function tests, prolactin and day 21 progesterone concentration confirms ovulatory cycles. Laparoscopy and dye test shows evidence of chronic pelvic inflammatory disease with

Your answer: A Correct answer: A

bilateral tubal occlusion. Her partner’s semen analysis is normal. | EXPLANATION | Options for Questions 27-28 A Counsel and recommend appropriate contraception

B Counsel and offer termination of pregnancy

C Commence folic acid 0.4mg/day

D Commence folic acid 4mg/day

E Change anti-hypertensive drug to captopril

F

G Stop propylthiouracil

H Change therapy to carbimazole

I

Stop anti-hypertensive medication

J

Commence block and replace therapy

K Explain risk of neonatal thyrotoxicosis

L

Change therapy to propylthiouracil

M Increase dose of thyroxine

N Explain 20% risk of neonatal hypothyroidism

Reassure

Instructions:For each scenario described below, choose the singlemost appropriate management from the above list of options. Each option may be used once, more than once, or not at all. Question 27

Question 28

A 30 year old woman with Graves disease is adequately controlled on carbimazole is planning a pregnancy. She attends the pre-conception clinic enquiring about the risks to her fetus

A 25 year old insulin dependent diabetic with well controlled disease is planning a pregnancy and attends the pre-conception clinic. All observations and investigations are within normal limits.

Your answer: A Correct answer: K Your answer: A Correct answer: D

| EXPLANATION | Options for Questions 29-30 A TAH + BSO

B Vaginal hysterectomy

C Radical abdominal hysterectomy

D Laparoscopic assisted vaginal hysterectomy

E TAH + BSO + omentectomy G TAH

F BSO H Chemotherapy

I

J

Radiotherapy

Combined chemo-radiotherapy

K Endometrial biopsy Instructions:For each scenario described below, choose the singlemost appropriate management from the above list of options. Each option may be used once, more than once, or not at all.

Question 29

A 65 year old woman presents with a 6 months history of postmenopausal bleeding. Endometrial biopsy showed an endometrial adenocarcinoma and MRI scan shows that the tumour has infiltrated the outer 50% of the myometrium but does not involve the cervix or adnexal structures

Your answer: A Correct answer: A

Question 30

An asymptomatic 56 years old woman is referred to the gynaecology clinic because of a cervical smear showing atypical glandular cells. The trans-vaginal scan requested by the general practitioner shows a normal size uterus with a regular 8mm endometrium with normal ovaries.

Your answer: A Correct answer: K

| EXPLANATION | Options for Questions 31-32 A Maternal treatment with antibiotics

B Detailed fetal anomaly scan

C Deliver by caesarean section

D Active immunisation of neonate

E Avoid breast-feeding

F Counsel and offer termination of pregnancy

G Delay delivery by 5-7 days

H Administer corticosteroids

I

J

Administer tocolytics

Reassurance

K Screen for maternal syphilis infection Instructions:For each scenario described below, choose the singlemost appropriate management from the above list of options. Each option may be used once, more than once, or not at all.

Question 31

Question 32

A 20 year old woman presents with intermittent abdominal pain and a febrile illness at 20 weeks gestation. Blood cultures are positive for Listeria monocytogenes

A 30 year old woman has the following results following routine antenatal screening at 15 weeks gestation: VDRL positive, FTAabs positive

Your answer: A Correct answer: A Your answer: A Correct answer: A

| EXPLANATION | Options for Questions 33-34 A Cancel IVF cycle

B Freeze embryos

C Admit for iv fluids and thromboprophylaxis

D Termination of pregnancy

E Transfer to ITU

F

G Drain pleural effusion

H Drain ascites

I

Drain ovarian cysts

K Avoid unprotected sexual intercourse

Fluid restriction

J

Proceed with embryo replacement

L

Diuretics

Instructions:For each of the case histories described below, choose the singlemost appropriate management from the above list. Each option may be used once, more than once, or not at all.

Question 33

Question 34

A 35 year old woman with PCOS is undergoing IVF treatment. She attends for embryo replacement 6 days after HCG administration and complains of abdominal distension and pain with nausea and vomiting 2-3 times per day over the previous 48h. Clinical examination showed a mildly distended abdomen and ultrasound scan confirmed bilateral ovarian enlargement 810cm. All her blood tests were normal. A 35 year old woman with primary infertility is undergoing IVF treatment. She attends 10 days after embryo replacement and complains of abdominal distension and pain with nausea and persistent severe vomiting over the previous 48h. Clinical

Your answer: A Correct answer: B

Your answer: A Correct answer: C

examination showed a markedly distended abdomen with ascites and pleural effusion. Abdominal and pelvic ultrasound scan confirmed 12cm diameter bilateral cystic ovaries. All her blood tests were normal.

| EXPLANATION | Options for Questions 35-36 A Damage to bladder / ureter

B Damage to bowel

C Failure rate 1 in 200 E Failure to identify disease

D Failure to gain entry into abdominal cavity F Failure to visualise uterine cavity

G Haemorrhage requiring blood transfusion

H Haemorrhage requiring return to theatre

I

J

Laparotomy

Uterine perforation

Instructions:For each of the case histories described below, choose the singlemost relevant complication that you must discuss with the patient when taking consent prior to surgery from the above list of options. Each option may be used once, more than once, or not at all.

Question 35

Question 36

A 48-year-old nulliparous woman is scheduled for vaginal hysterectomy because of menorrhagia. Her uterus is enlarged equivalent to 14 weeks' gestation. A 30 year old woman with a 3 year history of abdominal and pelvic pain that has failed to respond to medical treatment. Her pain is unrelated to menses and she has no bowel symptoms. She is otherwise fit and well with a BMI of 25 and is listed for diagnostic laparoscopy.

Your answer: A Correct answer: I

Your answer: A

Correct answer: E

| EXPLANATION | Options for Questions 37-38 A Oestrogen cream

B Testosterone cream

C Oral HRT

D Topical anti-histamines

E Vaginoplasty

F Radical vulvectomy

G Skinning vulvectomy

H Topical anti-fungal cream

I

J

Clobetasol cream

Excision biopsy

Instructions:For each clinical scenario below choose the singlemost appropriate treatment from the list above. Each diagnosis may be used once, more than once, or not at all.

Question 37

Question 38

| EXPLANATION |

A 23-year-old woman presents with a two-year history of vulval, perineal and perianal irritation. The vulva is red, excoriated and there areas of white, thickened skin. Application of 3% Acetic acid shows areas of mosaic and coarse punctuation. A 47 year old woman complains of a 3 months history of vulval irritation and superficial dyapareunia. The vulval skin is thin and white with fissures and narrowing of the introitus and fusion of the labia minora over the clitoris

Your answer: A Correct answer: J

Your answer: A Correct answer: I

Options for Questions 39-40 A Tranexamic acid

B Danazol

C GnRH analogues

D Laparoscopy and ablation of endometriosis

E Diagnostic laparoscopy

F

G Pelvic ultrasound scan

H MRI scan of the pelvis

I

Mefenamic acid

J

Endocervical and urethral swabs

K Combined oral contraceptive pill

L

Depo-medroxyprogesterone acetate

M Refer to pain clinic

N Refer to psychiatrist

Dilatation and curettage

Instructions:For each of the case histories described below, choose the singlemost appropriate management from the above list. Each option may be used once, more than once, or not at all.

Question 39

Question 40

A 35 year old woman attends the gynaecology clinic with her 14 year old daughter who is complaining of severe period pains since the onset of menstruation at the age of 13 years. Menstrual loss is not heavy and she has no bowel symptoms. Her symptoms are now interfering with her education as she takes several days off school every month. She does not require contraception and clinical examination is normal. A 33 year old mother of 3 children complains of progressively painful periods over 5 years. There is occasional deep dyspareunia but she has no bowel symptoms. Her symptoms have not responded to simple analgesia and non-steroidal antiinflammatory drugs. Clinical examination is normal.

Your answer: A Correct answer: F

Your answer: A Correct answer: K

Options for Questions 1-2 A Raloxifene

B Oral sequential oestrogen + progestogen

C Oral continuous oestrogen + progestogen

D Oestrogen patches

E Vaginal oestrogen tablets

F Levonorgestrel IUS + oral oestrogens

G Levonorgestrel IUS

H Refer to special oncology clinic

I

J

Counsel and recommend discontinuation of HRT

HRT not recommended

Instructions: For each scenario described below, choose the single most appropriate management from the above list of options. Each option may be used once, more than once, or not at all.

Question 1

Question 2

A 60 year old woman complains of vaginal dryness and superficial / deep dyspareunia since discontinuing oral HRT

Your answer: A

because of media reports. She has used lubricants without improvement

Correct answer: E

A 40 year old woman has TAH + BSO for stage Ia endometroid adenocarcinoma of the ovary and complains of debilitating hot flushes.

| EXPLANATION | Options for Questions 3-4

Your answer: A Correct answer: D

Administer VZIG to mother if maternal serology ve

A Administer VZIG as soon as possible to mother

B

C Administer VZIG to neonate

D Detailed ultrasound examination

E

Immediate caesarean section and transfer baby to the neonatal unit

G Induction of labour I

F

Advise avoid contact with other pregnant women and neonates

H Reassurance

Separate mother and baby after delivery

K Give intravenous Aciclovir

J

Serum for VZV IgM antibodies

L

Treat with oral Aciclovir

Instructions: For each scenario described below, choose the single most appropriate management from the above list of options. Each option may be used once, more than once, or not at all.

Question 3

Question 4

A 26-year-old Para 1+ 0 at 38 weeks gestation contacts her GP immediately after hearing that a child in her son's nursery has developed chickenpox. On direct questioning, she admits to having had chicken pox as a child.

A 26-year-old Para 1+ 0 at 38 weeks gestation contacts her GP immediately after discovering that her husband has shingles. She has no memory of having chicken-pox in the past.

Your answer: A Correct answer: H

Your answer: A Correct answer: B

| EXPLANATION | Options for Questions 5-6 A Genital swabs for viral culture

B Immediate induction of labour

C Delay delivery by 5-7 days if possible

D Offer prophylactic oral acyclovir

E Treat with intra-venous acyclovir

F

G Treat neonate with intravenous acyclovir

H Reassurance

I

Avoid breast-feeding

K Refer to genito-urinary medicine clinic

Deliver by caesarean section

J

Avoid contact with other pregnant women and neonates

L

Screen for other sexually transmitted infections

Instructions: For each scenario described below, choose the single most appropriate management from the above list of options. Each option may be used once, more than once, or not at all.

Question 5

A 17 year old woman complains of vulval soreness at 18 weeks gestation. She is found to have vulval herpetic vesicles on examination. There is no history of previous genital herpes.

Question 6

A 20 year old woman presents in spontaneous labour at 37 weeks gestation. She developed primary genital herpes at 34 weeks gestation but is now symptom-free.

| EXPLANATION | Options for Questions 7-8 A Amniocentesis and karyotype

B Amniocentesis and PCR

Your answer: A Correct answer: K

Your answer: A Correct answer: F

C CVS and karyotype

D CVS and PCR

E Amniocentesis and enzyme assay

F Fetal blood sampling and karyotype

G Fetal blood sampling and measurement of PO2

H Fetal blood sampling and haemoglobin electrophoresis

I

Fetal blood sampling and haemoglobin concentration

J

Fetal blood sampling and viral culture

K Reassurance

Instructions: For each of the case histories described below, choose the single most appropriate management from the above list of options. Each option may be used once, more than once, or not at all.

Question 7

Question 8

A 30 year old woman with myotonic dystrophy is 11 weeks pregnant and would like to know whether or not her fetus has inherited the condition

A 21 year old woman with 3 previous consecutive first trimester miscarriages is found to be a t(14:21) balanced translocation carrier. She is 10 weeks pregnant and would like to know whether or not her fetus has inherited an unbalanced translocation.

Your answer: A Correct answer: D

Your answer: A Correct answer: C

| EXPLANATION | Options for Questions 9-10 A Commence iv oxytocin

B Deliver by caesarean section

C Fetal blood sampling

D

No additional intervention at this stage

E Umbilical artery Doppler

F

Reduce oxytocin dose

G Uterine artery Doppler

H Forceps delivery

I

Intravenous ritodrine

K Sub-cutaneous terbutalline

J

Administer maternal facial oxygen

L

Administer uterotonic agent

Instructions: For each of the case histories described below, choose the single most appropriate initial management from the above list. Each option may be used once, more than once, or not at all.

Question 9

You have been asked to review a 35 year old woman half an hour after spontaneous vaginal delivery of her first child because the membranes are ragged although the placenta appears complete. The uterus is well contracted, the lochia are normal and all maternal observations are satisfactory.

Question 10

A 35 year old mother of two children presents in spontaneous labour at 39 weeks gestation. She has one previous caesarean section for breech presentation followed by a spontaneous vaginal delivery. At 12:00, the cervix is 7cm dilated with a direct occipito-anterior position with 3:10 strong contractions. At 16:00, the cervix is 7cm dilated and the contractions are inco-ordinate occurring 2-3:10. The CTG is normal

| EXPLANATION |

Your answer: A Correct answer: D

Your answer: A Correct answer: B

Options for Questions 11-12 A Oestrogen cream

B Testosterone cream

C Oral HRT

D Topical anti-histamines

E Vaginoplasty

F Radical vulvectomy

G Skinning vulvectomy

H Topical anti-fungal cream

I

J

Clobetasol cream

Excision biopsy

Instructions: For each clinical scenario below choose the single most appropriate treatment from the list above. Each diagnosis may be used once, more than once, or not at all.

Question 11

Question 12

A 23-year-old woman presents with a two-year history of vulval, perineal and perianal irritation. The vulva is red, excoriated and there areas of white, thickened skin. Application of 3% Acetic acid shows areas of mosaic and coarse punctuation.

A 47 year old woman complains of a 3 months history of vulval irritation and superficial dyapareunia. The vulval skin is thin and white with fissures and narrowing of the introitus and fusion of the labia minora over the clitoris

Your answer: A Correct answer: J

Your answer: A Correct answer: I

| EXPLANATION | Options for Questions 13-14 A Administer regional analgesia if APTT is normal C

Administer protamine sulphate then regional analgesia

B Administer regional analgesia if APTT and PT are normal D Advice that regional analgesia is contra-indicated

E Administer regional analgesia

F

G Remove epidural catheter

H

Check APTT then remove epidural catheter if result is normal

I

J

Check anti-Xa levels then administer regional analgesia if normal

Advise against removal of epidural catheter

Administer prophylactic dose of heparin

K Wait for 12h then administer heparin

Instructions: For each scenario described below, choose the single most appropriate management from the above list of options. Each option may be used once, more than once, or not at all.

Question 13

Question 14

A 40 year old woman with a BMI of 39 has had an emergency caesarean section for failure to progress at 6cm dilatation under combined spinal epidural (CSE) analgesia. The procedure was uncomplicated with a blood loss of 700ml. A prophylactic does of LMWH was administered at noon, 6h after insertion of CSE. She requests removal of the epidural catheter at 18:00h.

A 25 year old woman with a BMI of 38 has an emergency caesarean section at full dilatation under spinal anaesthesia because of fetal distress. The procedure was uncomplicated with blood loss of 700ml. She is now 8h post-surgery and your attention is drawn to instructions on post-partum

Your answer: A Correct answer: I

Your answer: A Correct answer: F

thromboprophylaxis.

| EXPLANATION | Options for Questions 15-16 A Quadruple test

B Integrated test

C Nuchal transluscency

D Serum integrated test

E Triple test

F Anomaly scan

G Fluorescent in-situ hybridisation

H Amniocentesis

I

Chorionic villus sampling

K No investigation

J

PCR

L

Fetal blood sampling

Instructions: For each of the case histories described below, choose the single most important investigation from the above list of options. Each option may be used once, more than once, or not at all.

Question 15

Question 16

A 30 year old woman has one previous termination of pregnancy at 22 weeks gestation for Edward’s syndrome now presents at 10 weeks gestation.

A 30 year old woman is found to have a fetus with duodenal atresia on anomaly scan at 20 weeks gestation

Your answer: A Correct answer: I

Your answer: A

Correct answer: H

| EXPLANATION | Options for Questions 17-18 A In-vitro fertilisation

B Intra-uterine insemination

C Laparoscopy and dye test

D Laparoscopic ovarian drilling

E Clomephene citrate

F

G Metformin

H Carbegolline

I

Hystero-salpingogram

K Weight gain

Gonadotrophin induction of ovulation

J

Weight reduction

L

Measure serum androgen concentrations

Instructions: For each of the case histories described below, choose the single most appropriate initial management from the above list. Each option may be used once, more than once, or not at all.

Question 17

Question 18

A 34 year old woman with her 35 year old partner has been referred to the infertility clinic because of 2 years of primary infertility. The woman has a regular and normal 28 day cycle, her BMI is 26 and she has no other symptoms. Investigations have shown normal LH, FHS, Thyroid function tests, prolactin and day 21 progesterone concentration confirms ovulatory cycles. Her partner’s semen analysis is normal.

A 36 year old woman with her 35 year old partner has been referred to the infertility clinic because of 2 years of primary

Your answer: A Correct answer: I

Your answer: A

infertility. The woman has a regular and normal 28 day cycle, her BMI is 26 and she has no other symptoms. Investigations have shown normal LH = 7.5mIU/ml, FHS = 17mIU/ml, Thyroid function tests, prolactin and day 21 progesterone concentration confirms ovulatory cycles. Her partner’s semen analysis is normal. Hystero-salpingogram confirms bilateral patent fallopian tubes.

Correct answer: B

| EXPLANATION | Options for Questions 19-20 Administer VZIG to neonate and avoid breastfeeding

A Avoid breast-feeding

B

C Administer VZIG to neonate but breast-feeding encouraged

D Immediate induction of labour

E Delay delivery with tocolytics

F Offer termination of pregnancy

G Treat neonate with oral acyclovir

H Treat neonate with intravenous acyclovir

I

Examine neonate for evidence of congenital anomalies

K Delay delivery for 5-7 days if possible

J

Reassurance

L

Detailed ultrasound examination

Instructions: For each scenario described below, choose the single most appropriate management from the above list of options. Each option may be used once, more than once, or not at all.

Question 19

Question 20

A 20 year old woman is in hospital 2 days after vaginal delivery of her first baby and is informed that her 8 year old step-daughter has chicken-pox. She does not remember having chicken pox in the past and is worried about taking her new baby home. Investigations show that she is varicella zoster IgG positive and IgM negative

A 20 year old woman is being treated in the infectious diseases unit for chicken pox and reports having missed a period with a positive pregnancy test. Dating scan shows that she has a viable 11 week pregnancy and she is discharged following resolution of her symptoms

Your answer: A Correct answer: J

Your answer: A Correct answer: L

| EXPLANATION | Options for Questions 21-22 A Serial beta-HCG assay

B Laparoscopy

C Repeat trans-vaginal scan in 24h

D Repeat trans-vaginal scan in 7 days

E Counsel and discharge

F

G Offer medical treatment with methotrexate

H Laparotomy

I

Serum progesterone assay

K Refer to molar pregnancy centre

Offer surgical evacuation of products of conception

J

Serum AFP assay

L

Refer to other specialty

Instructions: Instructions: For each patient described below choose the single most appropriate initial management option from the list. Each option may be used once, more

than once, or not at all.

Question 21

Question 22

An asymptomatic 30 year old woman with a previous ectopic pregnancy presents at 6 weeks amenorrhoea for trans-vaginal scan to exclude a repeat ectopic pregnancy. The scan is reported to show a 14mm endometrium with no gestation sac. There are no adnexal masses and no free fluid in the pouch of Douglas. HCG concentration is 500IU and 1600IU 48h later

An asymptomatic 30 year old woman with a previous ectopic pregnancy presents at 6 weeks amenorrhoea for trans-vaginal scan to exclude a repeat ectopic pregnancy. Her pregnancy test had been positive 2 weeks earlier. The scan is reported to show a 14mm endometrium with no gestation sac. There is a 4cm cystic structure in the right adnexum suggestive of an ectopic pregnancy but no free fluid in the pouch of Douglas

Your answer: A Correct answer: D

Your answer: A Correct answer: B

| EXPLANATION | Options for Questions 23-24 A Bilateral oophrectomy

B CA-125 assays every 6-12 months

C TAH + BSO

D Cyst aspiration for cytology

E Refer to cancer centre

F Refer to palliative care team

G Reassure

H Measure serum CA-125

I

J

Yearly pelvic ultrasound scans

Repeat pelvic ultrasound scan in 4-6 months

Instructions: For each scenario described below, choose the single most appropriate management from the above list of options. Each option may be used once, more than once, or not at all.

Question 23

Question 24

A 40 year old woman presents with a 12 months history of vague abdominal discomfort that did not respond to simple analgesia. Ultrasound scan shows bilateral complex ovarian cysts with right sided hydronephrosis and ascites. Her CA-125 is 1500iu

A 20 year old woman is admitted with sudden onset left sided pelvic pain 23 days after her LMP. Her pregnancy test is negative and her symptoms are improving with simple analgesia. Pelvic ultrasound scan shows a 5cm left ovarian cyst with internal echoes consistent with a haemorrhagic cyst.

Your answer: A Correct answer: E

Your answer: A Correct answer: J

| EXPLANATION | Options for Questions 25-26 A Closure of visceral peritoneum

B Closure of parietal peritoneum

C Closure of Scarpa’s fascia

D Non-closure of the rectus sheath

E Instillation of local anaesthetic into pelvis

F Supra-pubic catheterisation

G Mass closure of abdominal incision

H Prophylactic heparin 2h pre-op

I

Application of methylene blue to vagina

K Excision of vaginal cuff

J

Closure of vaginal vault

L

Use of prophylactic antibiotics after delivery

Instructions: For each of the case histories described below, choose the single most appropriate intervention that would reduce peri-operative morbidity and mortality from the above list. Each option may be used once, more than once, or not at all.

Question 25

Question 26

A 25 year old woman is undergoing an emergency caesarean section at 9cm dilatation for failure to progress

A 42 year old woman is to undergo TAH for dysfunctional uterine bleeding. She is otherwise fit and well with a BMI of 35.

Your answer: A Correct answer: L

Your answer: A Correct answer: H

| EXPLANATION | Options for Questions 27-28 A Atrophic vulvovaginitis

B Human papilloma virus infection

C Benign mucous membrane pemphigoid

D Candida infection

E Lichen sclerosus

F Contact dermatitis

G Lichen simplex et chronicus

H Herpes simplex infection

I

J

Vulval intraepithelial neoplasia

Vulvodynia

Instructions: each clinical may scenario below choose single most likely diagnosis from the list above.For Each diagnosis be used once, morethe than once, or not at all.

Question 27

Question 28

A 23-year-old woman presents with a two-year history of vulval, perineal and perianal irritation. The vulva is red, excoriated and there areas of white, thickened skin. Application of 3% Acetic acid shows areas of mosaic and coarse punctuation.

A 78-year-old woman presents with vulval irritation and soreness. On examination the vulva is red in colour, slightly oedematous and there are small, red papules scattered randomly beyond the perimeter of the vulva. She also complains of soreness and irritation under the breasts.

Your answer: A Correct answer: I

Your answer: A Correct answer: D

| EXPLANATION | Options for Questions 29-30 A No additional intervention at this stage

B Deliver by caesarean section

C Increase dose of oxytocin

D Lovset’s manoeuvre

E Decrease dose of oxytocin

F

G Forceps delivery

H Delivery of the posterior arm

I

Episiotomy

K McRobert’s manoeuvre

Fetal blood sampling

J

Inhaled salbutamol

L

Administer maternal facial oxygen

Instructions: For each of the case histories described below, choose the single most appropriate management from the above list. Each option may be used once, more than

once, or not at all.

Question 29

Question 30

A 34 year old woman with 3 previous vaginal deliveries presents in spontaneous labour at 39 weeks gestation. The cervix is fully dilated with thick meconium stained liquor. The CTG shows a base-line fetal heart rate of 135bpm with variability of 3-4bpm with variable decelerations. There are 3-4 uterine contractions every 10 minutes. The fetus is in a direct occipito-anterior position 1cm below the ischial spines.

You have been called into a delivery room by the senior midwife because of difficulties delivering the fetal shoulders. On your arrival, the woman is in lithotomy and the head had been delivered 2 minutes earlier but moderate traction has thus far failed to deliver the shoulders.

Your answer: A Correct answer: G

Your answer: A Correct answer: K

| EXPLANATION | Options for Questions 31-32 A Fetal blood sampling

B Triple test

C Quadruple test

D Integrated test

E Chorionic villus sampling

F

G Amniocentesis

H Nuchal transluscency

I

J

Fluorescent in-situ hybridisation

L

Restriction fragment length polymorphisms

Anomaly scan

K PCR M Karyotype

Serum integrated test

Instructions: For each of the conditions described below, choose the single most appropriate diagnostic tests from the above list of options. Each option may be used once, more than once, or not at all. Your answer: A Question 31

Fetal anaemia Correct answer: A

Your answer: A Question 32

Fetal gastroschisis Correct answer: I

| EXPLANATION | Options for Questions 33-34 A Emergency caesarean section

B Ultrasound scan for placental site

C Induction of labour with prostaglandins

D Induction of labour by amniotomy

E Oxytocin augmentation of labour

F

G Vaginal operative delivery

H Maternal blood transfusion

I

Fetal scalp blood sampling

Transfer to high dependency unit

J

Expectant management

K Arrange antenatal clinic follow-up

L

Treatment with tocolytics

M Umbilical artery Doppler

N Perform Kleihauer test

Instructions: For each of the case histories described below, choose the single most appropriate management from the above list of options. Each option may be used once, more than once, or not at all.

Question 33

Question 34

A 42 year old woman had an elective caesarean section at 37 weeks gestation for major placenta previa. She returned to theatre 6h later because of suspected intra-abdominal bleeding and has undergone a total abdominal hysterectomy

A 35 year old Rhesus positive woman presented with fresh vaginal bleeding and intermittent abdominal pain at 30 weeks gestation. Maternal pulse on admission was 90bpm with BP 120/80. The fetal heart rate was normal. She has been in hospital for 48h and complains of a slight brown discharge but no other symptoms and has had no further bleeding.

Your answer: A Correct answer: I

Your answer: A Correct answer: K

| EXPLANATION | Options for Questions 35-36 A Increase iv fluids

B Insert central venous pressure line

C Intravenous magnesium sulphate

D

Measure serum aspartate transaminase immediately

E Measure serum magnesium

F Blood transfusion

G Monitor patellar reflex every 15 minutes I Provide intravenous Hartmann

H Provide a fluid challenge with colloids J Decrease iv fluids

K

Immediate dose of 10ml 10% calcium gluconate intravenously

L

Carry out visual field assessment

Instructions: For each patient described below choose the single most appropriate management option from the list. Each option may be used once, more than once, or not at all.

Question 35

Question 36

A 20-year-old primigravida is 30 weeks' pregnant and has been transferred to the delivery suite with severe gestational proteinuric hypertension. She complains of severe frontal headache but has no other symptoms. She has a normal respiratory rate and her urine output has been satisfactory. Her blood pressure is 140/100mmHg. There are five beats of bilateral ankle clonus.

A 20-year-old primigravida delivered a live infant 5 hours previously by emergency caesarean section. She has developed severe gestational proteinuric hypertension. Blood loss at caesarean section was estimated at 1,000 ml. BP = 160/100mmHg on oral anti-hypertensive treatment and pulse = 95bpm. She has been given one litre of Hartmann's solution intravenously since her delivery but has passed 40ml of urine since delivery. Her clotting, liver enzymes and serum creatinine are normal.

| EXPLANATION |

Your answer: A Correct answer: C

Your answer: A Correct answer: B

Options for Questions 37-38 A Uterine perforation

B Laparotomy

C Bladder injury

D Bleeding requiring transfusion

E Continuing pregnancy

F Infertility

G Wound infection

H Failure to identify any pathology

I

J

Failure to gain access to abdominal cavity

Risk of failure of the procedure

Instructions: For each of the case histories described below, choose the single most relevant complication that you must discuss with the patient when taking consent prior to surgery from the above list of options. Each option may be used once, more than once, or not at all.

Question 37

Question 38

A 34 year old woman with a previous left salpingectomy for ectopic pregnancy presents with abdominal pain and vaginal bleeding and is found to have an empty uterus with a right adnexal mass consistent with an ectopic pregnancy.

A 35 year old woman with a 10 week missed miscarriage is undergoing evacuation of retained products of conception under general anaesthesia

Your answer: A Correct answer: F

Your answer: A Correct answer: A

| EXPLANATION | Options for Questions 39-40 A Rocker bottom feet

B Exomphalos

C Holoprosencephaly

D Cystic hygroma

E Gastroschisis

F

G Intra-cranial calcification

H Duodenal atresia

I

J

Spina bifida

atrio-ventricular septal defect Polyhydramnios

K Echogenic bowel

Instructions: For each of the conditions described below, choose the single most characteristic abnormality from the above list of options. Each option may be used once, more than once, or not at all. Your answer: A Question 39

Turner’s syndrome Correct answer: D

Your answer: A Question 40

Trisomy 13 Correct answer: C

Your score is 8% for this EMQ Exam You answered 3 out of 40 questions correctly. 40 random questions for EMQ Mock Exam

| EXPLANATION | Options for Questions 1-2

A No additional intervention at this stage

B Deliver by caesarean section

C Evacuation of retained products of conception

D Artificial rupture of membranes

E Bimanual compression of the uterus

F

G I

Controlled artificial rupture of membranes in theatre

H Intra-muscular analgesia J

Prostaglandin induction of labour

K Sub-cutaneous terbutalline

Venous access and resuscitation

L

Manual replacement of the uterus Commence iv oxytocin

Welcome SEMSEM Your last visit was on Wednesday Apr 05th, 2006 You have been a member for 1 days. Account Manager •

Instructions:For each of the case histories described below, choose the single most appropriate management from the above list. Each option may be used once, more than once, or not at all. You have been asked to review a 25 year old woman half an hour after spontaneous vaginal delivery of her first child because a placental Question 1 cotyledon is retained. The uterus is well contracted, the lochia are normal and all maternal observations are satisfactory.

• • • •



Edit my details MCQ test scores Mock MCQ scores EMQ test scores Mock EMQ scores Logout

Your answer: A Correct answer: C

BUSY SpR Home About us

A 23 year old woman with two previous vaginal deliveries is admitted for induction of labour at Your answer: A 42 weeks gestation. The pregnancy has been Question 2 uncomplicated and the fetal head is three fifths Correct palpable. The cervix is 1cm long, central, 2cm answer: D dilated with bulging membranes and the fetal head is 2cm above the ischial spines.

Signup/Join Lost password? Medical forums Contributors Bibliography Contact us

| EXPLANATION |

Recommend us Options for Questions 3-4

Privacy

A Tranexamic acid

B Danazol

C GnRH analogues

D TAH + BSO

E Diagnostic laparoscopy

F

G Pelvic ultrasound scan

H MRI scan of the pelvis

I

J

Dilatation and curettage

Medroxyprogesterone acetate Endocervical and urethral swabs

Combined oral contraceptive Levonorgestrel releasing intra-uterine K L pill system M Refer to pain clinic N Refer to gastroenterologist Instructions:For each of the case histories described below, choose the single

most appropriate management from the above list. Each option may be used once, more than once, or not at all. A 22 year old woman is known to have endometriosis diagnosed at laparoscopy. She is referred to the gynaecology clinic because of Your answer: A progressively severe pelvic pain, Question 3 dysmenorrhoea and deep dyspareunia. There Correct is a past medical history of focal migraine. answer: F Clinical examination showed a tender pelvis with a normal size anteverted uterus and no adnexal masses. A 22 year old woman is referred to the gynaecology clinic because of a 2 year history of persistent pelvic pain which is worse during and after intercourse and during menstruation. Your answer: A She also complains of intermittent abdominal Question 4 bloating with alternating constipation and Correct diarrhoea and urgency of defecation. There is answer: N occasional rectal bleeding which is unrelated to her menses and usually occurs during episodes of constipation. Clinical examination is normal and there is no significant past medical history.

| EXPLANATION | Options for Questions 5-6

A Raloxifene

B Oral sequential oestrogen + progestogen

C Oral continuous oestrogen + progestogen

D Oestrogen patches

E Vaginal oestrogen tablets

F Tamoxifen

G Levonorgestrel IUS

H Refer to special oncology clinic

I

Counsel and recommend discontinuation of HRT

J

HRT not recommended

Instructions:For each scenario described below, choose the singlemost appropriate management from the above list of options. Each option may be used once, more than once, or not at all.

A 40 year old woman has TAH + BSO for stage Question 5 Ia ovarian cancer and complains of distressing hot flushes and vaginal dryness

Question 6

Your answer: A Correct answer: D

Your answer: A A 52 year old woman with a previous PE complains of infrequent periods and distressing hot flushes. Her sister died from PE while on Correct the COCP at the age of 38 years. answer: J

| EXPLANATION | Options for Questions 7-8 A Urinary retention

B Damage to bowel

C Removal of ovaries

D Failure to gain entry into abdominal

cavity E Failure to identify disease

F Failure to visualise uterine cavity

G Haemorrhage requiring blood transfusion I Laparotomy

H Haemorrhage requiring return to theatre J Uterine perforation

Instructions:For each of the case histories described below, choose the single most relevant complication that you must discuss with the patient when taking consent prior to surgery from the above list of options. Each option may be used once, more than once, or not at all.

A 30 year old woman with genuine stress Question 7 incontinence who is due to have a tension-free vaginal tape

Your answer: A Correct answer: A

A 30 year old woman with primary infertility who Your answer: A is due to undergo IVF treatment and is Question 8 scheduled to have laparoscopy and ovarian Correct cystectomies for severe endometriosis with answer: C bilateral ovarian endometriomas.

| EXPLANATION | Options for Questions 9-10 A Abdominal pressure due to mass

B Bladder neck weakness

C Pelvic floor muscle damage

D Congenital weakness of the pelvic floor

E Obstetric trauma

F

G Neurological disease

H Drug side-effects

I

Tumour infiltration into bladder

K Schistosomiasis

Oestrogen deficiency

J

Bacterial urinary tract infection

L

Radiation injury

Instructions:For each of the case histories described below, choose the single most likely cause of urinary tract symptoms from the above list of options. Each option may be used once, more than once, or not at all. A 35 year old woman complains of a 2 month history of urinary frequency, urgency and haematuria. She had an abdominal X-ray 2 Question 9 weeks earlier for an unrelated complaint which showed evidence of bladder calcification. Cystoscopy showed numerous polypoid lesions in the bladder

Your answer: A Correct answer: K

A 52 year old woman complains of progressively worsening urinary frequency and Your answer: A urgency 3 years after radical abdominal Question 10 hysterectomy and radiotherapy for cervical Correct cancer. There is no haematuria and all answer: F investigations including cystoscopy are normal

| EXPLANATION |

Options for Questions 11-12 A Commence oral warfarin

B Await results of D-dimers assay

C Commence therapeutic dose of heparin

D Prophylactic dose of heparin for 3-5 days post-partum

E

Antenatal prophylactic treatment with heparin

G

Prophylactic heparin for 6 weeks H Continue s/c heparin for 6 weeks post-partum

I

Stop heparin therapy

K No treatment required

F

J

Antenatal treatment with warfarin

Continue heparin therapy for 6

months Continue heparin prophylaxis during L labour

Instructions:For each scenario described below, choose the singlemost appropriate management from the above list of options. Each option may be used once, more than once, or not at all. A 20 year old woman is known to be a carrier of Your answer: A the factor V Leiden mutation following screening because of a DVT in her sister. She Question 11 attends the antenatal clinic at 15 weeks in her Correct first pregnancy. Her BMI is 22 and she is answer: G otherwise fir and well. A 30 year old woman complains of a painful swollen left lower limb at 32 weeks gestation Question 12 and is treated with heparin. Subsequent

Your answer: A

Doppler studies have shown a left popliteal vein thrombosis.

Correct J answer:

| EXPLANATION | Options for Questions 13-14 A Stop heparin on the evening before B Commence oral warfarin induction C Await results of V/Q scan then commence treatment

D Await results of venogram then commence treatment

E Await results of D-dimers assay

F

G I

Commence prophylactic dose of heparin post-partum Antenatal prophylactic treatment

with heparin Prophylactic heparin for 6 weeks K post-partum

Commence therapeutic dose of heparin

H Oral aspirin therapy J

Antenatal treatment with warfarin

L

Stop heparin therapy

Instructions:For each scenario described below, choose the singlemost appropriate management from the above list of options. Each option may be used once, more than once, or not at all. A 30 year old woman with a previous DVT Your answer: A presents for antenatal care at 15 weeks Question 13 gestation. There is no family history of VTE and Correct her BMI is 25. She is otherwise fit and well with answer: K a negative thrombophilia screen

A 30 year old woman with recurrent first trimester miscarriages is known to have the anti-phospholipid antibody syndrome and has Question 14 had a spontaneous vaginal delivery at 39 weeks gestation. Her BMI is 27 and she is otherwise fit and well with no family history of VTE.

Your answer: A Correct answer: G

| EXPLANATION | Options for Questions 15-16

A Take pill as soon as possible

B Take pill as soon as possible + barrier contraception for 7 days

Take pill as soon as possible and Administer emergency contraceptive D omit pill-free week pill E Insert copper IUCD F Await results of pregnancy test

C

G Replace detached patch with a new patch

H Replace with a new patch and use barrier contraception for 7 days

I

Apply new patch and use barrier contraception for 7 days

J

Apply new patch but keep the same change date

K

Apply new patch and delay change day by 24h

L

Apply new patch and delay change day by 36h

Instructions:For each of the case histories described below, choose the single most appropriate advice from the above list. Each option may be used once, more than once, or not at all. A 30 year old nulliparous woman is using the Your answer: A combined contraceptive patch. She placed her Question 15 second patch at 10pm before going to bed and Correct having sexual intercourse and realised at 10am answer: J that the patch had detached. A 29 year old mother of 3 children is using the combined contraceptive patch. She placed her second patch at 10pm on a Wednesday and Question 16 realised 3 days later that the patch had detached. She had sexual intercourse on several occasions.

Your answer: A Correct answer: I

| EXPLANATION | Options for Questions 17-18 A Chorionic villus sampling

B Amniocentesis

C Fetal blood sampling

D Triple test

E Karyotype

F Integrated test

G Western blotting

H Northern blotting

I

J

Anomaly scan

Southern blotting

K PCR Instructions:For each of the case histories described below, choose the single most relevant investigation from the above list of options. Each option may be used once, more than once, or not at all.

A 35 year old woman with myotonic dystrophy Question 17 is undergoing IVF treatment in order to ensure that the disease is not passed on to her child

A 35 year old woman is found to have raised maternal serum AFP at 16 weeks gestation. Question 18 Anomaly scan is equivocal because of high body mass index and she has opted to have amniocentesis

Your answer: A Correct answer: K

Your answer: A Correct answer: G

| EXPLANATION | Options for Questions 19-20 A Tranexamic acid

B Danazol

C GnRH analogues

D

Laparoscopy and ablation of endometriosis

E Diagnostic laparoscopy

F

G Pelvic ultrasound scan

H MRI scan of the pelvis

I

Mefenamic acid

J

Endocervical and urethral swabs

K Combined oral contraceptive pill

L

Depo-medroxyprogesterone acetate

M Refer to pain clinic

N Refer to psychiatrist

Dilatation and curettage

Instructions:For each of the case histories described below, choose the single most appropriate management from the above list. Each option may be used once, more than once, or not at all. A 35 year old woman attends the gynaecology clinic with her 14 year old daughter who is complaining of severe period pains since the Your answer: A onset of menstruation at the age of 13 years. Menstrual loss is not heavy and she has no Question 19 bowel symptoms. Her symptoms are now Correct interfering with her education as she takes answer: F several days off school every month. She does not require contraception and clinical examination is normal. A 33 year old mother of 3 children complains of progressively painful periods over 5 years. Your answer: A There is occasional deep dyspareunia but she Question 20 has no bowel symptoms. Her symptoms have Correct not responded to simple analgesia and nonsteroidal anti-inflammatory drugs. Clinical examination is normal.

answer: K

| EXPLANATION | Options for Questions 21-22 A Counsel and offer evacuation of retained products of conception

B Counsel and offer support group

Counsel and offer laparoscopy & salpingectomy

D

E Counsel and offer treatment with

F

C

Counsel and offer investigation for recurrent miscarriage Counsel and offer laparotomy &

methotrexate G I

salpingectomy

Counsel and offer termination of pregnancy

H Perform salpingectomy J

Proceed to laparotomy

Serial HCG assay

K Counsel and refer to early pregnancy L Repeat trans-vaginal scan 7 days assessment unit later Instructions:For each patient described below choose the singlemost appropriate initial management option from the list. Each option may be used once, more than once, or not at all. A 25 year old woman has a transvaginal scan Your answer: A following IVF treatment during which 2 embryos Question 21 were replaced and is found to have a 6 week Correct singleton intra-uterine pregnancy with no fetal answer: A heart activity. A 25 year old nulliparous woman has had 2 Your answer: A previous first trimester miscarriages presents at Question 22 8 weeks gestation for an early pregnancy scan. Correct She is found to have a 7 week missed answer: A miscarriage.

| EXPLANATION | Options for Questions 23-24 A Congenital adrenal hyperplasia

B Maternal androgen therapy

C Down’s syndrome

D Turner’s syndrome

E Maternal diabetes mellitus

F

G Maternal warfarin therapy

H Maternal heparin therapy

I

Group B streptococcus infection

K Maternal smoking

Maternal lithium therapy

J

Maternal anti-epileptic drug therapy

L

Maternal cocaine abuse

Instructions:For each of the case histories described below, choose the single most likely cause of neonatal collapse from the above list of options. Each option may be used once, more than once, or not at all.

A 6 hour old neonate delivered at 35 weeks Question 23 gestation following prolonged pre-term rupture of the membranes

A 6 hour old neonate delivered by rotational forceps delivery for prolonged second stage of Question 24 labour. Initial examination showed epicanthic folds with up-slanting palpebral fissures and a loud systolic murmur

Your answer: A Correct answer: I

Your answer: A Correct answer: C

| EXPLANATION | Options for Questions 25-26 A Endometriosis

B Acute PID

C Chronic PID

D Irritable bowel syndrome

E Residual ovary syndrome

F

Ovarian remnant syndrome

G Diverticular disease I

H Interstitial cystitis J

Adenomyosis

K Uterine fibroids

L

Polycystic ovary syndrome

M Inflammatory bowel disease

N Chronic constipation

Primary dysmenorrhoea

Instructions:For each of the case histories described below, choose the single most likely cause of chronic pelvic pain from the above list. Each option may be used once, more than once, or not at all. A 23 year old woman has been referred to the gynaecology clinic with a 6 months history of lower abdominal and pelvic pain and deep Your answer: A dyspareunia. Her symptoms are particularly Question 25 worse during menstruation. Over the last 4 Correct months, she has experienced episodes of answer: M bloody diarrhoea and has lost about 5kg in weight. Her Hb = 10.0g/dl, white cell count = 15 x 10E9/l and platelet count = 450 x 10E9/l. A 35 year old woman complains of a 2 year history of progressively painful periods with occasional deep pain during intercourse. She Your answer: A denies any bowel or urinary symptoms. Clinical Question 26 examination showed a bulky smooth and Correct tender uterus with no adnexal masses and no answer: J adnexal tenderness. Her symptoms have not responded to non-steroidal anti-inflammatory agents and diagnostic laparoscopy was normal.

| EXPLANATION | Options for Questions 27-28 A Commence iv oxytocin

B Deliver by caesarean section

Evacuation of retained products of C conception

D

Artificial rupture of membranes

E Bimanual compression of the uterus

F

Venous access and resuscitation

G Examination under spinal anaesthesia

H Intra-muscular analgesia

I

J

Intravenous ritodrine

K Sub-cutaneous terbutalline

L

Manual replacement of the uterus Administer uterotonic agent

Instructions:For each of the case histories described below, choose the single most appropriate management once, more than once, or not atfrom all. the above list. Each option may be used A 34 year old woman is bleeding heavily during Your answer: A the third stage of labour. Her pulse is 60bpm and systolic BP is 100/40mmHg. There is a Question 27 mass at the introitus consistent with uterine Correct inversion. She has venous access with answer: J adequate fluid resuscitation

Question 28

A 23 year old primigravida is admitted for induction of labour at 39 weeks gestation because of symphysis pubis diastasis. Three hours after vaginal prostaglandin

Your answer: A Correct answer: H

administration, she complains of painful contractions every minute. CTG shows contractions 8:10 lasting ~30s. The fetal heart rate is 140bpm with good variability and accelerations.

| EXPLANATION | Options for Questions 29-30

A Intravenous labetalol C

Intravenous magnesium sulphate

E

Measure FBC and clotting profile

B Immediate delivery by caesarean section D

Measure serum aspartate transaminase immediately

F Blood transfusion

G Immediate induction of labour

H Provide a fluid challenge with colloids

I

Antihypertensive treatment

J

Measure 24h urine protein K excretion

L

Administer iv phenytoin Arrange in-utero transfer to tertiary centre

Instructions:For each patient described below choose the singlemost appropriate management option from the list. Each option may be used once, more than once, or not at all. A 30 year old primigravida attends as an emergency at 32 weeks gestation with a twin pregnancy and complains of 12h of severe vomiting and upper abdominal pain. Her BP is Your answer: A 130/80mmHg with a trace of proteinuria. A growth scan one week earlier had shown Question 29 normal fetal growth. Investigations show a Correct serum aspartate transaminase concentration of answer: B 600iu/l, alkaline phosphatase = 1550iu/l, serum uric acid = 0.57mmol/l and serum creatinine = 130micromol/l. FBC and coagulation profile are normal. A 32 year old primigravida with a previously normal pregnancy is seen in the antenatal clinic Your answer: A at 39 weeks gestation with a BP of Question 30 160/95mmHg and 3+ proteinuria. The uterus is Correct appropriate for gestation age with a cephalic answer: G presentation.

| EXPLANATION | Options for Questions 31-32 A Routine recall in 3 years

B Routine recall in 5 years

C Hysteroscopy + D&C

D Laser vapourisation

E Refer for colposcopy

F

G Radical trachylectomy

H Cryotherapy

I

J

No further cervical smears required

Cold knife cone biopsy Repeat cervical smear in 6 months

Instructions:For each scenario described below, choose the singlemost appropriate management from the above list of options. Each option may be

used once, more than once, or not at all. A 40 year old nulliparous woman had a cervical Your answer: A smear showing severe dyskaryosis. Question 31 Colposcopy with diathermy loop excision has Correct been performed and the histology shows CIN III answer: J which has been completely excised A 53 year old asymptomatic post menopausal woman has a cervical smear showing no dyskaryotic cells but no evidence of

Your answer: A

Question 32 transformation zone sampling. The cervix was well visualised and normal and she has previously had regular smears which have all been negative.

Correct answer: B

| EXPLANATION | Options for Questions 33-34

A Take pill as soon as possible C

B

Take pill as soon as possible + barrier contraception for 7 days

Take pill as soon as possible and Administer emergency contraceptive D omit pill-free week pill

E Insert copper IUCD

F Await results of pregnancy test

Replace detached patch with a G new patch

H

Apply new patch and use barrier I contraception for 7 days

Apply new patch but keep the same J change date

K Apply new patch and delay change day by 24h

L

Replace with a new patch and use barrier contraception for 7 days

Apply new patch and delay change day by 36h

Instructions:For each of the case histories described below, choose the single most appropriate advice from the above list. Each option may be used once, more than once, or not at all. A 30 year old nulliparous woman is using the combined contraceptive patch. She placed her Question 33 second patch at 10pm before going to bed and having sexual intercourse and realised 36h later that the patch had detached

A 35 year old woman is using the contraceptive Question 34 patch and telephones for advice because she has missed her patch change date by 24h

Your answer: A Correct answer: I

Your answer: A Correct answer: J

| EXPLANATION | Options for Questions 35-36 A Bilateral oophrectomy

B CA-125 assays every 6-12 months

C TAH + BSO

D Cyst aspiration for cytology

E Refer to cancer centre

F Ovarian cystectomy

G Reassure

H Measure serum CA-125

I

J

Refer to palliative care team

Paracentesis for cytology

K Chemotherapy

L

Radiotherapy

Instructions:For each scenario described below, choose the singlemost appropriate management from the above list of options. Each option may be used once, more than once, or not at all. A 56 year old post-menopausal woman with Your answer: A diverticular disease has an ultrasound scan because of a suspected diverticular abscess. Question 35 There is no evidence of pelvic abscess but the Correct scan shows a 4cm septate left ovarian cyst with answer: H no free fluid. A 30 year old nulliparous woman complains of Your answer: A a 6 months history of deep dyspareunia that Question 36 has not responded to medical treatment. Pelvic Correct ultrasound scan shows a 6cm complex right answer: F ovarian mass consistent with a dermoid cyst.

| EXPLANATION | Options for Questions 37-38

A Atrophic vulvovaginitis C

Benign mucous membrane pemphigoid

B

Human papilloma virus infection

D Candida infection

E Lichen sclerosus

F Contact dermatitis

G Lichen simplex et chronicus

H Herpes simplex infection

I

J

Vulval intraepithelial neoplasia

Vulvodynia

Instructions:For each clinical scenario below choose the singlemost likely diagnosis from the list above. Each diagnosis may be used once, more than once, or not at all. A 23-year-old woman presents with a two-year history of vulval, perineal and perianal irritation. Your answer: A The vulva is red, excoriated and there areas of Question 37 white, thickened skin. Application of 3% Acetic Correct acid shows areas of mosaic and coarse answer: I punctuation. A 78-year-old woman presents with vulval irritation and soreness. On examination the vulva is red in colour, slightly oedematous and Question 38 there are small, red papules scattered randomly beyond the perimeter of the vulva. She also complains of soreness and irritation under the breasts.

Your answer: A Correct answer: D

| EXPLANATION | Options for Questions 39-40 A Failure to relieve symptoms

B Damage to bowel

C Removal of ovary

D Irregular bleeding for 3-4 months

E Failure to identify disease

F Failure to visualise uterine cavity

G Shoulder-tip pain

H Haemorrhage requiring return to theatre

I

J

Laparotomy

Failure rate 1 in 200

Instructions:For each of the case histories described below, choose the single most relevant complication that you must discuss with the patient when taking consent prior to surgery from the above list of options. Each option may be used once, more than once, or not at all. An 18 year old woman is known to have an 8cm simple ovarian cyst and admitted as an Question 39 emergency with torsion. She is due to undergo laparoscopic ovarian cystectomy. She is otherwise healthy with a BMI of 27.

Your answer: A Correct answer: C

A 56 year old woman had successful transYour answer: A cervical resection of the endometrium eight years earlier for menorrhagia and now presents Question 40 with post-menopausal bleeding. She is Correct otherwise healthy and has been listed for answer: F hysteroscopy D&C

Home | Signup | Subscribe| Contact us | Medical forums | Privacy | Legal Notices | Copyright © Busy SpR Busy SpR Ltd. is a Registered Company of the United Kingdom. Company Registration Number:4407908 Developed by Last Digital

Options for Questions 1-2 A Congenital adrenal hyperplasia

B Maternal androgen therapy

C Down’s syndrome

D Turner’s syndrome

E Maternal diabetes mellitus

F Maternal lithium therapy

G Maternal warfarin therapy

H Maternal heparin therapy

I

Group B streptococcus infection

K Maternal smoking

J L

Maternal anti-epileptic drug therapy Maternal cocaine abuse

Instructions:For each of the case histories described below, choose the singlemost likely cause of neonatal collapse from the above list of options. Each option may be used once, more than once, or not at all.

Question 1

A 6 hour old neonate with ambiguous genitalia

Your answer: A Correct answer: A

Welcome SEMSEM Your last visit was on Wednesday Apr 05th, 2006 You have been a member for 1 days.

Account Manager

A 3 hour old neonate weighing 4900g at Question 2 birth

Your answer: A

• • •

Correct answer: E

• • •

| EXPLANATION |

Edit my details MCQ test scores Mock MCQ scores EMQ test scores Mock EMQ scores Logout

BUSY SpR Options for Questions 3-4

Home

Congenital adrenal A hyperplasia

B Maternal androgen therapy

C Down’s syndrome

D Turner’s syndrome

E Maternal diabetes mellitus

F Maternal lithium therapy

Lost password?

G Maternal warfarin therapy

H Maternal heparin therapy

Medical forums

I

Group B streptococcus infection

J

K Maternal smoking

L

About us Signup/Join

Maternal anti-epileptic drug therapy Maternal cocaine abuse

Contributors Bibliography Contact us

Instructions:For each of the case histories described below, choose the singlemost likely cause of neonatal collapse from the above list of options. Each option may be used once, more than once, or not at all.

A 6 hour old neonate delivered at 35 Question 3 weeks gestation following prolonged preterm rupture of the membranes

A 6 hour old neonate delivered by rotational forceps delivery for prolonged second stage of labour. Initial Question 4 examination showed epicanthic folds with palpebral fissures and a loud up-slanting systolic murmur

Recommend us Privacy

Your answer: A Correct answer: I

Your answer: A Correct answer: C

Home | Signup | Subscribe| Contact us | Med Options for Questions 1-2

A Take pill as soon as possible B

Take pill as soon as possible + barrier contraception for 7 days

C Take pill as soon as possible D Administer emergency and omit pill-free week contraceptive pill E Insert copper IUCD F Await results of pregnancy test G Replace detached patch with a new patch Apply new patch and use I barrier contraception for 7 days K

Apply new patch and delay change day by 24h

Replace with a new patch and H use barrier contraception for 7 days J

Apply new patch but keep the same change date

L

Apply new patch and delay change day by 36h

Instructions:For each of the case histories described below, choose the singlemost appropriate advice from the above list. Each option may be used once, more than once, or not at all. Question 1

A 30 year old nulliparous woman is using Your answer: the combined contraceptive patch. She A

Welcome SEMSEM Your last visit was on Wednesday Apr 05th, 2006 You have been a member for 1 days.

placed her second patch at 10pm before going to bed and having sexual intercourse and realised 36h later that the patch had detached

Account Manager Correct answer: I

• • •

A 35 year old woman is using the contraceptive patch and telephones for Question 2 advice because she has missed her patch change date by 24h

Your answer: A Correct answer: J

• •

BUSY SpR Home

| EXPLANATION |

About us

Options for Questions 3-4

A Commence iv oxytocin



Edit my details MCQ test scores Mock MCQ scores EMQ test scores Mock EMQ scores Logout

Signup/Join

B

Deliver by caesarean section

Lost password? Medical forums

Evacuation of retained products of Artificial rupture of C D conception membranes

Contributors

E Bimanual compression of the uterus

F Venous access and resuscitation

Bibliography

H Intra-muscular analgesia

Recommend us

Manual replacement of J the uterus

Privacy

G I

Examination under spinal anaesthesia Intravenous ritodrine

K Sub-cutaneous terbutalline

L

Administer uterotonic agent

Instructions:For each of the case histories described below, choose the singlemost appropriate management from the above list. Each option may be used once, more than once, or not at all. A 34 year old woman is bleeding heavily during the third stage of labour. Her pulse Your answer: A is 60bpm and systolic BP is Question 3 100/40mmHg. There is a mass at the introitus consistent with uterine inversion. Correct She has venous access with adequate answer: J fluid resuscitation A 23 year old primigravida is admitted for induction of labour at 39 weeks gestation Your answer: because of symphysis pubis diastasis. A Three hours after vaginal prostaglandin Question 4 administration, she complains of painful contractions 8:10 everylasting minute. CTGThe shows contractions ~30s. fetal heart rate is 140bpm with good variability and accelerations.

Correct answer: H

| EXPLANATION | Options for Questions 5-6

A

No additional intervention at this stage

C Increase dose of oxytocin

B

Deliver by caesarean section

D

Artificial rupture of membranes

Contact us

E Decrease dose of oxytocin

F Fetal blood sampling

Controlled artificial rupture of G membranes in theatre

H Intra-muscular analgesia

I

Continue oxytocin at current dose

K Sub-cutaneous terbutalline

J

Inhaled salbutamol

L

Administer maternal facial oxygen

Instructions:For each of the case histories described below, choose the singlemost appropriate management from the above list. Each option may be used once, more than once, or not at all. A 24 year old woman with 2 previous vaginal deliveries presents in spontaneous labour at 40 weeks Your answer: gestation. She progresses to 9cm A dilatation and there is thick old meconium Question 5 stained liquor. The CTG shows a baseline fetal heart rate of 135bpm with Correct variability of ~10bpm, with occasional answer: A accelarations and no decelerations. There are 3-4 uterine contractions every 10 minutes. A 34 year old primigravida presents in spontaneous labour at 39 weeks gestation. The cervix is 4cm dilated with thin meconium stained liquor. The CTG Question 6 over 25 minutes shows a base-line fetal heart rate of 135bpm with variability of 34bpm, no accelerations and variable decelerations. There are 3-4 uterine contractions every 10 minutes.

Your answer: A Correct answer: F

| EXPLANATION | Options for Questions 7-8 A No additional intervention at this stage

B Deliver by caesarean section

C Evacuation of retained products of conception

D Artificial rupture of membranes

E Oxytocin induction of labour

F

G Controlled artificial rupture of membranes in theatre

H Intra-muscular analgesia

I

Prostaglandin induction of labour

K Sub-cutaneous terbutalline

Venous access and resuscitation

J the Manual replacement of uterus L

Delay induction of labour

Instructions:For each of the case histories described below, choose the singlemost appropriate management from the above list. Each option may be used once, more than once, or not at all. A 34 year old nulliparous woman presents in spontaneous labour at 37 weeks gestation with the cervix 3cm Question 7 dilated. Four hours later, the cervix is 8cm dilated and a frank breech presentation is diagnosed.

Your answer: A Correct answer: B

A 24 year old primigravida has been admitted for induction at 38 weeks gestation with a dichorionic twin Your answer: pregnancy because of maternal A discomfort. The pregnancy has otherwise Question 8 been uncomplicated and twin 1 is Correct cephalic. The cervix is 2cm long, answer: I posterior, soft and the os is closed. The presenting part is 3cm above the spines. CTG is reactive.

| EXPLANATION | Options for Questions 9-10 A No additional intervention at this stage C

Evacuation of retained products of conception

E Commence iv oxytocin regimen G I

Controlled artificial rupture of membranes in theatre Prostaglandin induction of labour

K Sub-cutaneous terbutalline

B Deliver by caesarean section D

Artificial rupture of membranes

F

Venous access and resuscitation

H Intra-muscular analgesia J L

Manual replacement of the uterus Delay induction of labour

Instructions:For each of the case histories described below, choose the singlemost appropriate management from the above list. Each option may be used once, more than once, or not at all. A 25 year old primigravida presents for induction of labour at 42 weeks gestation. Artificial rupture of the membranes (ARM) Your answer: is performed at 09:00 at 1cm dilatation A and oxytocin commenced at 10:00. An Question 9 epidural is subsequently inserted and Correct oxytocin is increased to a maximum of answer: B 32mu/min according to the unit protocol. 12h after ARM, the cervix is 1cm dilated and partially effaced. A 34 year old woman with four previous vaginal deliveries has been admitted for induction of labour at 40 weeks gestation Your answer: because she has a history of precipitate A labours with her last baby being delivered Question 10 in the car. The fetal head is five fifths Correct palpable and the cervix is 2cm long, soft answer: E and posterior with a multiparous os. The presenting part is not reached on vaginal examination.

| EXPLANATION | Options for Questions 11-12 A Continue iv oxytocin

B Deliver by caesarean

section C

Evacuation of retained products of conception

D

No additional intervention at this stage

E

Bimanual compression of the uterus

F

Venous access and resuscitation

G

Examination under spinal anaesthesia

H Lovset’s manoeuvre

I

J

Intravenous ritodrine

K Sub-cutaneous terbutalline

L

Manual replacement of the uterus Administer uterotonic agent

Instructions:For each of the case histories described below, choose the singlemost appropriate initial management from the above list. Each option may be used once, more than once, or not at all. A 16 year old primigravida attends for induction of labour following intra-uterine fetal death at 36 weeks gestation. Maternal BP is 120/80 with no proteinuria Your answer: A and all blood tests are normal. The fetus Question 11is in a breech presentation. Prostaglandin is administered and oxytocin commenced Correct at 12:00 according to the unit protocol. At answer: A 00:00, the cervix remains 1cm long and the os is closed. There are 3 uterine contractions every 10 minutes. A 34 year old woman is bleeding heavily collapses during the third stage of Question 12and labour. Her pulse is 40bpm and systolic BP is 70mmHg. There is a mass at the introitus consistent with uterine inversion

Your answer: A Correct answer: F

| EXPLANATION | Options for Questions 13-14

A Take pill as soon as possible B

Take pill as soon as possible + barrier contraception for 7 days

C Take pill as soon as possible D Administer emergency and omit pill-free week contraceptive pill E Insert copper IUCD F Await results of pregnancy test G

Replace detached patch with a new patch

Replace with a new patch and H use barrier contraception for 7 days

Apply new patch and use I barrier contraception for 7 days

J

Apply new patch but keep the same change date

K Apply new patch and delay change day by 24h

L

Apply new patch and delay change day by 36h

Instructions:For each of the case histories described below, choose the singlemost appropriate advice from the above list. Each option may be used once, more than once, or not at all. A 30 year old nulliparous woman is using Your answer: A Question 13the combined contraceptive patch. She placed her second patch at 10pm before

going to bed and having sexual intercourse and realised at 10am that the patch had detached.

Correct answer: J

A 29 year old mother of 3 children is Your answer: using the combined contraceptive patch. A She placed her second patch at 10pm on Question 14 a Wednesday and realised 3 days later Correct that the patch had detached. She had answer: I sexual intercourse on several occasions.

| EXPLANATION | Options for Questions 15-16 A Commence iv oxytocin

B Deliver by caesarean section

C Fetal blood sampling

D

No additional intervention at this stage

E Umbilical artery Doppler

F Reduce oxytocin dose

G Uterine artery Doppler

H Forceps delivery

I

J

Administer maternal facial oxygen

L

Administer uterotonic agent

Intravenous ritodrine

K Sub-cutaneous terbutalline

Instructions:For each of the case histories described below, choose the singlemost appropriate initial management from the above list. Each option may be used once, more than once, or not at all. You have been asked to review a 35 year old woman half an hour after Your answer: spontaneous vaginal delivery of her first A child because the membranes are ragged Question 15 although the placenta appears complete. Correct The uterus is well contracted, the lochia answer: D are normal and all maternal observations are satisfactory. A 35 year old mother of two children presents in spontaneous labour at 39 weeks gestation. She has one previous Your answer: caesarean section for breech A presentation followed by a spontaneous Question 16vaginal delivery. At 12:00, the cervix is 7cm dilated with a direct occipito-anterior Correct position with 3:10 strong contractions. At answer: B 16:00, the cervix is 7cm dilated and the contractions are inco-ordinate occurring 2-3:10. The CTG is normal

| EXPLANATION | Options for Questions 17-18 A Increase oxytocin dose

B Deliver by caesarean section

C Fetal blood sampling

D Continuous CTG monitoring

E Umbilical artery Doppler

F

G Uterine artery Doppler

H Inhaled salbutamol

Reduce oxytocin dose

I

Intravenous ritodrine

J

K Sub-cutaneous terbutalline L

Administer maternal facial oxygen Administer uterotonic agent

Instructions:For each of the case histories described below, choose the singlemost appropriate initial management from the above list. Each option may be used once, more than once, or not at all. A 34 year old primigravida attends for induction of labour at 42 weeks gestation. Your answer: She progresses normally to 8cm A dilatation at which point the dose of Question 17oxytocin is 16mu/min. The CTG shows 7 uterine contractions every 10 minutes Correct lasting 30-40s with a base-line fetal heart answer: F rate of 160bpm with good variability and variable decelerations. A 23 year old primigravida is admitted for induction of labour at 42 weeks gestation. Your answer: Three hours after vaginal prostaglandin A administration, she complains of painful Question 18 contractions every minute. CTG shows Correct contractions 8:10 lasting ~30s. The fetal answer: K heart rate is 140bpm with good variability and deep variable decelerations.

| EXPLANATION |

Options for Questions 19-20 A No additional intervention at this B Deliver by caesarean stage section C Increase dose of oxytocin D Lovset’s manoeuvre E Decrease dose of oxytocin

F Fetal blood sampling

G Forceps delivery

H Delivery of the posterior arm

I

Episiotomy

K McRobert’s manoeuvre

J L

Inhaled salbutamol Administer maternal facial oxygen

Instructions:For each of the case histories described below, choose the singlemost appropriate management from the above list. Each option may be used once, more than once, or not at all. A 34 year old woman with 3 previous vaginal deliveries presents in spontaneous labour at 39 weeks gestation. The cervix is fully dilated with thick meconium stained liquor. The CTG Question 19shows a base-line fetal heart rate of 135bpm with variability of 3-4bpm with variable decelerations. There are 3-4 uterine contractions every 10 minutes. The fetus is in a direct occipito-anterior position 1cm below the ischial spines.

Your answer: A Correct answer: G

Your answer: You have been called into a delivery A room by the senior midwife because of Question 20 difficulties delivering the fetal shoulders. On your arrival, the woman is in lithotomy Correct

and the head had been delivered 2 minutes earlier but moderate traction has thus far failed to deliver the shoulders.

answer: K

| EXPLANATION | Options for Questions 21-22

A

No additional intervention at this stage

B

Deliver by caesarean section

C Evacuation of retained products of conception E Commence iv oxytocin regimen

D Artificial rupture of membranes F Fetal blood sampling

G Controlled artificial rupture of membranes in theatre

H Intra-muscular analgesia

I

J

Inhaled salbutamol

L

Administer maternal facial oxygen

Prostaglandin induction of labour

K Sub-cutaneous terbutalline

Instructions:For each of the case histories described below, choose the singlemost appropriate management from the above list. Each option may be used once, more than once, or not at all. A 34 year old primigravida presents in spontaneous labour at 39 weeks gestation. The cervix is 4cm dilated with thin meconium stained liquor. The CTG Question 21over 25 minutes shows a base-line fetal heart rate of 135bpm with variability of 34bpm, no accelerations and no decelerations. There are 3-4 uterine contractions every 10 minutes.

Your answer: A Correct answer: A

A 34 year old primigravida presents in spontaneous labour at 39 weeks gestation. The cervix is 4cm dilated with Your answer: A thin meconium stained liquor. The CTG Question 22over shows a base-line fetal heart rate of 135bpm with variability of 3-4bpm, no Correct accelerations and no decelerations over a answer: F period of 120minutes. There are 3-4 uterine contractions every 10 minutes.

| EXPLANATION | Options for Questions 23-24 A

No additional intervention at this stage

C Increase dose of oxytocin E Decrease dose of oxytocin

B

Deliver by caesarean section

D Artificial rupture of membranes F Fetal blood sampling

G Controlled artificial rupture of membranes in theatre

H Intra-muscular analgesia

I

J

Inhaled salbutamol

L

Administer maternal facial oxygen

Continue oxytocin at current dose

K Sub-cutaneous terbutalline

Instructions:For each of the case histories described below, choose the singlemost appropriate management from the above list. Each option may be used once, more than once, or not at all. A 34 year old woman attends for induction of labour at 42 weeks gestation. 4hours after prostaglandin administration, Your answer: she complains of painful contractions. A The CTG shows contractions 6:10 lasting Question 23 30-45s with a baseline fetal heart rate of Correct 170bpm and late decelerations. answer: B cervix is 2cm dilated and there isThe thick fresh meconium on artificial rupture of membranes. A 40 year woman with three previous vaginal deliveries attends for induction of labour at 42 weeks. She has artificial rupture of membranes at 09:00 (multiparous os) and oxytocin commenced at 11:00. Epidural analgesia Your answer: A is inserted at 13:00 but is ineffective. As a Question 24consequence, the maximum dose of oxytocin administered is 4mu/min. Correct Epidural is re-sited at 19:00. At 23:00, answer: I she has adequate analgesia, the oxytocin dose is 16mu/min and she has 3-4 contractions every 10 minutes. The cervix is fully effaced, 2cm dilated and the CTG is reactive.

| EXPLANATION | Options for Questions 25-26 No additional intervention at this Deliver by caesarean B stage section C Increase dose of oxytocin D Lovset’s manoeuvre A

E Decrease dose of oxytocin

F Fetal blood sampling

G Forceps delivery

H Delivery of the posterior arm

I

J

Episiotomy

K McRobert’s manoeuvre

L

Inhaled salbutamol Administer maternal facial oxygen

Instructions:For each of the case histories described below, choose the singlemost appropriate management from the above list. Each option may be used once, more than once, or not at all. A 40 year woman with three previous vaginal deliveries attends for induction of labour at 42 weeks. She has artificial Your answer: rupture of membranes at 09:00 A (multiparous os) and oxytocin Question 25commenced at 11:00. Epidural analgesia is inserted at 13:00 but is ineffective. As a Correct consequence, the maximum dose of answer: I oxytocin administered is 4mu/min. Epidural is re-sited at 19:00. At 23:00, she has adequate analgesia, the oxytocin

dose is 16mu/min and she has 3-4 contractions every 10 minutes. The cervix is fully effaced, 2cm dilated and the CTG is reactive. A 40 year woman with three previous vaginal deliveries attends for induction of labour at 42 weeks. She has artificial rupture of membranes at 09:00 (multiparous os) and oxytocin commenced at 11:00. Epidural analgesia Your answer: A inserted at 13:00 but is ineffective. Question 26is consequence, the maximum dose of As a oxytocin administered is 4mu/min. Correct Epidural is re-sited at 19:00. At 23:00, answer: I she has adequate analgesia, the oxytocin dose is 16mu/min and she has 3-4 contractions every 10 minutes. The cervix is fully effaced, 2cm dilated and the CTG is reactive.

| EXPLANATION | Options for Questions 27-28 A No additional intervention at this stage C

Evacuation of retained products of conception

B Deliver by caesarean section D

E Bimanual compression of the uterus F

Artificial rupture of membranes Venous access and resuscitation

G Controlled artificial rupture of membranes in theatre

H Intra-muscular analgesia

I

J

Manual replacement of the uterus

L

Commence iv oxytocin

Prostaglandin induction of labour

K Sub-cutaneous terbutalline

Instructions:For each of the case histories described below, choose the singlemost appropriate management from the above list. Each option may be used once, more than once, or not at all. You have been asked to review a 25 year Your answer: old woman half an hour after A spontaneous vaginal delivery of her first Question 27child because a placental cotyledon is retained. The uterus is well contracted, Correct the lochia areare normal and all maternal observations satisfactory.

answer: C

A 23 year old woman with two previous vaginal deliveries is admitted for induction Your answer: of labour at 42 weeks gestation. The A pregnancy has been uncomplicated and Question 28 the fetal head is three fifths palpable. The Correct cervix is 1cm long, central, 2cm dilated answer: D with bulging membranes and the fetal head is 2cm above the ischial spines.

Home | Signup | Subscribe| Contact us | Medical forums | Privacy | Legal Notices | Copyright © Busy SpR Busy SpR Ltd. is a Registered Company of the United Kingdom. Company Registration Number:4407908 Developed by Last Digital

Options for Questions 1-2 A Urinary retention

B Damage to bowel

C Removal of ovaries

D Failure to gain entry into abdominal cavity

E Failure to identify disease

F

G Haemorrhage requiring blood transfusion

H Haemorrhage requiring return to theatre

I

J

Laparotomy

Failure to visualise uterine cavity Failure rate 1 in 200

Instructions:For each of the case histories described below, choose the singlemost relevant complication that you must discuss with the patient when taking consent prior to surgery from the above list of options. Each option may be used once, more than once, or not at all.

Question 1

Question 2

A 30 year old mother of 5 children who is scheduled to undergo laparoscopic sterilisation and has expressed a personal objection to blood transfusion.

A 35 year old mother of three children is using the combined oral contraceptive pill and is scheduled to undergo laparoscopic sterilisation

Your answer: A Correct answer: G Your answer: A Correct answer: J

| EXPLANATION | Options for Questions 3-3 A Closure of visceral peritoneum

B Closure of parietal peritoneum

C Closure of Scarpa’s fascia

D Non-closure of the rectus sheath

E Instillation of local anaesthetic into pelvis

F Supra-pubic catheterisation

G Mass closure of abdominal incision

H Prophylactic heparin 2h pre-op

I

Application of methylene blue to vagina

K Instillation of local anaesthetic into pelvis

J

Closure of vaginal vault

L

Use of prophylactic antibiotics after delivery

Instructions:For each of the case histories described below, choose the singlemost appropriate intervention that would reduce peri-operative morbidity and mortality from the above list. Each option may be used once, more than once, or not at all. Your answer: A Question 3

A 35 year old mother of 4 children is to undergo laparoscopic sterilisation. She is otherwise healthy with a BMI of 27.

Correct answer: K

| EXPLANATION | Options for Questions 4-5 A Urinary retention

B Damage to bowel

C Removal of ovaries

D Failure to gain entry into abdominal cavity

E Failure to identify disease

F

G Haemorrhage requiring blood transfusion

H Haemorrhage requiring return to theatre

Failure to visualise uterine cavity

I

J

Laparotomy

Uterine perforation

Instructions:For each of the case histories described below, choose the singlemost relevant complication that you must discuss with the patient when taking consent prior to surgery from the above list of options. Each option may be used once, more than once, or not at all.

Question 4

Question 5

A 30 year old woman with genuine stress incontinence who is due to have a tension-free vaginal tape

A 30 year old woman with primary infertility who is due to undergo IVF treatment and is scheduled to have laparoscopy and ovarian cystectomies for severe endometriosis with bilateral ovarian endometriomas.

Your answer: A Correct answer: A

Your answer: A Correct answer: C

| EXPLANATION | Options for Questions 6-7 A Damage to bladder / ureter

B Damage to bowel

C Failure rate 1 in 200

D Failure to gain entry into abdominal cavity

E Failure to identify disease

F

G Haemorrhage requiring blood transfusion

H Haemorrhage requiring return to theatre

I

J

Laparotomy

Failure to visualise uterine cavity Uterine perforation

Instructions:For each of the case histories described below, choose the singlemost relevant complication that you must discuss with the patient when taking consent prior to surgery from the above list of options. Each option may be used once, more than once, or not at all.

Question 6

Question 7

A 52-year-old woman with frequent heavy periods is listed for diagnostic hysteroscopy. She has had two children both delivered by caesarean section. She is hypertensive and her BMI is 26. A 56-year-old woman is scheduled for laparotomy and possible bilateral salpingo-oophorectomy for an ovarian mass. She had a total abdominal hysterectomy at the age of forty for fibroids and is in discomfort with an ovarian mass which measures 15cm in diameter on ultrasound examination.

Your answer: A Correct answer: J

Your answer: A Correct answer: B

| EXPLANATION | Options for Questions 8-9 A Failure to relieve symptoms

B Damage to bowel

C Removal of ovary

D Irregular bleeding for 3-4 months

E Failure to identify disease

F

G Shoulder-tip pain

H Haemorrhage requiring return to theatre

I

J

Laparotomy

Failure to visualise uterine cavity Failure rate 1 in 200

Instructions:For each of the case histories described below, choose the singlemost relevant complication that you must discuss with the patient when taking consent prior to surgery from the above list of options. Each option may be used once, more than once, or not at all.

Question 8

An 18 year old woman is known to have an 8cm simple ovarian cyst and admitted as an emergency with torsion. She is due to

Your answer: A

undergo laparoscopic ovarian cystectomy. She is otherwise healthy with a BMI of 27.

Question 9

A 56 year old woman had successful trans-cervical resection of the endometrium eight years earlier for menorrhagia and now presents with post-menopausal bleeding. She is otherwise healthy and has been listed for hysteroscopy D&C

Correct answer: C

Your answer: A Correct answer: F

| EXPLANATION | Options for Questions 10-11 A Failure to relieve symptoms

B Damage to bowel

C Removal of ovaries

D Irregular bleeding for 3-4 months

E Failure to identify disease

F

G Shoulder-tip pain

H Haemorrhage requiring return to theatre

I

J

Laparotomy

Failure to visualise uterine cavity Failure rate 1 in 200

Instructions:For each of the case histories described below, choose the singlemost relevant complication that you must discuss with the patient when taking consent prior to surgery from the above list of options. Each option may be used once, more than once, or not at all.

Question 10

Question 11

A 35 year old nulliparous woman with menorrhagia is having the levonorgestrel releasing intra-uterine system inserted under general anaesthesia

A 40 year old woman with a previous failed TVT for genuine stress incontinence is having a Burch colposuspension

Your answer: A Correct answer: D

Your answer: A Correct answer: A

| EXPLANATION | Options for Questions 12-13 A Closure of visceral peritoneum

B Closure of parietal peritoneum

C Closure of Scarpa’s fascia

D Non-closure of the rectus sheath

E Instillation of local anaesthetic into pelvis

F

G Mass closure of abdominal incision

H Use of absorbable suture material

I

Application of methylene blue to vagina

K Excision of vaginal cuff

Supra-pubic catheterisation

J

Closure of vaginal vault

L

Non-closure of the peritoneum

Instructions:For each of the case histories described below, choose the singlemost appropriate intervention that would reduce peri-operative morbidity and mortality from the above list. Each option may be used once, more than once, or not at all.

Question 12

Question 13

A 70 year old woman with stage Ia endometrial cancer on MRI scan is to undergo TAH +BSO. She has no significant medical or surgical history and her BMI is 27

A 45 year old woman is to undergo laparotomy for a 15cm complex ovarian mass. She is otherwise healthy and has a BMI of 40

Your answer: A Correct answer: L Your answer: A Correct answer: G

| EXPLANATION | Options for Questions 14-15 A Closure of visceral peritoneum

B Closure of parietal peritoneum

C Closure of Scarpa’s fascia

D Non-closure of the rectus sheath

E Instillation of local anaesthetic into pelvis

F Supra-pubic catheterisation

G Mass closure of abdominal incision

H Prophylactic heparin 2h pre-op

I

J

Closure of vaginal vault

L

Use of prophylactic antibiotics after delivery

Application of methylene blue to vagina

K Excision of vaginal cuff

Instructions:For each of the case histories described below, choose the singlemost appropriate intervention that would reduce peri-operative morbidity and mortality from the above list. Each option may be used once, more than once, or not at all.

Question 14

Question 15

A 25 year old woman is undergoing an emergency caesarean section at 9cm dilatation for failure to progress

A 42 year old woman is to undergo TAH for dysfunctional uterine bleeding. She is otherwise fit and well with a BMI of 35.

Your answer: A Correct answer: L Your answer: A Correct answer: H

| EXPLANATION |

Options for Questions 16-17 A Damage to bladder / ureter

B Damage to bowel

C Failure rate 1 in 200

D Failure to gain entry into abdominal cavity

E Failure to identify disease

F

G Haemorrhage requiring blood transfusion

H Haemorrhage requiring return to theatre

I

J

Laparotomy

Failure to visualise uterine cavity Uterine perforation

Instructions:For each of the case histories described below, choose the singlemost relevant complication that you must discuss with the patient when taking consent prior to surgery from the above list of options. Each option may be used once, more than once, or not at all.

Question 16

Question 17

A 48-year-old nulliparous woman is scheduled for vaginal hysterectomy because of menorrhagia. Her uterus is enlarged equivalent to 14 weeks' gestation.

A 30 that yearhas old failed woman a 3 year historytreatment. of abdominal pain to with respond to medical Her and painpelvic is unrelated to menses and she has no bowel symptoms. She is otherwise fit and well with a BMI of 25 and is listed for diagnostic laparoscopy.

Your answer: A Correct answer: I

Your answer: A Correct answer: E

Options for Questions 1-2 A Raloxifene

B Oral sequential oestrogen + progestogen

C Oral continuous oestrogen + progestogen

D Oestrogen patches

E Alteration of therapy not necessary

F Levonorgestrel IUS + oral oestrogens

G Levonorgestrel IUS

H Refer to special oncology clinic

I

J

Counsel and recommend discontinuation of HRT

HRT not recommended

Instructions: For each scenario described below, choose the single most appropriate management from the above list of options. Each option may be used once, more than once, or not at all. Question 1

Question 2

A 50 year old woman has been referred to the gynaecology clinic because her 60 year old sister has developed an osteoporotic fracture. She is healthy with mild menopausal symptoms but is concerned about the risk of osteoporosis A 55 year old woman attends the gynaecology clinic for oestrogen implant. She had TAH + BSO at the age of 45 years for a torted benign ovarian cyst and has yearly oestrogen implants. She currently has minimal menopausal symptoms

Your answer: A Correct answer: J

Your answer: A Correct answer: I

| EXPLANATION | Options for Questions 3-4 A Raloxifene

B Oral sequential oestrogen + progestogen

C Oral continuous oestrogen + progestogen

D Oestrogen patches

E Alteration of therapy not necessary

F Levonorgestrel IUS + oral oestrogens

G Levonorgestrel IUS

H Refer to special oncology clinic

I

J

Counsel and recommend discontinuation of HRT

HRT not recommended

Instructions: For each scenario described below, choose the single most appropriate management from the above list of options. Each option may be used once, more than once, or not at all. Question 3

Question 4

A healthy 52 year old woman is due to be admitted for pelvic floor repair. She is taking continuous combined HRT and her BMI is 28.

A healthy 48 year old woman complains of infrequent periods and severe hot flushes. She experienced mood swings, depression and fluid retention when she took the COCP in her thirties and had to discontinue treatment. She uses the sheath for contraception.

Your answer: A Correct answer: E

Your answer: A Correct answer: F

| EXPLANATION | Options for Questions 5-6 A Commence iv erythromycin C Commence oral erythromycin

B Commence oral co-amoxiclav D Corticosteroids + oral erythromycin

E Administer corticosteroids for fetal lung maturity

F Administer corticosteroids + tocolytic

G Arrange in-utero transfer to tertiary centre

H Arrange ex-utero transfer to tertiary centre

I

J

Induction of labour + iv benzyl penicillin

Counsel and recommend termination of pregnancy

Instructions: For each scenario described below, choose the single most appropriate management from the above list of options. Each option may be used once, more than once, or not at all. Question 5

A 32 year old primigravida was found to have group B streptococcal bacteriuria at 16 weeks gestation which was successfully treated. She presents at 36 weeks gestation with

Your answer: A Correct answer: I

spontaneous rupture of the membranes. All maternal and fetal observations are within normal limits, there are no contractions and the cervix is not dilated

Question 6

An 18 year old primigravida presents with spontaneous rupture of the membranes at 20 weeks gestation and is managed expectantly. Seven days later, she complains of feeling generally unwell and is noted to have a temperature of 38C and pulse = 115bpm. The uterus is tender with mild contractions every 10 minutes and the liquor is offensive. The fetal heart is 120bpm

Your answer: A Correct answer: J

| EXPLANATION | Options for Questions 7-8 A Raloxifene

B Oral sequential oestrogen + progestogen

C Oral continuous oestrogen + progestogen

D

Check thrombophilia screen before commencing HRT

E Vaginal oestrogen tablets

F

G Levonorgestrel IUS

H Refer to special oncology clinic

I

Counsel and recommend discontinuation of HRT

J

Levonorgestrel IUS + oral oestrogens

HRT not recommended

Instructions: For each scenario described below, choose the single most appropriate management from the above list of options. Each option may be used once, more than once, or not at all. Question 7

Question 8

A 52 year old woman on continuous combined HRT complains of persistent vaginal bleeding but no other symptoms. Trans-vaginal ultrasound scan shows an endometrial thickness of 18mm with no adnexal masses A 52 year old woman is wheel-chair bound because of an accident and complains of debilitating hot flushes. Her BMI is 29 and her sister died from a pulmonary embolus at the age of 48 years.

Your answer: A Correct answer: I

Your answer: A Correct answer: D

| EXPLANATION | Options for Questions 9-10 A Raloxifene

B Oral sequential oestrogen + progestogen

C Oral continuous oestrogen + progestogen

D Oestrogen patches

E Vaginal oestrogen tablets

F Levonorgestrel IUS + oral oestrogens

G Levonorgestrel IUS

H Refer to special oncology clinic

I

J

Counsel and recommend discontinuation of HRT

HRT not recommended

Instructions: For each scenario described below, choose the single most appropriate management from the above list of options. Each option may be used once, more than once, or not at all. Question 9

Question 10

A 60 year old woman complains of vaginal dryness and superficial / deep dyspareunia since discontinuing oral HRT because of media reports. She has used lubricants without improvement

A 40 year old woman has TAH + BSO for stage Ia endometroid adenocarcinoma of the ovary and complains of debilitating hot flushes.

Your answer: A Correct answer: E Your answer: A Correct answer: D

| EXPLANATION | Options for Questions 11-12 A Raloxifene

B Oral sequential oestrogen + progestogen

C Oral continuous oestrogen + progestogen

D Oestrogen patches

E Vaginal oestrogen tablets

F Tamoxifen

G Levonorgestrel IUS

H Refer to special oncology clinic

I

J

Counsel and recommend discontinuation of HRT

HRT not recommended

Instructions: For each scenario described below, choose the single most appropriate management from the above list of options. Each option may be used once, more than once, or not at all. Question 11

Question 12

A 50 year old woman is known to have anti-thrombin III deficiency. She complains of distressing hot flushes and vaginal dryness 6 months after her last menstrual period A 50 year old woman had been on sequential HRT for 6 months prior to a diagnosis and treatment for breast cancer. She is still having the occasional menstrual period but complains of worsening menopausal symptoms. She wishes to re-commence HRT which was discontinued at the time of breast cancer diagnosis

Your answer: A Correct answer: J

Your answer: A Correct answer: H

| EXPLANATION | Options for Questions 13-14 A Commence iv erythromycin

B Commence oral co-amoxiclav

C Commence oral erythromycin

D Corticosteroids + oral erythromycin

E Administer corticosteroids for fetal lung maturity

F Administer corticosteroids + tocolytic

G Arrange in-utero transfer to tertiary centre

H Arrange ex-utero transfer to tertiary centre

I

J

Immediate caesarean section

Recommend bed-rest

Instructions: For each scenario described below, choose the single most appropriate management from the above list of options. Each option may be used once, more than once, or not at all. Question 13

Question 14

A 24 year old primigravida presents at 30 weeks gestation with painful uterine contractions every 3 minutes but no other symptoms. The cervix is partially effaced and the os is closed. You have been informed that there are no SCBU cots in the unit. A 32 year old P 3+0 with two previous caesarean sections presents with spontaneous rupture of the membranes at 32 weeks gestation. Her temperature is 37.8C, pulse = 110bpm and the baseline fetal heart rate is 180bpm. The uterus is tender with mild contractions every 10 minutes.

Your answer: A Correct answer: F

Your answer: A Correct answer: I

| EXPLANATION | Options for Questions 15-16 A Commence iv erythromycin

B Commence oral co-amoxiclav

C Commence oral erythromycin

D Treat with antenatal iv benzyl-penicillin

E Treat with antenatal oral benzyl-penicillin

F

G Arrange in-utero transfer to tertiary centre

H Arrange ex-utero transfer to tertiary centre

I

J

Insert rescue cervical suture

No treatment required Recommend bed-rest

Instructions: For each scenario described below, choose the single most appropriate management from the above list of options. Each option may be used once, more than once, or not at all. Question 15

A 30 year old primigravida presents with spontaneous rupture of the membranes at 28 weeks gestation. All maternal and fetal observations are within normal limits, there are no uterine contractions and the cervix is not dilated

Question 16

A 32 year old woman presents with spontaneous rupture of the membranes at 28 weeks gestation. All maternal and fetal observations are within normal limits and despite tocolytic therapy, uterine contractions persist and the cervix is now 4cm dilated. The local neonatal unit is unable to accommodate neonates born before 32 weeks

Your answer: A Correct answer: C

Your answer: A Correct answer: H

| EXPLANATION | Options for Questions 17-18 A Commence iv erythromycin

B Commence oral co-amoxiclav

C Commence oral erythromycin

D Corticosteroids + oral erythromycin

E Administer corticosteroids for fetal lung maturity

F No treatment required

G Arrange in-utero transfer to tertiary centre I Insert rescue cervical suture

H Arrange ex-utero transfer to tertiary centre J Recommend bed-rest

Instructions: For each scenario described below, choose the single most appropriate management from the above list of options. Each option may be used once, more than once, or not at all.

Question 17

Question 18

A 24 year old P4+0 with four previous term vaginal deliveries presents at 27 weeks gestation with fresh vaginal bleeding and uterine contractions every 3 minutes. The cervix is partially effaced and 3cm dilated with intact membranes. The placenta is fundal and all other maternal and fetal observations are within normal limits. SCBU cots are available. A 40 year woman with one previous pre-term delivery at 30 weeks gestation presents with regular painful contractions at 32 weeks gestation. Corticosteroids had been administered 5 days earlier when she presented with uterine contractions. The cervix is now fully effaced and 5cm dilated with intact membranes. All other maternal and fetal observations are within normal limits and SCBU cots are available.

Your answer: A Correct answer: E

Your answer: A

Correct answer: F

| EXPLANATION | Options for Questions 19-20 A Raloxifene

B Oral sequential oestrogen + progestogen

C Oral continuous oestrogen + progestogen

D Oestrogen patches

E Vaginal oestrogen tablets

F Tamoxifen

G Levonorgestrel IUS

H Refer to special oncology clinic

I

J

Counsel and recommend discontinuation of HRT

HRT not recommended

Instructions: For each scenario described below, choose the single most appropriate management from the above list of options. Each option may be used once, more than once, or not at all. Question 19

Question 20

A 40 year old woman has TAH + BSO for stage Ia ovarian cancer and complains of distressing hot flushes and vaginal dryness

A 52 year old woman with a previous PE complains of infrequent periods and distressing hot flushes. Her sister died from PE while on the COCP at the age of 38 years.

Your answer: A Correct answer: D Your answer: A Correct answer: J

Options for Questions 1-2 A Counsel and offer amniocentesis

B Counsel and offer chorionic villus sampling

C Counsel and offer pregnancy termination

D Counsel and offer intra-uterine therapy

E Reassure

F

G Counsel and offer fetal blood sampling

H Counsel and offer intra-uterine transfusion

I

Counsel and offer induction of labour at 36-38 weeks gestation

J

Counsel and offer fetal surgery

Counsel and offer serial fetal growth scans

K Counsel and refer for neonatal assessment after delivery

Instructions: For each of the case histories described below, choose the single most appropriate management from the above list of options. Each option may be used once, more than once, or not at all. Question 1

Question 2

A 20 year old woman is found to have a fetus with an isolated choroids plexus cyst at 21 weeks gestation. Integrated test showed a risk of Down’s syndrome of 1:5000

A 20 year old woman is found to have a fetus with gastroschisis but no other anomalies at 21 weeks gestation.

Your answer: A Correct answer: E Your answer: A Correct answer: J

| EXPLANATION | Options for Questions 3-4 A Tranexamic acid

B Hysteroscopy + polypectomy

C Danazol

D Fibroid embolisation

E Mefenamic acid

F

G Trans-cervical resection of fibroid

H Levonorgestrel IUS

I

J

Combined oral contraceptive pill

GnRH analogues Pipelle endometrial biopsy

Instructions: For each scenario described below, choose the single most appropriate management from the above list of options. Each option may be used once, more than once, or not at all. Question 3

A 17 year old woman complains of painful periods since menarche which are now interfering with her studies. She is not sexually active and does not wish to take contraceptives.

Your answer: A Correct answer: E

Question 4

A 30 year old mother of 2 children complains of heavy regular periods that were improved by cyclical progestogens. She has, however, discontinued treatment because of weight gain. She was sterilised 2 years ago.

Your answer: A Correct answer: H

| EXPLANATION | Options for Questions 5-6 A Myomectomy

B Hysteroscopy + polypectomy

C Danazol

D Fibroid embolisation

E TAH + BSO

F

G Trans-cervical resection of fibroid

H Tranexamic acid

I

J

Combined oral contraceptive pill

GnRH analogues Pipelle endometrial biopsy

Instructions: For each scenario described below, choose the single most appropriate management from the above list of options. Each option may be used once, more than once, or not at all. Question 5

Question 6

A 45 year old mother of 4 children complains of progressively heavy but regular periods. Clinical examination shows a 15 weeks size fibroid uterus and endometrial biopsy showed proliferative endometrium. Medical therapy has failed but she does not wish to have major surgery A 35 year old mother of 4 children complains of progressively heavy periods. Over the last 2 months, she has bled severely and required admission for blood transfusion. She has been listed for total abdominal hysterectomy in 3 months time

Your answer: A Correct answer: D

Your answer: A Correct answer: F

| EXPLANATION | Options for Questions 7-8 A Myomectomy

B Hysteroscopy + polypectomy

C Danazol

D Fibroid embolisation

E TAH + BSO

F

G Trans-cervical resection of fibroid

H Levonorgestrel IUS

I

J

Combined oral contraceptive pill

GnRH analogues Pipelle endometrial biopsy

Instructions: For each scenario described below, choose the single most appropriate management from the above list of options. Each option may be used once, more than once, or not at all. Question 7

Question 8

A 35 year old woman has been trying for a pregnancy for 12 months and complains of heavy periods. Clinical examination and pelvic ultrasound scan confirm the presence of a 15 weeks size fibroid uterus but no other abnormalities A 30 year old woman complains of progressively heavy periods over 2 years. Pelvic ultrasound scan shows a slightly enlarged uterus with multiple small fibroids. The endometrium is distorted by a sub-mucous fibroid

| EXPLANATION |

Your answer: A Correct answer: A

Your answer: A Correct answer: G

Options for Questions 9-10 A Sequential combined HRT

B Hysteroscopy + polypectomy

C Danazol

D Fibroid embolisation

E TAH + BSO

F

G Trans-cervical resection of fibroid

H Levonorgestrel IUS

I

J

Combined oral contraceptive pill

GnRH analogues Pipelle endometrial biopsy

Instructions: For each scenario described below, choose the single most appropriate management from the above list of options. Each option may be used once, more than once, or not at all. Question 9

Question 10

A 30 year old woman complains of a 2 year history of progressively heavy periods. She was on the oral contraceptive pill until 3 years ago when she developed a DVT during pregnancy. Clinical examination is normal and she does not wish to take tablets. A 43 year old mother of 5 children complains of a 5 year history of progressively heavy periods that have failed to respond to medical treatment. Trans-vaginal ultrasound scan performed in the proliferative phase shows a normal pelvis with a regular 14mm endometrium

Your answer: A Correct answer: H

Your answer: A Correct answer: J

| EXPLANATION | Options for Questions 11-12 A Counsel and offer amniocentesis C Counsel and offer pregnancy termination

B sampling Counsel and offer chorionic villus D Counsel and offer intra-uterine therapy

E Reassure

F

G Counsel and offer fetal blood sampling

H Counsel and offer intra-uterine transfusion

Counsel and offer induction of labour at 36-38 weeks gestation

J

K Counsel and refer for neonatal assessment after delivery

L

I

Counsel and offer fetal surgery

Counsel and offer serial fetal growth scans Counsel and offer fetal echocardiography

Instructions: For each of the case histories described below, choose the single most appropriate management from the above list of options. Each option may be used once, more than once, or not at all. Question 11

Question 12

A 35 year old woman is found to have a fetus with a cleft lip and palate on anomaly scan at 20 weeks gestation

A 42 year old woman has had amniocentesis with a normal karyotype at 16 weeks gestation. The fetus is found to have talipes equinovarus on anomaly scan at 20 weeks gestation

Your answer: A Correct answer: A Your answer: A Correct answer: K

| EXPLANATION | Options for Questions 13-14 A Sequential combined HRT

B Hysteroscopy + polypectomy

C Danazol

D Fibroid embolisation

E TAH + BSO

F

GnRH analogues

G Trans-cervical resection of fibroid

H Tranexamic acid

I

J

Combined oral contraceptive pill

Hysteroscopy D&C

Instructions: For each scenario described below, choose the single most appropriate management from the above list of options. Each option may be used once, more than once, or not at all. Question 13

Question 14

A 48 year old woman complains of a 6 months history of progressively heavy and irregular periods (occurring every 4-7 weeks) with occasional hot flushes. Trans-vaginal ultrasound scan shows a normal pelvis with a 7mm regular endometrium A 48 year old woman complains of continuous vaginal bleeding over the last 6 months, varying from slight staining to heavy period-like bleeding. Trans-vaginal ultrasound scan shows a normal pelvis with a 7mm regular endometrium

Your answer: A Correct answer: A

Your answer: A Correct answer: J

| EXPLANATION | Options for Questions 15-16 A Sequential combined HRT

B Hysteroscopy + polypectomy

C Danazol

D Fibroid embolisation

E TAH + BSO

F

G Trans-cervical resection of fibroid

H Mefenamic acid

I

J

Combined oral contraceptive pill

GnRH analogues Hysteroscopy D&C

Instructions: For each scenario described below, choose the single most appropriate management from the above list of options. Each option may be used once, more than once, or not at all. Question 15

Question 16

A 30 year old woman complains of a 2 year history of progressively heavy periods. She was on the oral contraceptive pill until 3 years ago when she developed a DVT during pregnancy. Clinical examination is normal. A 30 year old nulliparous woman is referred to the gynaecology clinic because of progressively heavy periods. She has been treated with oral progestogens, mefenamic acid and tranexamic acid without improvement.

Your answer: A Correct answer: H

Your answer: A Correct answer: J

| EXPLANATION | Options for Questions 17-18 A Counsel and offer amniocentesis

B Counsel and offer chorionic villus sampling

C Counsel and offer pregnancy termination

D Counsel and offer intra-uterine therapy

E Reassure

F

Counsel and offer fetal surgery

G Counsel and offer fetal blood sampling

H

Counsel and offer intra-uterine transfusion

Counsel and offer induction of labour at 36-38 weeks gestation

J

Counsel and offer serial fetal growth scans

K Counsel and refer for neonatal assessment after delivery

L

I

Counsel and offer fetal echocardiography

Instructions: For each of the case histories described below, choose the single most appropriate management from the above list of options. Each option may be used once, more

than once, or not at all. Question 17

Question 18

A 35 year old woman with phenylketonuria presents for anomaly scan at 21 weeks gestation.

A 20 year old woman with a history of tetralogy of Fallot corrected in childhood is referred for antenatal care at 10 weeks gestation.

Your answer: A Correct answer: L Your answer: A Correct answer: D

| EXPLANATION | Options for Questions 19-20 Counsel and offer chorionic villus sampling

A Counsel and offer amniocentesis

B

C Counsel and offer pregnancy termination

D Counsel and offer intra-uterine therapy

E Reassure

F

Counsel and offer fetal surgery

G Counsel and offer fetal blood sampling

H

Counsel and offer intra-uterine transfusion

J

Counsel and offer serial fetal growth scans

I

Counsel and offer induction of labour at 36-38 weeks gestation

K Counsel and refer for neonatal assessment after delivery

Instructions: For each of the case histories described below, choose the single most appropriate management from the above list of options. Each option may be used once, more than once, or not at all. Question 19

Question 20

A 20 year old woman is found to have a fetus with an exomphalos at 21 weeks gestation

A 30 year old Rhesus negative woman with Rhesus D antibodies is found to have a hydropic fetus at 26 weeks gestation

Your answer: A Correct answer: A Your answer: A Correct answer: H

Options for Questions 1-2 A Chorionic villus sampling

B Amniocentesis

C Fetal blood sampling

D Triple test

E Karyotype

F

G Western blotting

H Northern blotting

I

J

Anomaly scan

Integrated test Southern blotting

K PCR

Instructions: For each of the case histories described below, choose the single most relevant investigation from the above list of options. Each option may be used once, more than once, or not at all. Question 1

Question 2

A 20 year old HIV positive woman presents for antenatal care at 20 weeks gestation

A 35 year old woman is known to be a balanced 14:21 translocation carrier and has undergone amniocentesis at 16 weeks gestation

Your answer: A Correct answer: I Your answer: A Correct answer: E

| EXPLANATION | Options for Questions 3-4 A Le Fort procedure

B Boari flap procedure

C Catheterisation

D Short bladder distension

E Urethral dilatation

F Peri-urethral collagen injection

G Urodynamic studies

H Cystoscopy

I

Anterior colporrhaphy

K Sacro-colpopexy

J

Pelvic floor physiotherapy

L

Long-term intermittent self-catheterisation

Instructions: For each scenario described below, choose the single most appropriate management from the above list of options. Each option may be used once, more than once, or not at all. Question 3

Question 4

A 35 year old woman re-attends 6 days after TAH for menorrhagia because of persistent leakage of clear fluid from the vagina. Speculum examination showed the presence of urine in the vagina and a three swab test showed the presence of dye in the vagina. A healthy 57 years old woman complains of a vaginal lump 20 years after total anbdominal hysterectomy for a large fibroid uterus. Clinical examination showed marked vault prolapse.

Your answer: A Correct answer: C

Your answer: A Correct answer: K

| EXPLANATION | Options for Questions 5-6 A Duloxetine

B Tolterodine

C Amitriptyline

D Short bladder distension

E Urethral dilatation

F Tolterodine + bladder re-training

G Urodynamic studies

H Cystoscopy

I

Prolonged bladder distension

K Solifenacine

J

Pelvic floor physiotherapy

L

Long-term intermittent self-catheterisation

Instructions: For each scenario described below, choose the single most appropriate management from the above list of options. Each option may be used once, more than once, or not at all. Question 5

Question 6

A 33 year old woman complains of debilitating urinary frequency, urgency and urge incontinence but no other symptoms. Clinical examination is normal and urine microscopy and culture are negative. A 34 year old nulliparous woman complains of urinary frequency, urgency, deep dyspareunia and bladder pain. Urine dipstix shows persistent haematuria but culture is negative.

| EXPLANATION | Options for Questions 7-8 A Cystoscopy and cystodistension

B Bladder re-training

C Tension-free vaginal tape

D Anterior repair + TVT

E Posterior repair

F Paravaginal repair

Your answer: A Correct answer: F

Your answer: A Correct answer: H

G Para-urethral collagen injection I

Pelvic floor physiotherapy

K Intermittent self-catherterisation

H Hysterectomy J

Burch colposuspension

L

Oestrogen replacement therapy

Instructions: For each of the case histories described below, choose the single most appropriate treatment option from the above list of options. Each option may be used once, more than once, or not at all. Question 7

fibroid uterus with a small cystocele but no rectocele

Question 8

Your answer: A

A 45 year old woman complains of urinary frequency, urgency and urge incontinence. Clinical examination showed an 18 weeks size

A 45 year old woman remains in hospital 8 days after Burch colposuspension because she is unable to empty her bladder spontaneously. The supre-pubic catheter has been removed and she now has a urethral catheter

Correct answer: H Your answer: A Correct answer: K

| EXPLANATION | Options for Questions 9-10 A Cystoscopy

B MRI scan

C Urodynamic studies

D Bladder re-training

E Urine for cytology

F Cystoscopy and biopsy

G Examination under anaesthesia

H MSU for culture and sensitivity

I

J

Urine electrolytes

Renal function tests

Instructions: For each of the case histories described below, choose the single most important investigation from the above list of options. Each option may be used once, more than once, or not at all. Question 9

Question 10

Your answer: A

A 60 year old woman developed cervical cancer 12 months earlier and was treated with combined chemotherapy and radiotherapy. She now complains of blood in her urine which occurs all the time A 45 year old woman complained initially of urinary frequency urgency and urge incontinence and also leaked urine on coughing or straining. She was treated medically initially and her symptoms of urinary frequency and urgency have largely resolved. Over the last 6 months, she is finding that urinary leakage on coughing and straining is increasingly affecting her social life.

Correct answer: F

Your answer: A Correct answer: C

| EXPLANATION | Options for Questions 11-12 A Congenital syphilis

B Congenital varicella syndrome

C Parvovirus B19 infection

D Turner’s syndrome

E Parder-Willy syndrome

F

G Group B streptococcal infection

H Congenital rubella syndrome

I

Cri-du-chat syndrome

K Patau’s syndrome

Fetal hydantoin syndrome

J

Edward’s syndrome

L

Down’s syndrome

Instructions: For each of the case histories described below, choose the single most likely cause of fetal abnormality from the above list of options. Each option may be used once,

more than once, or not at all. Question 11

Question 12

A 42 year old woman is late booking for antenatal care. The estimated gestation age by ultrasound scan is 22 weeks and the fetus is found to have an atrio-ventricular septal defect and a double bubble sign in the upper abdomen A 42 year old woman had bleeding in early pregnancy and was found to have an 8 week viable pregnancy. Anomaly scan at 22 weeks showed a head and abdominal circumferences below the third centile , bilateral choroids plexus cysts and rockerbottom feet

Your answer: A Correct answer: L

Your answer: A Correct answer: J

| EXPLANATION | Options for Questions 13-14 A Stress incontinence

B Urinary retention with overflow

C Urinary retention

D Detrusor instability

E Detrusor hyper-reflexia

F Interstitial cystitis

G Genuine stress incontinence

H Mixed urinary incontinence

I

J

Bladder tumour

Urinary tract infection

K Diabetes mellitus

Instructions: For each of the case histories described below, choose the single most likely diagnosis from the above list of options. Each option may be used once, more than once, or not at all. A 30 year old woman complains of severe lower abdominal pain 3 Question 13

Question 14

days after hysterectomy. The urethral catheter had been removed 12h earlier and she has voided spontaneously on 2 occasions (total volume 75ml). Clinical examination shows a 14 week size tender supra-pubic mass.

A 35 year old woman with multiple sclerosis complains of urinary frequency, urgency and urge incontinence

Your answer: A Correct answer: C

Your answer: A Correct answer: E

| EXPLANATION | Options for Questions 15-16 A Amniocentesis and karyotype

B Amniocentesis and PCR

C CVS and karyotype

D CVS and PCR

E Amniocentesis and enzyme assay

F

G Fetal blood sampling and measurement of PO2

Fetal blood sampling and haemoglobin H electrophoresis

I

Fetal blood sampling and haemoglobin concentration

J

Fetal blood sampling and karyotype

Fetal blood sampling and viral culture

K Reassurance

Instructions: For each of the case histories described below, choose the single most appropriate management from the above list of options. Each option may be used once, more than once, or not at all. Question 15

A 30 year old woman with sickle cell disease is concerned about passing the disease onto her child. On haemoglobin electrophoresis, her partner is found to have haemoglobin AA.

Your answer: A Correct answer: K

Question 16

A 30 year old woman had amniocentesis because of a high risk of Down’s syndrome on serum screening. The karyotype is reported as 46XX. She is now 20 weeks pregnant and has discovered that she is a carrier of Duchenne muscular dystrophy after her sister’s son was diagnosed with the condition. She is worried about passing the condition to her child.

Your answer: A Correct answer: K

| EXPLANATION | Options for Questions 17-18 A Autosomal dominant

B Autosomal recessive

C X-linked recessive

D X-linked dominant

E Autosomal dominant with incomplete penetrance

F

G Polygeneic transmission

H Confined placental mosaicism

I

Non-mendelian inheritance

K X-linked recessive with lyonisation

Autosomal dominant with variable expressivity

J

Inheritance through mitochondrial DNA

L

X-linked dominant with lyonisation

Instructions: For each of the case histories described below, choose the single most likely mode of inheritance from the above list of options. Each option may be used once, more than once, or not at all. Question 17

Question 18

A 25 year old woman has been referred for pre-natal diagnosis. Her father and brother suffer from an inherited condition. She is found to be mildly affected by the condition and will require assessment and treatment during labour. She is found to be carrying a female fetus and has been reassured. A 25 year old primigravida is married to her first cousin. At 20 weeks gestation, anomaly scan shows multiple fetal abnormalities. Both parents have been found to have normal karyotypes.

Your answer: A Correct answer: K

Your answer: A Correct answer: B

| EXPLANATION | Options for Questions 19-20 A Autosomal dominant

B Autosomal recessive

C X-linked recessive

D X-linked dominant

E Autosomal dominant with incomplete penetrance

F

G Polygeneic transmission

H Confined placental mosaicism

I

Non-mendelian inheritance

K X-linked recessive with lyonisation

Autosomal dominant with variable expressivity

J

Inheritance through mitochondrial DNA

L

X-linked dominant with lyonisation

Instructions: For each of the case histories described below, choose the single most likely mode of inheritance from the above list of options. Each option may be used once, more than once, or not at all. Question 19

Question 20

A 25 year old woman is known to be mildly affected by an inherited condition. Her father suffered from a severe form of the same condition and died at the age 30 years. She has been found to have a female fetus with a severe form of the condition and is requesting termination of pregnancy A 25 year old woman has been referred for pre-natal diagnosis. Her father and brother suffer from an inherited condition. She is found to be carrying a female fetus and has been reassured.

Your answer: A Correct answer: F

Your answer: A Correct answer: C

| EXPLANATION | Options for Questions 21-22 A Rocker bottom feet

B Exomphalos

C Holoprosencephaly

D Cystic hygroma

E Gastroschisis

F atrio-ventricular septal defect

G Intra-cranial calcification

H Duodenal atresia

I

J

Spina bifida

Polyhydramnios

K Echogenic bowel

Instructions: For each of the conditions described below, choose the single most characteristic ultrasound marker from the above list of options. Each option may be used once, more than once, or not at all. Your answer: A Question 21

Trisomy 18 Correct answer: A Your answer: A

Question 22

Cystic fibrosis Correct answer: K

| EXPLANATION | Options for Questions 23-24 A Amniocentesis

B Chorionic villus sampling

C Fetal blood sampling

D Triple test

E Quadruple test

F

G Serum integrated test

H Nuchal transluscency

I

Anomaly scan

K PCR

Integrated test

J

Fluorescent in-situ hybridisation

L

Restriction fragment length polymorphisms

Instructions: For each of the case histories described below, choose the single most appropriate investigation from the above list of options. Each option may be used once, more than once, or not at all. Question 23

Question 24

A 17 year old woman books for antenatal care at 16 weeks gestation and would like to know whether or not her baby has Down’s syndrome

A 30 year old woman is known to be a carrier of Haemophilia A. Amniocentesis at 16 weeks gestation shows that she is carrying an unaffected male fetus. She is now 20 weeks pregnant.

| EXPLANATION | Options for Questions 25-26 A Chorionic villus sampling

B Amniocentesis

C Fetal blood sampling

D Triple test

E Karyotype

F

G Western blotting

H Northern blotting

Integrated test

Your answer: A Correct answer: E Your answer: A Correct answer: I

I

J

Anomaly scan

Southern blotting

K PCR

Instructions: For each of the case histories described below, choose the single most relevant investigation from the above list of options. Each option may be used once, more than once, or not at all. Question 25

Question 26

A 35 year old woman with myotonic dystrophy is undergoing IVF treatment in order to ensure that the disease is not passed on to her child

A 35 year old woman is found to have raised maternal serum AFP at 16 weeks gestation. Anomaly scan is equivocal because of high body mass index and she has opted to have amniocentesis

Your answer: A Correct answer: K Your answer: A Correct answer: G

| EXPLANATION | Options for Questions 27-28 A Risk of Down’s syndrome of ~1:10

B Risk of culture failure

C Risk of miscarriage of 0.5-1% above background

D Risk of miscarriage of 1-2% above background

E Risk of fetal death

F Risk of confined placental mosaicism

G Risk of contamination with maternal blood

H Risk of fetal bradycardia requiring emergency caesarean section J Risk of trisomy 18

I

Risk of Rhesus iso-immunisation

K Risk of infection

L

Risk of fetal needle injury

Instructions: Instructions: For each of the case histories described below, choose the single most relevant complication that you must discuss with the patient when taking consent prior to invasive testing from the above list of options. Each option may be used once, more than once, or not at all. Question 27

Question 28

A 42 year old primigravida is found to have a risk of having a baby with Down’s syndrome of 1:10 following integrated testing and is scheduled to undergo amniocentesis at 17 weeks gestation A 42 year old woman with a previous baby with cystic fibrosis. She and her partner have both been found to be carriers of the cystic fibrosis gene mutation and she is scheduled to undergo chorionic villus sampling at 11 weeks gestation

Your answer: A Correct answer: C

Your answer: A Correct answer: D

| EXPLANATION | Options for Questions 29-30 A Amniocentesis

B Chorionic villus sampling

C Fetal blood sampling

D Triple test

E Quadruple test

F

G Serum integrated test

H Nuchal transluscency

I

Anomaly scan

K PCR

Integrated test

J

Fluorescent in-situ hybridisation

L

Restriction fragment length polymorphisms

Instructions: For each of the case histories described below, choose the single most appropriate investigation from the above list of options. Each option may be used once,

more than once, or not at all. Question 29

Question 30

A 30 year old Rhesus positive woman is found to have a hydropic fetus at 22 weeks gestation. No other anomalies have been identified on detailed ultrasound examination.

A 30 year old woman is found to have a ‘high risk’ of having a baby with Down’s syndrome following serum screening. Amniocentesis has been performed at 18 weeks gestation.

Your answer: A Correct answer: C Your answer: A Correct answer: J

| EXPLANATION | Options for Questions 31-32 A Fetal blood sampling

B Triple test

C Quadruple test

D Integrated test

E Chorionic villus sampling

F

G Amniocentesis

H Nuchal transluscency

I

Anomaly scan

K PCR

Serum integrated test

J

Fluorescent in-situ hybridisation

L

Restriction fragment length polymorphisms

M Karyotype

Instructions: For each of the conditions described below, choose the single most appropriate diagnostic tests from the above list of options. Each option may be used once, more than once, or not at all. Your answer: A Question 31

Fragile X-syndrome Correct answer: M Your answer: A

Question 32

Down’s syndrome Correct answer: M

| EXPLANATION | Options for Questions 33-34 A Fetal blood sampling

B Triple test

C Quadruple test

D Integrated test

E Chorionic villus sampling

F

G Amniocentesis

H Nuchal transluscency

I

Anomaly scan

K PCR

Serum integrated test

J

Fluorescent in-situ hybridisation

L

Restriction fragment length polymorphisms

M Karyotype

Instructions: For each of the conditions described below, choose the single most appropriate diagnostic tests from the above list of options. Each option may be used once, more than once, or not at all. Your answer: A Question 33

Fetal anaemia Correct answer: A

Question 34

Fetal gastroschisis

Your answer: A

Correct answer: I

| EXPLANATION | Options for Questions 35-36 A Cystoscopy and cystodistension

B Bladder re-training

C Tension-free vaginal tape

D Anterior repair

E Posterior repair

F Paravaginal repair

G Para-urethral collagen injection

H Tolterodine + bladder re-training

I

Solifenacin

K Amitriptyline

J

Duloxetine

L

Oestrogen replacement therapy

Instructions: For each of the case histories described below, choose the single most appropriate treatment option from the above list of options. Each option may be used once, more than once, or not at all. Question 35

Question 36

A 65 year old mother of 6 children presents with a 3 year history of urinary frequency, leakage on coughing and straining. Urodynamic investigations confirm a diagnosis of genuine stress incontinence

A 35 year old woman complains of urinary frequency, urgency and urge incontinence but no other symptoms.

Your answer: A Correct answer: C Your answer: A Correct answer: H

| EXPLANATION | Options for Questions 37-38 A Abdominal pressure due to mass

B Bladder neck weakness

C Pelvic floor muscle damage

D Congenital weakness of the pelvic floor

E Obstetric trauma

F Oestrogen deficiency

G Neurological disease

H Drug side-effects

I

Tumour infiltration into bladder

K Schistosomiasis

J

Bacterial urinary tract infection

L

Radiation injury

Instructions: For each of the case histories described below, choose the single most likely cause of urinary tract symptoms from the above list of options. Each option may be used once, more than once, or not at all. Question 37

Question 38

A 35 year old woman complains of a 2 month history of urinary frequency, urgency and haematuria. She had an abdominal X-ray 2 weeks earlier for an unrelated complaint which showed evidence of bladder calcification. Cystoscopy showed numerous polypoid lesions in the bladder A 52 year old woman complains of progressively worsening urinary frequency and urgency 3 years after radical abdominal hysterectomy and radiotherapy for cervical cancer. There is no haematuria and all investigations including cystoscopy are normal

| EXPLANATION | Options for Questions 39-40

Your answer: A Correct answer: K

Your answer: A Correct answer: F

A Cystoscopy & cystodistension

B Cystoscopy

C Urine microscopy, culture and sensitivity

D Cystometry

E Pelvic ultrasound scan

F Colposuspension

G Bladder retraining

H Three swab test

I

Intra-venous urogram

K Catheterise

J

Return to theatre

L

Pelvic floor exercises

Instructions: For each of the case histories described below, choose the single most appropriate initial management from the above list of options. Each option may be used once, more than once, or not at all. Question 39

Question 40

A 60 year old woman complains of urinary frequency and urgency for 9 months and the presence of blood in her urine over the last 7 days

A 35 year old woman complains of feeling wet all the time 10 days after an uneventful colposuspension. Her initial symptoms of urinary leakage on coughing have improved significantly.

Your answer: A Correct answer: B Your answer: A Correct answer: H

| EXPLANATION | Options for Questions 1-2 A Offer hepatitis B passive immunisation to woman

B Offer hepatitis B active immunisation to woman

C Deliver by caesarean section

D Amniocentesis for fetal viraemia by PCR F Offer hepatitis B passive immunisation to neonate from day 7

E Fetal blood sampling for viral culture G I

Offer hepatitis B active immunisation to neonate Avoid breast-feeding

K Reassurance

H

Offer hepatitis B active & passive immunisation to neonate

J

Intra-venous acyclovir in labour

L

Screen for hepatitis B infection

Instructions: For each scenario described below, choose the single most appropriate management from the above list of options. Each option may be used once, more than once, or not at all. Question 1

A 30 year old woman has hepatitis B screening at 14 weeks gestation with the following results: Hep B surface antigen negative, antibodies to Hep B surface antigen negative

Your answer: A Correct answer: K Your answer: A

Question 2

A 25 yearinold woman withlabour a history ofweeks hepatitis B infection presents spontaneous at 39 gestation. The cervix is 2cm dilated with intact membranes.

Correct answer: L

| EXPLANATION | Options for Questions 3-4 A Avoid breast-feeding C Administer VZIG to neonate but breast-feeding encouraged E Delay delivery with tocolytics

B Administer VZIG to neonate and avoid breastfeeding D Immediate induction of labour F Offer termination of pregnancy

G Treat neonate with oral acyclovir

H Treat neonate with intravenous acyclovir

Examine neonate for evidence of congenital I anomalies

J

Reassurance

K Delay delivery for 5-7 days if possible

L

Delay discharge for at least 7 days

Instructions: For each scenario described below, choose the single most appropriate management from the above list of options. Each option may be used once, more than once, or not at all. A 30 year old woman has delivered her second child by Your answer: A Question 3

Question 4

caesarean section for breech presentation. shehas is in hospital, she is informed that her 5 year old While daughter chickenpox. She has had chicken pox in the past but is concerned about taking her baby home A 20 year old woman is in hospital 2 days after vaginal delivery of her first baby and is informed that her 8 year old step-daughter has chicken-pox. She does not remember having chicken pox in the past and is worried about taking her new baby home. Investigations show that she is varicella zoster IgG negative and IgM negative

Correct answer: J

Your answer: A Correct answer: L

| EXPLANATION | Options for Questions 5-6 A Endometriosis

B Acute PID

C Chronic PID

D Irritable bowel syndrome

E Residual ovary syndrome

F

G Diverticular disease

H Interstitial cystitis

I

Ovarian remnant syndrome

J

Adenomyosis

K Uterine fibroids

L

Polycystic ovary syndrome

M Inflammatory bowel disease

N Chronic constipation

Primary dysmenorrhoea

Instructions: For each of the case histories described below, choose the single most likely cause of chronic pelvic pain from the above list. Each option may be used once, more than once, or not at all.

Question 5

Question 6

A 34 year old mother of 3 children with a 10 year menorrhagia underwent a total abdominal hysterectomy with conservation of the ovaries 2 years earlier. She now complains of persistent pelvic pain that is worse when her period would have been due and deep dyspareunia. Pelvic ultrasound scan shows small bilateral ovarian cysts 2-3cm in diameter but no other abnormalities. A 35 year old woman complains of a 10 months history of pelvic pain, dysmenorrhoea and deep dyspareunia since discontinuing the combined oral contraceptive pill. She also complains of rectal bleeding during menstruation but has a regular bowel habit and there is no recent weight loss. Clinical examination showed a bulky tender uterus with decreased mobility and palpable nodules within the pouch of Douglas. Rectal examination was normal.

| EXPLANATION | Options for Questions 7-8 A Tranexamic acid

B Danazol

Your answer: A Correct answer: E

Your answer: A Correct answer: A

C GnRH analogues

D Laparoscopy and ablation of endometriosis

E Diagnostic laparoscopy

F Mefenamic acid

G Pelvic ultrasound scan

H MRI scan of the pelvis J

Endocervical and urethral swabs

K Combined oral contraceptive pill

I

L

Depo-medroxyprogesterone acetate

M Refer to pain clinic

N Refer to psychiatrist

Dilatation and curettage

Instructions: For each of the case histories described below, choose the single most appropriate management from the above list. Each option may be used once, more than once, or not at all.

Question 7

Question 8

A 35 year old woman attends the gynaecology clinic with her 14 year old daughter who is complaining of severe period pains since the onset of menstruation at the age of 13 years. Menstrual loss is not heavy and she has no bowel symptoms. Her symptoms are now interfering with her education as she takes several days off school every month. She does not require contraception and clinical examination is normal. A 33 year old mother of 3 children complains of progressively painful periods over 5 years. There is occasional deep dyspareunia but she has no bowel symptoms. Her symptoms have not responded to simple analgesia and non-steroidal antiinflammatory drugs. Clinical examination is normal.

Your answer: A Correct answer: F

Your answer: A Correct answer: K

| EXPLANATION | Options for Questions 9-10 A Maternal treatment with antibiotics

B Detailed fetal anomaly scan

C Deliver by caesarean section

D Active immunisation of neonate

E Avoid breast-feeding

F

G Delay delivery by 5-7 days

H Administer corticosteroids

I

J

Administer tocolytics

Counsel and offer termination of pregnancy Reassurance

K Screen for maternal syphilis infection

Instructions: For each scenario described below, choose the single most appropriate management from the above list of options. Each option may be used once, more than once, or not at all. Question 9

Question 10

A 20 year old woman presents with intermittent abdominal pain and a febrile illness at 20 weeks gestation. Blood cultures are positive for Listeria monocytogenes

A 30 year old woman has the following results following routine antenatal screening at 15 weeks gestation: VDRL positive, FTAabs positive

Your answer: A Correct answer: A Your answer: A Correct answer: A

| EXPLANATION | Options for Questions 11-12 A Counsel and offer termination of pregnancy

B Counsel and offer rubella immune globulin

C Treat with intravenous zidovudine

D Counsel and offer intra-uterine therapy

E Offer amniocentesis for karyotype

F Reassurance

G Immunise neonate against rubella

H Immunise mother against rubella

I

Treat with intravenous acyclovir

J

Amniocentesis and viral culture

K Amniocentesis and PCR for fetal viraemia

Instructions: For each scenario described below, choose the single most appropriate management from the above list of options. Each option may be used once, more than once, or not at all. Question 11

Question 12

A 20 year old student has been immunised against Rubella 2 weeks before travelling to the UK. She attends as an emergency because she has missed a period and is found to be 10 weeks pregnant on ultrasound scan. She is concerned about the effect of the vaccine on her fetus A 30 year old school teacher is 32 weeks pregnant and is concerned because she has been in contact with a child who has been diagnosed with rubella. Antenatal screening had revealed low antibody titres and post-partum immunisation had been recommended.

Your answer: A Correct answer: F

Your answer: A Correct answer: F

| EXPLANATION | Options for Questions 13-14 A Endometriosis

B Acute PID

C Chronic PID

D Irritable bowel syndrome

E Residual ovary syndrome

F

G Diverticular disease

H Interstitial cystitis

I Primary dysmenorrhoea K Uterine fibroids

J L

M Inflammatory bowel disease

N Chronic constipation

Ovarian remnant syndrome

Adenomyosis Polycystic ovary syndrome

Instructions: For each of the case histories described below, choose the single most likely cause of chronic pelvic pain from the above list. Each option may be used once, more than once, or not at all.

Question 13

A 23 year old woman has been referred to the gynaecology clinic with a 6 months history of lower abdominal and pelvic pain and deep dyspareunia. Her symptoms are particularly worse during menstruation. Over the last 4 months, she has experienced episodes of bloody diarrhoea and has lost about 5kg in weight. Her Hb = 10.0g/dl, white cell count = 15 x 10E9/l and platelet count = 450 x 10E9/l.

Your answer: A Correct answer: M

A 35 year old woman complains of a 2 year history of progressively painful periods with occasional deep pain during Question 14

intercourse. She denies any bowel or urinary symptoms. Clinical examination showed a bulky smooth and tender uterus with no adnexal masses and no adnexal tenderness. Her symptoms have not responded to non-steroidal anti-inflammatory agents and diagnostic laparoscopy was normal.

| EXPLANATION | Options for Questions 15-16 A Tranexamic acid

B Danazol

C GnRH analogues

D TAH + BSO

E Diagnostic laparoscopy

F Medroxyprogesterone acetate

Your answer: A Correct answer: J

G Pelvic ultrasound scan I

H MRI scan of the pelvis J

Endocervical and urethral swabs

K Combined oral contraceptive pill

L

Levonorgestrel releasing intra-uterine system

M Refer to pain clinic

N Refer to gastroenterologist

Dilatation and curettage

Instructions: For each of the case histories described below, choose the single most appropriate management from the above list. Each option may be used once, more than once, or not at all. A 22 year old woman is known to have endometriosis diagnosed Question 15

Question 16

at laparoscopy. She is referred to the gynaecology clinic because of progressively severe pelvic pain, dysmenorrhoea and deep dyspareunia. There is a past medical history of focal migraine. Clinical examination showed a tender pelvis with a normal size anteverted uterus and no adnexal masses. A 22 year old woman is referred to the gynaecology clinic because of a 2 year history of persistent pelvic pain which is worse during and after intercourse and during menstruation. She also complains of intermittent abdominal bloating with alternating constipation and diarrhoea and urgency of defecation. There is occasional rectal bleeding which is unrelated to her menses and usually occurs during episodes of constipation. Clinical examination is normal and there is no significant past medical history.

Your answer: A Correct answer: F

Your answer: A Correct answer: N

| EXPLANATION | Options for Questions 17-18 A Refer to surgical team

B Danazol

C GnRH analogues

D TAH + BSO

E Diagnostic laparoscopy

F Medroxyprogesterone acetate

G Pelvic ultrasound scan

H MRI scan of the pelvis J

Endocervical and urethral swabs

K Combined oral contraceptive pill

I

L

Levonorgestrel releasing intra-uterine system

M Refer to pain clinic

N Refer to gastroenterologist

Dilatation and curettage

Instructions: For each of the case histories described below, choose the single most appropriate management from the above list. Each option may be used once, more than once, or not at all. A 78 year old woman has been investigated by the gynaecologists over a 2 year period for persistent pelvic pain and was admitted overnight because of severe pain with nausea and vomiting. Her Question 17

Question 18

LMP was over 20 years ago and there is no post-menopausal bleeding. On admission, she had a temperature of 38C, pulse 110bpm with marked left iliac fossa tendernedd. Her CRP and white cell count are elevated and pelvic ultrasound scan shows a normal size uterus with normal ovaries and a 7cm complex mass adjacent to the left ovary. A 34 year old woman with severe endometriosis which has not responded to the combined oral contraceptive pill and progestogens has undergone laparoscopy with laser ablation of endometriosis. Six months later, she complains of a relapse with severe pelvic pain, dysmenorrhoea and deep dyspareunia

Your answer: A Correct answer: A

Your answer: A Correct answer: C

| EXPLANATION | Options for Questions 19-20 A Tranexamic acid

B Danazol

C GnRH analogues

D TAH + BSO

E Diagnostic laparoscopy

F Mefenamic acid

G Pelvic ultrasound scan

H MRI scan of the pelvis J

Endocervical and urethral swabs

K Combined oral contraceptive pill

I

L

Levonorgestrel releasing intra-uterine system

M Refer to pain clinic

N Refer to psychiatrist

Dilatation and curettage

Instructions: For each of the case histories described below, choose the single most appropriate management from the above list. Each option may be used once, more than once, or not at all.

Question 19

Question 20

A 37 year old woman complains of severe pain during menstruation over the last 12 months. She has a regular 28 day cycle and denies any pelvic or abdominal pain at other times. She has no bowel symptoms. She had been taking the combined oral contraceptive pill until 12 months ago when it was discontinued because she is a smoker. Clinical examination is normal. A 38 year old woman complains of a 12 months history of severe pelvic pain, deep pain during sexual intercourse and pain on defecation. Her symptoms are particularly worse during menstruation but are present at other times and have not responded to simple analgesia. Clinical examination showed a

Your answer: A Correct answer: F

Your answer: A Correct answer: E

markedly tender pelvis with a fixed retroverted uterus but no adnexal masses.

| EXPLANATION | Options for Questions 21-22 A Counsel and offer termination of pregnancy

B Counsel and offer rubella immune globulin

C Treat with intravenous zidovudine

D Counsel and offer intra-uterine therapy

E Offer amniocentesis for karyotype

F Reassurance

G Immunise neonate against rubella

H Immunise mother against rubella

I

J

Treat with intravenous acyclovir

Amniocentesis and viral culture

K Amniocentesis and PCR for fetal viraemia

Instructions: For each scenario described below, choose the single most appropriate management from the above list of options. Each option may be used once, more than once, or not at all. Question 21

Question 22

A 30 year old woman is known to be rubella immune but is concerned about taking her 3 day old baby home because her 10 year old step-daughter has rubella A 20 year old woman has returned from holiday with a rash which is subsequently diagnosed as rubella. Following resolution of her symptoms, she complains of missing a period 2 weeks later and is found to be 11 weeks pregnant on ultrasound scan

| EXPLANATION |

Your answer: A Correct answer: F

Your answer: A Correct answer: A

Options for Questions 23-24 A Administer VZIG as soon as possible to mother

B Administer VZIG to mother if maternal serology ve

C Administer VZIG to neonate

D Detailed ultrasound examination

E Immediate caesarean section and transfer baby to the neonatal unit G Induction of labour

F Advise avoid contact with other pregnant women and neonates H Reassurance

I

Separate mother and baby after delivery

K Give intravenous Aciclovir

J

Serum for VZV IgM antibodies

L

Treat with oral Aciclovir

Instructions: For each scenario described below, choose the single most appropriate management from the above list of options. Each option may be used once, more than once, or not at all. Question 23

Question 24

A 35 year old woman was administered VZIG three weeks ago because she had close contact with a child with chickenpox at 12 weeks gestation and was found to be susceptible on serological testing A 20 year old woman has just returned from holiday and is surprised to find that she is 10 weeks pregnant based on ultrasound scan. She reports having been in contact with a child with chickenpox while on holiday 3 weeks earlier and serological tests show that she is varicella zoster IgM negative but IgG positive

Your answer: A Correct answer: J

Your answer: A Correct answer: H

| EXPLANATION | Options for Questions 25-26 A Avoid breast-feeding C

Administer VZIG to neonate but breast-feeding encouraged

B Administer VZIG to neonate and avoid breastfeeding D Immediate induction of labour

E Delay delivery with tocolytics

F Offer termination of pregnancy

G Treat neonate with oral acyclovir

H Treat neonate with intravenous acyclovir

I

Examine neonate for evidence of congenital anomalies

K Delay delivery for 5-7 days if possible

J

Reassurance

L

Detailed ultrasound examination

Instructions: For each scenario described below, choose the single most appropriate management from the above list of options. Each option may be used once, more than once, or not at all.

Question 25

Question 26

A 20 year old woman is in hospital 2 days after vaginal delivery of her first baby and is informed that her 8 year old step-daughter has chicken-pox. She does not remember having chicken pox in the past and is worried about taking her new baby home. Investigations show that she is varicella zoster IgG positive and IgM negative A 20 year old woman is being treated in the infectious diseases unit for chicken pox and reports having missed a period with a positive pregnancy test. Dating scan shows that she has a viable 11 week pregnancy and she is discharged following resolution of her symptoms

Your answer: A Correct answer: J

Your answer: A Correct answer: L

| EXPLANATION | Options for Questions 27-28 A Endometriosis

B Acute PID

C Chronic PID

D Irritable bowel syndrome

E Residual ovary syndrome

F

G Diverticular disease

H Interstitial cystitis

I

Ovarian remnant syndrome

J

Adenomyosis

K Uterine fibroids

L

Polycystic ovary syndrome

M Inflammatory bowel disease

N Chronic constipation

Primary dysmenorrhoea

Instructions: For each of the case histories described below, choose the single most likely cause of chronic pelvic pain from the above list. Each option may be used once, more than once, or not at all.

Question 27

A 34 year old woman with one previous ectopic pregnancy complains of a 3 year history of progressive pelvic and lower abdominal pain, dysmenorrhoea and deep dyspareunia. She has no bowel symptoms but has two previous laparoscopies for drainage of tubo-ovarian abscesses. Endocervical swabs for Chlamydia are negative and her white cell count and CRP are within normal limits. Pelvic ultrasound scan shows a normal size anteverted uterus with normal ovaries and a small amount of free fluid in the pouch of Douglas.

Your answer: A Correct answer: C

A 19 year old woman complains of progressively painful periods since menarche with occasional deep dyspareunia. She normally Question 28

opens her bowels 2-3 times a week and denies any bowel symptoms. She is sexually active and has one previous surgical termination of an unwanted pregnancy at 11 weeks gestation at the age of 17 years. Her symptoms have not responded to simple analgesia.

Your answer: A Correct answer: I

| EXPLANATION | Options for Questions 29-30 A Maternal treatment with antibiotics

B Detailed fetal anomaly scan

C Deliver by caesarean section

D Active immunisation of neonate

E Avoid breast-feeding

F

G Delay delivery by 5-7 days

H Administer corticosteroids

I

J

Administer tocolytics

Counsel and offer termination of pregnancy Reassurance

K Screen for maternal syphilis infection

Instructions: For each scenario described below, choose the single most appropriate management from the above list of options. Each option may be used once, more than once, or not at all. Question 29

Question 30

A 32 year old woman had a diagnosis of secondary syphilis at 16 weeks gestation which was successfully treated. She presents in spontaneous labour at 38 weeks gestation. The cervix is 2cm dilated with intact membranes. A 30 year old woman who has not booked for antenatal care presents with fetal death in-utero. She has a vaginal delivery of a fetus weighing 2700g. Post-mortem shows features consistent with congenital syphilis.

Your answer: A Correct answer: J

Your answer: A Correct answer: K

| EXPLANATION | Options for Questions 31-31 A Refer to surgical team

B Perform adhesiolysis and ovarian cystectomy

C GnRH analogues

D TAH + BSO

E Perform laser ablation of endometriosis

F Perform diathermy ablation of endometriosis

G Arrange clinic appointment to discuss further treatment

H MRI scan of the pelvis

I

Dilatation and curettage

J

Endocervical and urethral swabs Insert Levonorgestrel releasing intra-uterine system

K Commence combined oral contraceptive pill

L

M Refer to pain clinic

N Refer to gastroenterologist

Instructions: For each of the case histories described below, choose the single most appropriate management from the above list. Each option may be used once, more than once, or not at all.

Question 31

A 23 year old woman with a 3 year history of pelvic pain and deep dyspareunia has been consented for a diagnostic laparoscopy because her symptoms have not responded to medical treatment. At laparoscopy, she is found to have deposits of endometriosis in the ovarian fissae, utero-sacral ligaments and bilateral 3-4cm ovarian endometriomas with adhesions.

Your answer: A Correct answer: G

| EXPLANATION | Options for Questions 32-33 A Genital swabs for viral culture

B Immediate induction of labour

C Delay delivery by 5-7 days if possible

D Offer prophylactic oral acyclovir

E Treat with intra-venous acyclovir

F

G Treat neonate with intravenous acyclovir

H Reassurance

I

Avoid breast-feeding

K Refer to genito-urinary medicine clinic

Deliver by caesarean section

J

Avoid contact with other pregnant women and neonates

L

Screen for other sexually transmitted infections

Instructions: For each scenario described below, choose the single most appropriate management from the above list of options. Each option may be used once, more than once, or not at all.

Question 32

A 17 year old woman complains of generalised abdominal pain and vulval soreness at 38 weeks gestation. On examination, she is in urinary retention with florid genital herpetic vesicles. There are uterine contractions 3:10isand the cervixhistory is 2cmofdilated bulging membranes. There no previous genitalwith herpes.

Question 33

A 30 year old woman with previous genital herpes presents in spontaneous labour at 39 weeks gestation. There are no recurrent lesions and no prodromal symptoms. She is worried about her baby contracting the infection

| EXPLANATION | Options for Questions 34-34

Your answer: A Correct answer: F

Your answer: A Correct answer: H

A Endometriosis

B Acute PID

C Chronic PID

D Irritable bowel syndrome

E Residual ovary syndrome

F

G Diverticular disease

H Interstitial cystitis

I

Ovarian remnant syndrome

J

Adenomyosis

K Uterine fibroids

L

Polycystic ovary syndrome

M Inflammatory bowel disease

N Chronic constipation

Primary dysmenorrhoea

Instructions: For each of the case histories described below, choose the single most likely cause of chronic pelvic pain from the above list. Each option may be used once, more than once, or not at all.

Question 34

A 77 year old woman presents with a 5 year history of left iliac fossa pain and deep dyspareunia. Her LMP was 30 years ago and there has not been any post menopausal bleeding. She has a long history of constipation but her bowel habit has been regular in recent years with the use of laxatives. There is no significant past medical history and clinical examination is normal. Her white cell count, CRP and ESR are elevated and pelvic ultrasound scan shows a normal size anteverted uterus with normal ovaries and two small (2-3cm) complex cystic lesions posterior to the uterus slightly to the left of the midline.

Your answer: A Correct answer: G

| EXPLANATION | Options for Questions 35-36 A Administer VZIG as soon as possible to mother

Administer VZIG to mother if maternal serology B ve

C Administer VZIG to neonate

D Detailed ultrasound examination

E Immediate caesarean section and transfer baby to the neonatal unit

F Advise avoid contact with other pregnant women and neonates

G Induction of labour

H Reassurance

I

Separate mother and baby after delivery

K Give intravenous Aciclovir

J

Serum for VZV IgM antibodies

L

Treat with oral Aciclovir

Instructions: For each scenario described below, choose the single most appropriate management from the above list of options. Each option may be used once, more than once, or not at all. Question 35

Question 36

A 26-year-old Para 1+ 0 at 38 weeks gestation contacts her GP immediately after hearing that a child in her son's nursery has developed chickenpox. She has no memory of having the disease herself. Mrs Jones is seen in the antenatal clinic at 40 weeks. She has a cough and smokes 20 cigarettes per day. She has a rash and feels generally unwell. Her sister's child has developed chickenpox. They spent a weekend together two weeks ago. She does not think she has ever had chickenpox. Serological investigation shows that she is susceptible to varicella zoster .

| EXPLANATION | Options for Questions 37-38 A Genital swabs for viral culture

B Immediate induction of labour

Your answer: A Correct answer: B

Your answer: A Correct answer: K

C Delay delivery by 5-7 days if possible

D Offer prophylactic oral acyclovir

E Treat with intra-venous acyclovir

F

G Treat neonate with intravenous acyclovir

H Reassurance

I

Avoid breast-feeding

K Refer to genito-urinary medicine clinic

Deliver by caesarean section

J

Avoid contact with other pregnant women and neonates

L

Screen for other sexually transmitted infections

Instructions: For each scenario described below, choose the single most appropriate management from the above list of options. Each option may be used once, more than once, or not at all. Question 37

Question 38

A 17 year old woman complains of vulval soreness at 18 weeks gestation. She is found to have vulval herpetic vesicles on examination. There is no history of previous genital herpes.

A 20 year old woman presents in spontaneous labour at 37 weeks gestation. She developed primary genital herpes at 34 weeks gestation but is now symptom-free.

Your answer: A Correct answer: K Your answer: A Correct answer: F

| EXPLANATION | Options for Questions 39-40 A

Offer hepatitis B passive immunisation to woman

B Offer hepatitis B active immunisation to woman

C Deliver by caesarean section

D Amniocentesis for fetal viraemia by PCR

E Fetal blood sampling for viral culture

F

G Offer hepatitis B active immunisation to neonate

day 7 H Offer hepatitis B active & passive immunisation to neonate

I

Avoid breast-feeding

K Reassurance

Offer hepatitis B passive immunisation to neonate from

J

Intra-venous acyclovir in labour

L

Screen for hepatitis B infection

Instructions: For each scenario described below, choose the single most appropriate management from the above list of options. Each option may be used once, more than once, or not at all. Question 39

Question 40

A 25 year old nurse has been screened for hepatitis B infection at 15 weeks gestation with the following results: Hep B surface antigen negative, Hep B e antigen negative, antibodies to Hep B surface antigen positive A 20 year old woman has been screened for hepatitis B infection at 15 weeks gestation with the following results: Hep B surface antigen positive, Hep B e antigen positive, antibodies to Hep B surface antigen positive

Welcome to Busy SpR - Your professional on-line learning experience for Obstetricians and Gynaecologists.

MRCOG I PASS-RATE ANALYSIS

Your answer: A Correct answer: K

Your answer: A Correct answer: H

Click here to signout

• Possible metastatic breast cancer – peritoneal fluid for cytology would provide tissue diagnosis • Tissue diagnosis required and best obtained by biopsy of cervical lesion

Copyright © Busy SpR

Your score is 8% for this EMQ Exam You answered 3 out of 40 questions correctly. 40 random questions for EMQ Test

| EXPLANATION | Options for Questions 1-2 A Routine recall in 3 years

B Routine recall in 5 years

C Hysteroscopy + D&C

D Laser vapourisation

E Refer for colposcopy

F Cold knife cone biopsy

G Radical trachylectomy

H Cryotherapy

I

J

No further cervical smears required

Repeat cervical smear in 6 months

Instructions:For each scenario described below, choose the singlemost appropriate management from the above list of options. Each option may be used once, more than once, or not at all.

Question 1

A 40 year old nulliparous woman had a cervical smear showing severe dyskaryosis. Colposcopy with diathermy loop excision has been performed and the histology shows CIN III which has been completely excised

Question 2

A 53 year old asymptomatic post menopausal woman has a cervical smear showing no dyskaryotic cells but no evidence of transformation zone sampling. The cervix was well visualised and normal and she has previously had regular smears which have all been negative.

Your answer: A Correct answer: J

Your answer: A Correct answer: B

| EXPLANATION | Options for Questions 3-4 A Induction of labour

B Discharge home

C Community midwife follow-up

D Anti-hypertensive therapy

E Caesarean section at 39 weeks F Manage as out-patient with day assessment unit G External cephalic version

H Induction of labour at 40 weeks

I

J

Induction of labour at 42 weeks

Measure 24h urine protein excretion

Instructions:For each patient described below choose the singlemost appropriate management option from the list. Each option may be used once, more than once, or not at all. Question 3

A 35 year old primigravida is an in-patient because of hypertension and 1+ proteinuria on stix testing at 37

Your answer: A

weeks gestation. Her BP is 130/80mmHg on oral methyldopa and all her blood tests are within normal limits. 24h urine protein excretion is 200mg/24h.

Question 4

A 20 year old woman is an in-patient because of proteinuric hypertension at 37 weeks gestation. Her BP is 140/85mmHg on oral methyldopa and all her blood tests are within normal limits. 24h urine protein excretion is 800mg. Fetal growth scan shows the head and abdominal circumferences on the 50th centile with normal liquor volume.

Correct answer: F

Your answer: A Correct answer: A

| EXPLANATION | Options for Questions 5-6 A Routine recall in 3 years

B Routine recall in 5 years

C Hysteroscopy + D&C

D Laser vapourisation

E Refer for colposcopy

F Cold knife cone biopsy

G Radical trachylectomy

H Cryotherapy

I

J

No further cervical smears required

Repeat cervical smear in 6 months

Instructions:For each scenario described below, choose the singlemost appropriate management from the above list of options. Each option may be used once, more than once, or not at all. Your answer: A Question 5

A 35 year old nulliparous woman has a cervical smear showing severe dyskaryosis

Question 6

A 68 year old woman attends the gynaecology clinic Your answer: A because she is concerned that she has not been called for a cervical smear. She has always attended for 3 yearly smears over the previous 15 years which Correct answer: I have all been negative and she is asymptomatic

Correct answer: E

| EXPLANATION | Options for Questions 7-8 A Increase iv fluids

B Insert central venous pressure line

C Intravenous magnesium sulphate

D

E Measure serum magnesium

transaminase immediately F Blood transfusion

G Monitor patellar reflex every 15 minutes

H Provide a fluid challenge with colloids

I

Provide intravenous Hartmann

K Immediate dose of 10ml 10% calcium gluconate intravenously

Measure serum aspartate

J

Decrease iv fluids

L

Carry out visual field assessment

Instructions:For each patient described below choose the singlemost appropriate management option from the list. Each option may be used once, more than once, or not at all. Question 7

A 20-year-old primigravida is 30 weeks' pregnant and

Your answer: A

has been transferred to the delivery suite with severe gestational proteinuric hypertension. She complains of severe frontal headache but has no other symptoms. She has a normal respiratory rate and her urine output has been satisfactory. Her blood pressure is 140/100mmHg. There are five beats of bilateral ankle clonus.

Correct answer: C

A 20-year-old primigravida delivered a live infant 5 hours previously by emergency caesarean section. She has developed severe gestational proteinuric Your answer: A Question 8

hypertension. Bloodml. loss section estimated at 1,000 BPat= caesarean 160/100mmHg on was oral anti-hypertensive treatment and pulse = 95bpm. She has been given one litre of Hartmann's solution intravenously since her delivery but has passed 40ml of urine since delivery. Her clotting, liver enzymes and serum creatinine are normal.

Correct answer: B

| EXPLANATION | Options for Questions 9-10 A Routine recall in 3 years

B Routine recall in 5 years

C Hysteroscopy + D&C

D Laser vapourisation

E Refer for colposcopy

F

G Repeat cervical smear as soon as possible

H Repeat cervical & endocervical smear in 6 months

I

Wertheim’s hysterectomy + lymphadenectomy

J

Cold knife cone biopsy

Repeat cervical smear in 6 months

Instructions:For each scenario described below, choose the singlemost appropriate management from the above list of options. Each option may be used once, more than once, or not at all.

Question 9

A 32 year old woman has a cervical smear showing atypical glandular cells. She has colposcopy and a cone biopsy and the histology shows cervical glandular intra-epithelial neoplasia which has been completely excised. She has not completed her family.

A 35 year old woman has a cervical smear showing Question 10 atypical glandular cells. Cone biopsy shows microinvasive cervical adenocarcinoma.

Your answer: A Correct answer: H

Your answer: A Correct answer: I

| EXPLANATION | Options for Questions 11-12 A Antihypertensive treatment

B Insert central venous pressure line

C Intravenous magnesium sulphate E Measure serum magnesium

D Measure serum aspartate transaminase immediately F Transfer to intensive treatment unit

G Monitor patellar reflex every 15 minutes

H

I

J

Provide intravenous Hartmann's solution at

Provide a fluid challenge with colloids Calculate the mean arterial blood

the rate of 85ml per hour K

pressure

Immediate dose of 10ml 10% calcium gluconate intravenously

L

Carry out visual field assessment

Instructions:For each patient described below choose the singlemost appropriate initial treatment option from the list. Each option may be used once, more than once, or not at all. A 20-year-old primigravida had a normal delivery of a live infant 12 hours previously. She has developed severe gestational proteinuric hypertension, her Question 11

clotting is normal, serum albumin is 43g/dl, there is no ankle clonus and her blood pressure is 160/100mmHg. She has been given one litre of Hartmann's solution intravenously since her delivery and has been anuric. The central venous pressure is +10mmHg, serum sodium 132mmol/l, serum potassium 7.1mmol/l and serum urea 22mmol/l.

A 20-year-old primigravida delivered a live infant 24 hours previously. She has developed severe gestational proteinuric hypertension. Treatment with Question 12 intravenous magnesium was required. Her fluid balance is satisfactory and serum urea, electrolytes and clotting profile are all normal. Her respiratory rate falls to 6 per minute and she is drowsy but rousable.

Your answer: A Correct answer: F

Your answer: A Correct answer: K

| EXPLANATION | Options for Questions 13-13 A Bilateral oophrectomy

B CA-125 assays every 6-12 months

C TAH + BSO

D Cyst aspiration for cytology

E Refer to cancer centre

F Ovarian cystectomy

G Reassure

H Measure serum CA-125

I

Refer to palliative care team

K Chemotherapy

J

TAH + BSO + Omentectomy

L

Radiotherapy

Instructions:For each scenario described below, choose the singlemost appropriate management from the above list of options. Each option may be used once, more than once, or not at all. A 40 year old nulliparous woman has had a laparotomy for suspected ovarian cancer. The findings were of disseminated malignancy with

Your answer: A

Question 13 tumour involving surface of the liver and bowel with a both largeovaries, omentalthe cake. De-bulking was Correct answer: K not possible and tissue for histology shows a poorly differentiated serous adenocarcinoma of the ovary.

| EXPLANATION | Options for Questions 14-15 A Administer iv frusemide C

Intravenous magnesium sulphate

B Immediate delivery by caesarean section D

Measure serum aspartate transaminase immediately

E Measure serum magnesium

F Blood transfusion

G Immediate induction of labour

H Provide a fluid challenge with colloids

I

Provide intravenous Hartmann

K

Measure 24h urine protein excretion

J

Decrease iv fluids

L

Carry out visual field assessment

Instructions:For each patient described below choose the singlemost appropriate management option from the list. Each option may be used once, more than once, or not at all. A 20-year-old primigravida is 35 weeks' pregnant and has been transferred to the delivery suite with severe Your answer: A gestational proteinuric hypertension. She complains Question 14 of severe epigastric pain with nausea and vomiting. Correct answer: Her respiratory rate and her urine output are normal. D Her blood pressure is 140/90mmHg on treatment with methyldopa An 18-year-old primigravida is 36 weeks' pregnant and has been referred to the delivery by the community midwife because of a 3 day history of feeling generally unwell. Her BP on admission is Question 15 145/95mmHg with 3+ proteinuria. The uterine size is appropriate for gestation age with a cephalic presentation. Her serum aspartate transaminase concentration is 245 iu/l but all other investigations are within normal limits.

Your answer: A Correct answer: G

| EXPLANATION | Options for Questions 16-17 A 85% 5 year survival

B 70% 5 year survival

C 60% 5 year survival

D 40% 5 year survival

E 25% 5 year survival

F

G Almost 100% 5 year survival

H Overall life-time risk ~1%

I

Virtually 0% 5 year survival

Overall life-time risk ~2%

J

Overall life-time risk _1.5%

K Overall life-time risk ~3%

L

Overall life-time risk of 0.1%

Instructions:For each scenario described below, choose the singlemost appropriate information that should be given to the woman from the above list of options. Each option may be used once, more than once, or not at all. A 68 year old woman attends the gynaecology clinic Question 16

4 weeks after TAH + BSO for endometrial carcinoma. Your answer: A The histology shows that the tumour had extended to involve the cervix and she has been advised to have Correct answer: radiotherapy. She enquires about the survival rate for B women with similar tumours.

A 32 year old teacher has been referred to the gynaecology clinic because of marked anxiety since Your answer: A her friend died suddenly from ovarian cancer at the Question 17 age of 35 years. She has no family history of ovarian Correct answer: I cancer and enquires about the likelihood of any woman developing the disease.

| EXPLANATION | Options for Questions 18-19 A Oestrogen cream

B Testosterone cream

C Oral HRT

D Topical anti-histamines

E Vaginoplasty

F

G Skinning vulvectomy

H Topical anti-fungal cream

I

J

Clobetasol cream

Radical vulvectomy Excision biopsy

Instructions:For each clinical scenario below choose the singlemost appropriate treatment from the list above. Each diagnosis may be used once, more than once, or not at all. A 23-year-old woman presents with a two-year history of vulval, perineal and perianal irritation. The Question 18 vulva is red, excoriated and there areas of white, thickened skin. Application of 3% Acetic acid shows areas of mosaic and coarse punctuation.

Your answer: A Correct answer: J

A 47 year old woman complains of a 3 months history of vulval irritation and superficial dyapareunia. Your answer: A Question 19 The vulval skin is thin and white with fissures and narrowing of the introitus and fusion of the labia Correct answer: I minora over the clitoris

| EXPLANATION | Options for Questions 20-21 A Bilateral oophrectomy

B CA-125 assays every 6-12 months

C TAH + BSO

D Cyst aspiration for cytology

E Refer to cancer centre

F Ovarian cystectomy

G Reassure

H Measure serum CA-125

I

Refer to palliative care team

K Chemotherapy

J

TAH + BSO + Omentectomy

L

Radiotherapy

Instructions:For each scenario described below, choose the singlemost appropriate management from the above list of options. Each option may be used once, more than once, or not at all. A healthy 20 year old woman has been referred by the on-call surgeon following admission for suspected appendicitis. Her LMP was 12 days earlier Question 20 and a pelvic ultrasound scan shows a 2cm simple cyst on her right ovary with a small amount of free fluid in the pouch of Douglas. She has a regular 28 day cycle and her pregnancy test is negative. A 56 year old woman has been admitted for laparotomy for suspected ovarian cancer. Laparotomy shows a 10cm mobile complex left Question 21 ovarian mass with a small amount of free fluid and a normal right ovary. The bowel, omentum, surface of the liver and diaphragmatic peritoneum are normal. There is no obvious lymphadenopathy.

Your answer: A Correct answer: G

Your answer: A Correct answer: J

| EXPLANATION | Options for Questions 22-23 A Bilateral oophrectomy

B CA-125 assays every 6-12 months

C TAH + BSO

D Cyst aspiration for cytology

E Refer to cancer centre

F Ovarian cystectomy

G Reassure

H Measure serum CA-125

I Refer to palliative care team K Chemotherapy

J L

Paracentesis for cytology Radiotherapy

Instructions:For each scenario described below, choose the singlemost appropriate management from the above list of options. Each option may be used once, more than once, or not at all. A 35 year old nulliparous woman has had an oophrectomy after emergency admission with a torted left ovarian mass. The histology report shows Question 22 a stage Ia serous cystadenocarcinoma. Her CA-125 on admission was 650iu and she is keen to maintain her reproductive potential A 35 year mother of 3 children is referred to the gynaecology clinic because she has been found to be a carrier of the BRCA-1 mutation following breast Question 23 cancer in her sister. She has completed her family

Your answer: A Correct answer: B

Your answer: A Correct answer:

and would like the most effective way of minimising the risk of ovarian cancer

A

| EXPLANATION | Options for Questions 24-25 A Intravenous labetalol

B Immediate delivery by caesarean section

C Intravenous magnesium sulphate D

Measure serum aspartate transaminase immediately

E Measure serum magnesium

F

G Immediate induction of labour

H Provide a fluid challenge with colloids

I

Antihypertensive treatment

J

Administer iv phenytoin

Measure 24h urine protein K excretion

L

Place in left lateral position

Blood transfusion

Instructions:For each patient described below choose the singlemost appropriate management option from the list. Each option may be used once, more than once, or not at all. A 20-year-old primigravida has an emergency caesarean section at 30 weeks gestation because of severe gestational proteinuric hypertension and IUGR. Her FBC, LFTs and U&E pre-op were within normal limits. 12 hours after delivery, investigations Question 24 show that her serum lactate dehydrogenase concentration is markedly elevated, serum aspartate transaminase concentration is 224 iu/l, platelet count is 80 x 10E9 /l and her renal function is normal. Her BP is 140/90 on oral anti-hypertensive therapy and

Your answer: A Correct answer: C

her urine out-put is 80ml/h. An asymptomatic 40 year old primigravida attends the antenatal clinic at 35 weeks gestation with a previously uncomplicated pregnancy. Her BP is Question 25 170/95 on two occasions 15 minutes apart and there is no proteinuria. The uterine size is appropriate for gestation age with a cephalic presentation.

Your answer: A Correct answer: I

| EXPLANATION | Options for Questions 26-27 A Bilateral oophrectomy

B CA-125 assays every 6-12 months

C TAH + BSO

D Cyst aspiration for cytology

E Refer to cancer centre

F

G Reassure

H Measure serum CA-125

I

J

Yearly pelvic ultrasound scans

Refer to palliative care team Repeat pelvic ultrasound scan in 4-6 months

Instructions:For each scenario described below, choose the singlemost appropriate management from the above list of options. Each option may be used once, more than once, or not at all. A 40 year old woman presents with a 12 months history of vague abdominal discomfort that did not Question 26 respond to simple analgesia. Ultrasound scan shows bilateral complex ovarian cysts with right sided hydronephrosis and ascites. Her CA-125 is 1500iu A 20 year old woman is admitted with sudden onset left sided pelvic pain 23 days after her LMP. Her pregnancy test is negative and her symptoms are Question 27 improving with simple analgesia. Pelvic ultrasound scan shows a 5cm left ovarian cyst with internal echoes consistent with a haemorrhagic cyst.

Your answer: A Correct answer: E

Your answer: A Correct answer: J

| EXPLANATION | Options for Questions 28-29 A Intravenous labetalol

B Immediate delivery by caesarean section

C Intravenous magnesium sulphate D

Measure serum aspartate transaminase immediately

E F Measure FBC and clotting profile Blood transfusion G Immediate induction of labour H Provide a fluid challenge with colloids I

Antihypertensive treatment

J

Administer iv phenytoin

K Measure 24h urine protein excretion

L

Arrange in-utero transfer to tertiary centre

Instructions:For each patient described below choose the singlemost appropriate management option from the list. Each option may be used once, more than once, or not at all. A 30 year old primigravida attends as an emergency Question 28 at 32 weeks gestation with a twin pregnancy and complains of 12h of severe vomiting and upper

Your answer: A Correct answer:

abdominal pain. Her BP is 130/80mmHg with a trace of proteinuria. A growth scan one week earlier had shown normal fetal growth. Investigations show a serum aspartate transaminase concentration of 600iu/l, alkaline phosphatase = 1550iu/l, serum uric acid = 0.57mmol/l and serum creatinine = 130micromol/l. FBC and coagulation profile are normal.

B

A 32 year old primigravida with a previously normal pregnancy is seen in the antenatal clinic at 39 weeks

Your answer: A

Question 29 gestation with a BP of 160/95mmHg 3+ proteinuria. The uterus is appropriateand for gestation age with a cephalic presentation.

Correct answer: G

| EXPLANATION | Options for Questions 30-31 A Atrophic vulvovaginitis

B Human papilloma virus infection

C Benign mucous membrane pemphigoid

D Candida infection

E Lichen sclerosus

F Contact dermatitis

G Lichen simplex et chronicus

H Herpes simplex infection

I

J

Vulval intraepithelial neoplasia

Lichen planus

Instructions:For each clinical scenario below choose the singlemost likely diagnosis from the list above. Each diagnosis may be used once, more than once, or not at all. A 47 year old woman complains of a 3 months Your answer: A history of vulval irritation and superficial dyapareunia. Question 30 The vulval skin is thin and white with fissures and Correct answer: narrowing of the introitus and fusion of the labia E minora over the clitoris A 42 year old woman with ulcerative colitis complains of a 6 months history of vulval irritation and a burning Your answer: A sensation during intercourse. Examination showed Question 31 popular eruptions on the flexor surface of her upper Correct answer: limbs, vulva and vagina with ulcerative lesions in the J mouth.

| EXPLANATION | Options for Questions 32-33 A Platelet transfusion

B Insert central venous pressure line

C Intravenous magnesium sulphate

D Continue iv fluids at current rate

E Measure FBC and clotting profile

F Blood transfusion

G Increase iv fluids

H Provide a fluid challenge with colloids

I

Antihypertensive treatment

K Administer iv frusemide

J

Measure 24h urine protein excretion

L

Administer iv albumin

Instructions:For each patient described below choose the singlemost appropriate management option from the list. Each option may be used once, more than once, or not at all.

A 32-year-old primigravida had a normal delivery of a live infant 10 hours previously. She has developed severe gestational proteinuric hypertension, her clotting and FBC are normal, serum albumin is 27g/dl serum sodium 132mmol/l, serum potassium Question 32 7.1mmol/l and serum urea 22mmol/l. Her blood pressure is 160/90mmHg, she has passed 50ml of urine and been given 1000ml of Hartmann's solution since delivery. She complains of being short of breadth and her oxygen saturation is 93% on 3l/min oxygen. A 20-year-old primigravida is 32 weeks pregnant and has been transferred to the delivery suite for emergency caesarean section because of gestational proteinuric hypertension and IUGR. Her BP is Question 33 160/100 on oral methyldopa. Her platelet count is 42 x10E9/l, Hb 13.5g/dl, AST 57iu/l and her clotting and renal function are normal. She has no symptoms and her urine out-put is satisfactory.

Your answer: A Correct answer: B

Your answer: A Correct answer: A

| EXPLANATION | Options for Questions 34-35 A Platelet transfusion

B Administer fresh frozen plasma

C Intravenous magnesium sulphate

D Continue iv fluids at current rate

E Measure FBC and clotting profile

F Blood transfusion

G Increase iv fluids I Antihypertensive treatment

H Provide a fluid challenge with colloids J Measure 24h urine protein excretion

K Administer iv frusemide

L

Decrease iv fluids

Instructions:For each patient described below choose the singlemost appropriate management option from the list. Each option may be used once, more than once, or not at all. A 40-year-old primigravida had an emergency caesarean section 10 hours previously because of severe gestational proteinuric hypertension at 33 weeks gestation. Estimated blood loss was 500ml. Your answer: A She has marked peripheral oedema and her clotting and FBC are normal. Serum albumin is 35g/dl serum Question 34 sodium 132mmol/l, serum potassium 7.1mmol/l and Correct answer: serum urea 22mmol/l. Her blood pressure is L 160/90mmHg, pulse 78bpm. She has passed 50ml of urine and been given 1000ml of Hartmann's solution since delivery. She has also taken 250ml of fluid orally. A 20-year-old primigravida has an emergency caesarean section at 34 weeks gestation because of severe gestational proteinuric hypertension and IUGR. The estimated blood loss at caesarean section Your answer: A was 2000ml. Four hours post-op, her BP is 165/100, Question 35 pulse 110bpm and she has passed 20ml of urine Correct answer: since delivery. Her Hb taken 2h after delivery is F 9.0g/dl, and her platelet count and renal function are normal. She is asymptomatic and there is 20ml of fluid in the abdominal drain.

| EXPLANATION | Options for Questions 36-36 A TAH + BSO

B Vaginal hysterectomy

C Radical abdominal hysterectomy

D Laparoscopic assisted vaginal hysterectomy

E TAH + BSO + omentectomy

F BSO

G TAH

H Chemotherapy

I

J

Radiotherapy

Combined chemo-radiotherapy

K Endometrial biopsy Instructions:For each scenario described below, choose the singlemost appropriate management from the above list of options. Each option may be used once, more than once, or not at all. A 65 years old woman complains of profuse postcoital bleeding 3 years after TAH + BSO for a stage Ib endometrial carcinoma. Clinical examination showed a polypoid lesion at the vaginal vault with Question 36 contact bleeding and biopsy confirmed recurrent endometrial adenocarcinoma. MRI scan of the pelvis and abdomen identified a 3cm lesion at the vault but no other abnormalities.

Your answer: A Correct answer: I

| EXPLANATION | Options for Questions 37-38 A Administer iv frusemide

B Immediate delivery by caesarean section

Intravenous magnesium C sulphate

D

Measure serum aspartate transaminase immediately

E Measure serum magnesium

F Blood transfusion

G Immediate induction of labour

H Provide a fluid challenge with colloids

I

J

Administer iv phenytoin

L

Place in left lateral position

Provide intravenous Hartmann

K Measure 24h urine protein excretion

Instructions:For each patient described below choose the singlemost appropriate management option from the list. Each option may be used once, more than once, or not at all. A 20-year-old primigravida is 30 weeks' pregnant and has been transferred to the delivery suite with severe Question 37

gestational proteinuric hypertension. Corticosteroids had been administered 3 days earlier and her BP is 170/115mmHg on oral methyldopa. Over the next 5 hours, she is treated with oral nifedipine and intravenous labetalol and her BP remains 170/110mmHg. She is asymptomatic.

Your answer: A Correct answer: B

You have been asked to attend the antenatal ward as Your answer: A an emergency because a 20 year old primigravida with severe gestational proteinuric hypertension at 32 Question 38 weeks gestation is having a seizure. On your arrival, Correct answer: the woman is actively fitting and there are two L midwives in attendance.

| EXPLANATION | Options for Questions 39-40 A Endocervical and urethral swabs for culture B EUA + cystoscopy C MRI scan

D Out-patient hysteroscopy and biopsy

E Hysteroscopy D&C

F

G Cervical smear

H Peritoneal fluid for cytology

I

J

Serum CA-125

Cervical biopsy Colposcopy

Instructions:For each scenario described below, choose the singlemost appropriate investigation from the above list of options. Each option may be used once, more than once, or not at all. A 48 year old woman with a history of breast cancer presents with a 6 months history of vague abdominal Question 39 discomfort. Clinical examination is suggestive of ascites which is confirmed on abdominal ultrasound scan. The ovaries appeared normal. A 73 year old woman complains of a 3 week history of persistent vaginal bleeding. Clinical examination Question 40 shows a florid 5cm lesion on the anterior lip of her cervix extending to the upper third of the vagina with contact bleeding.

Your answer: A Correct answer: H

Your answer: A Correct answer: F

Home | Signup | Subscribe| Contact us | Medical forums | Privacy | Legal Notices | Copyright © Busy SpR Busy SpR Ltd. is a Registered Company of the United Kingdom. Company Registration Number:4407908 Developed by Last Digital

Welcome to Busy SpR - Your professional on-line learning experience for Obstetricians and Gynaecologists. Thursday Apr 06th, 2006

Click here to signout

MRCOG I PASS-RATE ANALYSIS

PROBLEM BASED LEARNING ZONE

Gynaecology

Your score is 10% for this EMQ Exam

Obstetrics Surgical practice

You answered 4 out of 40 questions correctly.

PART II MRCOG e-COURSE

40 random questions for EMQ Test

Essays Notes MCQ

| EXPLANATION | Options for Questions 1-2

EMQ A Counsel and offer

B Counsel and offer detailed

PART I

termination of pregnancy

anomaly scan

ON-LINE TUTORIAL

C Reassure PAPER 1 E •



Anatomy & embryology

G Neonatal oral vitamin K

Immunology



Microbiology



Pharmacology



MOCK

PAPER 2 Physiology (free) •



Biochemistry



Genetics



Pathology



MOCK

Commence folic acid 0.4mg/day

F

Maternal vitamin K from 36 weeks

H Amniocentesis for karyotype

I

Discontinue anti-epileptic drug therapy

J

Monitor anti-epileptic drug levels every month

K

Advice against breastfeeding

L

Advice against pregnancy

Endocrinology



Commence folic acid 4mg/day

D

M Neonatal im vitamin K Instructions:For each scenario described below, choose the single most appropriate management from the above list of options. Each option may be used once, more than once, or not at all. A 30 year old epileptic has been seizure- Your answer: A free for 12 months on sodium valproate. Question 1 She has a positive pregnancy test following a missed period and is anxious Correct because she has not bee taking folic acid answer: B

A 30 year old epileptic has been seizurefree for 12 months on sodium valproate. Question 2 She is seen in the pre-conception clinic because she is planning a pregnancy.

Your answer: A Correct answer: E

| EXPLANATION | Options for Questions 3-4

A

FSH & LH concentration on days 2-5

C Day 21 progesterone E

Neonatal 17-hydroxy-progesterone concentration

B Serum prolactin D Random serum oestradiol F Pelvic ultrasound scan

G Serum androgen concentrations

H Progesterone challenge test

I

J

Dexamethasone suppression test

K Karyotype

L

MRI scan of abdomen and pelvis Random serum FSH concentration

Instructions:For each of the case histories described below, choose the singlemost appropriate investigation from the above list. Each option may be used once, more than once, or not at all. A 20 year old woman has been referred to the gynaecology clinic because she has not had any menstrual periods for over 2 years. She has no other Question 3 symptoms. On examination, her height is 1.6m and BMI is 22. There was slight webbing of the neck, widely spaced nipples and a wide carrying angle.

Your answer: A Correct answer: K

An 38 year old woman has just had a forceps delivery and the neonate has been found to have ambiguous external genitalia. Her medical records show that Question 4 she had amniocentesis at 18 weeks gestation because of a high risk of Down’s syndrome on serum screening and the fetus was found to have a 46XX karyotype.

Your answer: A Correct answer: E

| EXPLANATION | Options for Questions 5-6

B

Commence treatment with insulin

C Stop insulin

D

Commence insulin-glucose regimen

E Recommend delivery by caesarean section at 38 weeks

F Induce labour at 36 weeks gestation

G Induce labour at 39-40 weeks gestation

H Increase insulin dose

I

J

A

K

Commence treatment with metformin

Stop insulin-glucose regimen Counsel and offer termination of pregnancy

L

Re-introduce prepregnancy insulin regimen Reassure

M Repeat GTT 6-12 weeks postpartum Instructions:For each scenario described below, choose the single most appropriate management from the above list of options. Each option may be used once, more than once, or not at all. Your answer: A 35 year old woman with gestational A diabetes mellitus has been managed with Question 5 diet alone during pregnancy and has had a normal vaginal delivery at 39 weeks Correct with an insulin-glucose regimen. answer: I A 35 year old woman with gestational Your answer: diabetes has been treated with insulin A during the antenatal period. She has a Question 6 normal vaginal delivery at 38 weeks Correct gestation and is due for discharge 3 days answer: M post-partum | EXPLANATION | Options for Questions 7-8 A Commence treatment with metformin

B Commence treatment with insulin

C Stop insulin

D

Commence insulin-glucose regimen

F

Induce labour at 36 weeks gestation

E

Recommend delivery by caesarean section at 38 weeks

G Induce labour at 39-40 weeks gestation

H Increase insulin dose

I

J

K

Stop insulin-glucose regimen Counsel and offer termination of pregnancy

L

Re-introduce prepregnancy insulin regimen Reassure

M Repeat GTT 6-12 weeks postpartum Instructions:For each scenario described below, choose the single most appropriate management from the above list of options. Each option may be used once, more than once, or not at all. A 32 year old with IDDM attends the Your answer: antenatal clinic at 32 weeks gestation. A Her blood glucose concentration on home Question 7 monitoring over the preceding week have been as follows: Pre-breakfast: 9-12mM; Correct pre-lunch: 8-15mM, pre-supper: 9 answer: H 12mM; bedtime: 5-12mM A 35 year old woman with well controlled Your answer: gestational diabetes has a growth scan at A 38 weeks gestation showing fetal head Question 8 and abdominal circumferences on the Correct 50th centile. All other maternal and fetal answer: G observations are within normal limits.

| EXPLANATION | Options for Questions 9-10 A In-vitro fertilisation

B Intra-uterine insemination

C Laparoscopy and dye test

D Intra-cytoplasmic sperm injection

E Clomephene citrate

F

G Metformin

H Oocyte donation

I

J

Surrogacy

K Weight gain

L

Gonadotrophin induction of ovulation Weight reduction Measure serum androgen concentrations

Instructions:For each of the case histories described below, choose the singlemost appropriate initial management from the above list. Each option may be used once, more than once, or not at all. A 24 year old woman with her 35 year old partner has been referred to the infertility clinic because of 2 years of primary infertility. The woman has an irregular menstrual cycle every 2-4 months and Your answer: her partner’s semen analysis is normal. A The woman’s BMI is 26 and there is no Question 9 significant past medical history. Correct Investigations have confirmed a answer: C diagnosis of polycystic ovary. The couple have not achieved a pregnancy despite a 6 months course of clomephene citrate with evidence of ovulation based on day 21 progesterone concentration and

ultrasound scanning. A 34 year old woman with her 35 year old partner has been referred to the infertility clinic because of 2 years of primary infertility. The woman has a regular 28 day cycle and her periods are heavy with severe dysmenorrhoea. She also Your answer: complains of deep dyspareunia and A intermittent lower abdominal and pelvic Question 10 pain. Her BMI is 27. Investigations have Correct shown normal LH, Thyroid function answer: A tests, prolactin andFHS, day 21 progesterone concentration confirms ovulatory cycles. Laparoscopy and dye test shows evidence of chronic pelvic inflammatory disease with bilateral tubal occlusion. Her partner’s semen analysis is normal.

| EXPLANATION | Options for Questions 11-12

A

Stop heparin on the evening before induction

B Commence oral warfarin

C Await results of V/Q scan then D Await results of venogram commence treatment then commence treatment E Await results of D-dimers

F

assay Commence prophylactic dose G of heparin post-partum

of heparin H Oral aspirin therapy

Antenatal prophylactic treatment with heparin

J

K Prophylactic heparin for 6 weeks post-partum

L

I

Commence therapeutic dose

Antenatal treatment with warfarin Stop heparin therapy

Instructions:For each scenario described below, choose the single most appropriate management from the above list of options. Each option may be used once, more than once, or not at all. A 30 year old woman with a previous DVT presents for antenatal care at 15 weeks gestation. There is no family Question 11 history of VTE and her BMI is 25. She is otherwise fit and well with a negative thrombophilia screen A 30 year old woman with recurrent first trimester miscarriages is known to have the anti-phospholipid antibody syndrome Question 12and has had a spontaneous vaginal delivery at 39 weeks gestation. Her BMI is 27 and she is otherwise fit and well with no family history of VTE.

| EXPLANATION | Options for Questions 13-14

Your answer: A Correct answer: K

Your answer: A Correct answer: G

A Prolactinoma

B Pre-mature ovarian failure

C Asherman’s syndrome

D Ovarian hyper-stimulation syndrome

E Polycystic ovary syndrome

F

G Androgen secreting tumour

H Cervical stenosis

I

J

Hyperthyroidism

L

Pure gonadal dysgenesis

Hypothalamic amenorrhoea

Androgen insensitivity K syndrome

Hypothyroidism

Instructions: For each of theofcase histories described below,list. choose the singlemost likely cause amenorrhoea from the above Each option may be used once, more than once, or not at all. An 18 year virgin has been referred to the gynaecology clinic because she has not Your answer: started menstruating but she has no other A symptoms. Clinical examination showed Question 13 that her height was 1.67m with BMI of 23. Correct She had normally formed breasts but no answer: K axillary or pubic hair. There was a 2-3cm mass palpable in each inguinal canal. A 17 year old woman with primary Your answer: amenorrhoea has been investigated with A the following results: FSH = 9mIU/ml, LH Question 14 = 20mIU/ml, oestradiol = 200pg/ml, free Correct testosterone = 5.0ng/dl (1-4.3ng/dl). answer: K Karyotype is 46XY.

| EXPLANATION | Options for Questions 15-16 A Amniotic fluid embolism

B Placental abruption

C Cardiomyopathy

D Placenta praevia

E Chest infection

F

G CVA

H Pulmonary hypertension

I

Pulmonary embolism

J

Sepsis

K Haemorrhage

L

Substance misuse

M HELLP syndrome

N Thromboembolism

Endocarditis

O Myocardial infarcation Instructions:For each case described below, choose the singlemost likely cause of maternal death from the above list of options. Each option may be used once, more than once, or not at all. A 42 year old woman presents with a 30 minute history of constant abdominal pain Your answer: at 38 weeks gestation. Her blood A pressure is 110/70 with a pulse of 120 Question 15 bpm on admission. The fetal heart is not Correct detectable. She collapsed 15 minutes answer: B after admission and died despite intensive resuscitation A 32 year old woman with severe preQuestion 16 eclampsia has had an emergency

Your answer: A

caesarean section at 32 weeks gestation because of IUGR. She became hypotensive and tachycardic and was returned to theatre where 2l of blood was evacuated from the abdomen and hysterectomy was performed. Three days later, she developed sudden onset severe chest pain and breathlessness and dies despite resuscitation

Correct answer: N

| EXPLANATION | Options for Questions 17-18

A

Commence treatment with metformin

B

D Commence insulin-glucose regimen

C Stop insulin E

Commence treatment with insulin

Recommend delivery by caesarean section at 38 weeks

F

Induce labour at 36 weeks gestation

G Induce labour at 39-40 weeks gestation

H Increase insulin dose

I

Stop insulin-glucose regimen

J

K Counsel and offer termination of pregnancy

L

M

Re-introduce prepregnancy insulin regimen Reassure

Repeat GTT 6-12 weeks postpartum

Instructions:For each scenario described below, choose the single most appropriate management from the above list of options. Each option may be used once, more than once, or not at all.

A 20 year old insulin-dependent diabetic Question 17attends for induction of labour at 38 weeks gestation

A 32 year old with adequately controlled IDDM has a growth scan at 36 weeks gestation which shows that the fetal Question 18 abdominal circumference is well above the 97th centile with an estimated fetal weight of 4200g.

Your answer: A Correct answer: D

Your answer: A Correct answer: E

| EXPLANATION | Options for Questions 19-20

A

Stop heparin on the evening before induction

B Commence oral warfarin

C Await results of V/Q scan then D Await results of venogram commence treatment then commence treatment E Await results of D-dimers assay G Commence prophylactic dose

F

Commence therapeutic dose of heparin

H Oral aspirin therapy

of heparin post-partum I

Antenatal prophylactic treatment with heparin

J

K

Prophylactic heparin for 6 weeks post-partum

L

Antenatal treatment with warfarin Stop heparin therapy

Instructions:For each scenario described below, choose the single most appropriate management from the above list of options. Each option may be used once, more than once, or not at all. A 30 year old woman with a BMI 0f 25 has had a spontaneous vaginal delivery. Your answer: A She has a history of a previous DVT Question 19while on the COCP but the thrombophilia screen was negative and has been on s/c Correct heparin since delivery. She is now answer: K awaiting discharge A 30 year old woman attends the Your answer: antenatal clinic at 12 weeks gestation. A She had a DVT in her previous Question 20 pregnancy and her sister also had a DVT Correct during pregnancy. Thrombophilia screen answer: I is negative.

| EXPLANATION | Options for Questions 21-22 A In-vitro fertilisation

B Intra-uterine insemination

C Laparoscopy and dye test

D Laparoscopic ovarian drilling

E Clomephene citrate

F

G Metformin

H Carbegolline

I

J

Oocyte donation

K Weight gain

L

Gonadotrophin induction of ovulation Weight reduction Measure serum androgen concentrations

Instructions:For each of the case histories described below, choose the singlemost appropriate initial management from the above list. Each option may be used once, more than once, or not at all. A 34 year old woman with her 35 year old partner has been referred to the infertility clinic because of 2 years of primary infertility. The woman has a regular 28 day cycle and her periods are heavy with Your answer: A severe dysmenorrhoea. She also Question 21complains of deep dyspareunia and intermittent lower abdominal and pelvic Correct pain. Her BMI is 27. Investigations have answer: C shown normal LH, FHS, Thyroid function tests, prolactin and day 21 progesterone concentration confirms ovulatory cycles. Her partner’s semen analysis is normal. A 34 year old woman with her 35 year old Your answer: A Question 22partner has been referred to the infertility clinic because of 2 years of primary

infertility. The woman has a regular and normal 28 day cycle, her BMI is 26 and she has no other symptoms. Investigations have shown normal LH = 3.5mIU/ml, FHS = 5.0mIU/ml, Thyroid function tests, prolactin and day 21 progesterone concentration confirms ovulatory cycles. Her partner’s semen analysis is normal. Hystero-salpingogram confirms bilateral patent fallopian tubes.

Correct answer: A

| EXPLANATION | Options for Questions 23-24 A In-vitro fertilisation

B Intra-uterine insemination

C Laparoscopy and dye test

D Laparoscopic ovarian drilling

E Clomephene citrate

F

G Metformin

H Oocyte donation

I

J

Hystero-salpingogram

K Weight gain

L

Gonadotrophin induction of ovulation Weight reduction Measure serum androgen concentrations

Instructions:For each of the case histories described below, choose the singlemost appropriate initial management from the above list. Each option may be used once, more than once, or not at all. A 41 year old woman with her 35 year old partner has been referred to the infertility clinic because of 2 years of primary infertility. The woman has an irregular Your answer: menstrual cycle every 2-6 weeks, her A BMI is 26 and she has no other Question 23 symptoms. Investigations have shown Correct normal LH = 7.5mIU/ml, FHS = answer: H 22mIU/ml, day 21 progesterone = 3.5ng/ml. Thyroid function tests, prolactin and androgen concentrations are normal. Her partner’s semen analysis is normal. A 37 year old woman with her 45 year old partner has been referred to the infertility clinic because of 2 years of primary infertility. The woman has an irregular menstrual cycle every 2-4 months and Your answer: her partner’s semen analysis shows a volume of 3ml, concentration of Question 24 30,million/ml and motility of 60%. The woman’s BMI is 26 and there is no significant past medical history. Investigations have confirmed a diagnosis of polycystic ovary syndrome which has not responded to treatment with clomephene citrate over 6 months.

| EXPLANATION | Options for Questions 25-26

A Correct answer: D

A Prolactinoma

B Pre-mature ovarian failure

C Turner’s syndrome

D Ovarian hyper-stimulation syndrome

E Polycystic ovary syndrome

F

G Cushing’s syndrome

H Congenital adrenal hyperplasia

I

J

Hyperthyroidism

L

Pure gonadal dysgenesis

Hypothalamic amenorrhoea

Androgen insensitivity K syndrome

Hypothyroidism

Instructions: For each of theofcase histories described below, choose the singlemost likely cause menstrual abnormalities from the above list. Each option may be used once, more than once, or not at all. A 36 year old woman and her 40 year old partner have been referred to the fertility clinic because of a failure to conceive after 18 months of unprotected intercourse. The semen analysis is Your answer: normal. The woman has irregular periods A every 3-6 months but no other symptoms Question 25 and her BMI is 27. The woman’s results Correct are as follows: FSH (day 3) = 6mIU/ml, answer: E LH (day 3) = 8mIU/ml, prolactin = 110 ng/ml, progesterone (day 21) = 3ng/ml, testosterone = 4.0pg/ml, DHEA, DHEAsulphate and thyroid function tests were normal.

A 25 year old medical student attends the gynaecology clinic because she has not had a period for 12 months. Menarche was at the age of 13 and she is otherwise asymptomatic and healthy. Her BMI is 19 and she is a member of the rowing team, Your answer: practising 5 times a week. She denies A any recent weight loss or abnormal eating Question 26 behaviour and has normal secondary Correct sexual characteristics. Random answer: I endocrine profile is as follows: FSH = 5.5mIU/ml, LH = 4.0mIU/ml, progesterone = 1.2ng/ml, prolactin = 56ng/ml, testosterone = 1.2pg/ml. Thyroid function tests and pelvic ultrasound scan are normal.

| EXPLANATION | Options for Questions 27-28 A Cancel IVF cycle

B Freeze embryos

C Admit for iv fluids and thromboprophylaxis

D Termination of pregnancy

E Transfer to ITU

F Fluid restriction

G Drain pleural effusion

H Drain ascites

I

J

K

Drain ovarian cysts Avoid unprotected sexual intercourse

L

Proceed with embryo replacement Diuretics

Instructions:For each of the case histories described below, choose the singlemost appropriate management from the above list. Each option may be used once, more than once, or not at all. A 35 year old woman with PCOS is undergoing IVF treatment. She attends for embryo replacement 6 days after HCG administration and complains of abdominal distension and pain with Question 27nausea and vomiting 2-3 times per day over the previous 48h. examination showed a Clinical mildly distended abdomen and ultrasound scan confirmed bilateral ovarian enlargement 8-10cm. All her blood tests were normal.

Your answer: A

CorrectB answer:

A 35 year old woman with primary infertility is undergoing IVF treatment. She attends 10 days after embryo replacement and complains of abdominal Your answer: distension and pain with nausea and A persistent severe vomiting over the Question 28 previous 48h. Clinical examination Correct showed a markedly distended abdomen answer: C with ascites and pleural effusion. Abdominal and pelvic ultrasound scan confirmed 12cm diameter bilateral cystic ovaries. All her blood tests were normal.

| EXPLANATION | Options for Questions 29-30 A Counsel and offer termination of pregnancy

B Counsel and offer detailed anomaly scan

C Reassure

D Commence folic acid 0.4mg/day

E

Commence folic acid 4mg/day

F

Maternal vitamin K from 36 weeks

G Neonatal oral vitamin K

H Amniocentesis for karyotype

I Discontinue anti-epileptic drug therapy

J

Monitor anti-epileptic drug levels every month

L

Advice against pregnancy

K Advice against breastfeeding M Neonatal im vitamin K

Instructions:For each scenario described below, choose the single most appropriate management from the above list of options. Each option may be used once, more than once, or not at all. An 18 year old epileptic on phenytoin presents in spontaneous labour Question 29unbooked. She has a spontaneous vaginal delivery of a baby weighing 3600g.

Your answer: A Correct answer: M

A year old epileptic has been seizure-free Your answer: Question 30 for 18 months on sodium valproate has A

an anomaly scan showing evidence of fetal spina-bifida

Correct answer: A

| EXPLANATION | Options for Questions 31-32 A Commence oral warfarin

B Await results of D-dimers assay

C dose Commence therapeutic of heparin

D 3-5 Prophylactic dose of heparin for days post-partum

E

Antenatal prophylactic treatment with heparin

F

Antenatal treatment with warfarin

G

Prophylactic heparin for 6 weeks post-partum

H

Continue s/c heparin for 6 weeks

J

Continue heparin therapy for 6 months

L

Continue heparin prophylaxis during labour

I

Stop heparin therapy

K No treatment required

Instructions:For each scenario described below, choose the single most appropriate management from the above list of options. Each option may be used once, more than once, or not at all.

A 35 year old woman with a BMI of 39 has been admitted at 8 weeks gestation Question 31 because of persistent nausea and vomiting

A 28 year old woman with a previous DVT is known to have the antiQuestion 32phospholipid antibody syndrome. She has been referred to the antenatal clinic at 10 weeks gestation

Your answer: A Correct answer: E Your answer: A Correct answer: E

| EXPLANATION | Options for Questions 33-34 A Amniotic fluid embolism

B Placental abruption

C Cardiomyopathy

D Placenta praevia

E Chest infection

F

G CVA

H Pulmonary hypertension

I

Uterine inversion

J

Sepsis

K Haemorrhage

L

Substance misuse

M HELLP syndrome

N Thromboembolism

Endocarditis

O Myocardial infarcation Instructions:For each case described below, choose the singlemost likely cause of maternal death from the above list of options. Each option may be used once, more than once, or not at all. A previously healthy 18-year-old Your answer: Question 33 primigravida presents at 36 weeks feeling A

unwell and tired. Her brother died unexpectedly aged 19 years. Her CXR showed an enlarged heart. While being admitted she developed increasing shortness of breath and died despite intensive resuscitation.

Correct answer: C

A 30-year-old woman, 28 weeks' gestation in her sixth pregnancy presents Your answer: A to A&E with breathlessness and displays Question 34severe anxiety. She had complained of left-sided pelvic pain for a week. While being assessed she collapsed and it was not possible to resuscitate her.

Correct answer: N

| EXPLANATION | Options for Questions 35-36 A Administer regional analgesia if APTT is normal

B Administer regional analgesia if APTT and PT are normal

Administer protamine C sulphate then regional analgesia

D

Advice that regional analgesia is contra-indicated

Administer regional analgesia

F

Administer prophylactic dose of heparin

E

G Remove epidural catheter

Advise against removal of I epidural catheter

Check APTT then remove H epidural catheter if result is normal Check anti-Xa levels then J administer regional analgesia if normal

K Wait for 12h then administer heparin Instructions:For each scenario described below, choose the single most appropriate management from the above list of options. Each option may be used once, more than once, or not at all. A 40 year old woman with a BMI of 39 has had an emergency caesarean section for failure to progress at 6cm Your answer: dilatation under combined spinal epidural A (CSE) analgesia. The procedure was Question 35 uncomplicated with a blood loss of 700ml. Correct A prophylactic does of LMWH was answer: I administered at noon, 6h after of CSE. She requests removal of insertion the epidural catheter at 18:00h. A 25 year old woman with a BMI of 38 has an emergency caesarean section at full dilatation under spinal anaesthesia because of fetal distress. The procedure Question 36 was uncomplicated with blood loss of 700ml. She is now 8h post-surgery and your attention is drawn to instructions on post-partum thromboprophylaxis.

Your answer: A Correct answer: F

| EXPLANATION | Options for Questions 37-38 A In-vitro fertilisation Laparoscopy and dye C test E Clomephene citrate

B Intra-uterine insemination D Laparoscopic ovarian drilling F

Gonadotrophin induction of ovulation

G Metformin

H Carbegolline

I Oocyte donation K Weight gain

J L

Weight reduction Measure serum androgen concentrations

Instructions:For each of the case histories described below, choose the singlemost appropriate initial management from the above list. Each option may be used once, more than once, or not at all. A 35 year old woman with her 45 year old partner has been referred to the infertility clinic because of 2 years of primary infertility. The woman has an irregular Your answer: menstrual cycle every 2-4 months and A her partner’s semen analysis shows a Question 37 volume of 3ml, concentration of Correct 30,million/ml and motility of 60%. The answer: E woman’s BMI is 24 and there is no significant past medical history. Investigations have confirmed a diagnosis of polycystic ovary syndrome. A 24 year old woman with her 35 year old partner has been referred to the infertility clinic because of 2 years of primary infertility. The woman has an irregular Your answer: menstrual cycle every 2-4 months and A her partner’s semen analysis shows a Question 38 volume of 2.5ml, concentration of Correct 30,million/ml and motility of 65%. The answer: J woman’s BMI is 34 and there is no significant past medical history. Investigations have confirmed a diagnosis of polycystic ovary syndrome.

| EXPLANATION | Options for Questions 39-40 A Prolactinoma

B Pre-mature ovarian failure

C Turner’s syndrome E Polycystic ovary syndrome

D Ovarian hyper-stimulation syndrome F Hypothyroidism

G Cushing’s syndrome

H Congenital adrenal hyperplasia

I

J

Hyperthyroidism

L

Pure gonadal dysgenesis

Hypothalamic amenorrhoea

K Androgen insensitivity syndrome

Instructions:For each of the case histories described below, choose the singlemost likely cause of menstrual abnormalities from the above

list. Each option may be used once, more than once, or not at all. A 35 year old woman and her 40 year old partner have been referred to the fertility clinic because of a failure to conceive after 3 years of unprotected intercourse. The semen analysis is normal. The Your answer: woman has irregular periods every 3-6 A months but no other symptoms and her Question 39 BMI is 27. The woman’s results are as Correct follows: FSH (day 3) = 20mIU/ml, LH (day answer: B 3) = 8mIU/ml, prolactin = 110 ng/ml, progesterone (day 21) = 1.5ng/ml, testosterone = 2.0pg/ml, DHEA, DHEAsulphate and thyroid function tests were normal. A 35 year old woman with one previous normal pregnancy 3 years earlier attends the gynaecology clinic because of Your answer: irregular periods every 4-6 months. On A direct questioning, she complains of Question 40persistent headaches in the last 9 months which are worse first thing in the morning Correct and a milky discharge from her right answer: A breast. Her BMI is 27 and there is a bitemporal hemi-anopia on clinical examination.

Home | Signup | Subscribe| Contact us | Medical forums | Privacy | Legal Notices | Copyright © Busy SpR Busy SpR Ltd. is a Registered Company of the United Kingdom. Company Registration Number:4407908 Developed by Last Digital

Options for Questions 1-2 A Emergency caesarean section

B Ultrasound scan for placental site

C Induction of labour with prostaglandins

D Elective caesarean section at 37 weeks

E Oxytocin augmentation of labour

F Fetal scalp blood sampling

G Induction of labour with oxytocin

H Umbilical artery Doppler J

Arrange antenatal clinic follow-up

K Expectant management

I

L

Transfer to high dependency unit

M Maternal blood transfusion

N Vaginal operative delivery

Elective caesarean section at 39 weeks

Instructions:For each of the case histories described below, choose the singlemost appropriate management from the above list of options. Each option may be used once, more than once, or not at all.

Question 1

Question 2

A 20 year old woman complains of constant abdominal pain and vaginal bleeding at 34 weeks gestation. Her BP is 130/60 and pulse is 90bpm. CTG shows contractions every 3 minutes with deep variable decelerations. The cervix is partially effaced and the os is closed. A 35 year old woman presents at 41 weeks gestation with spontaneous rupture of the membranes and heavily bloodstained liquor. Maternal pulse and BP are normal and the

Your answer: A Correct answer: A

Your answer: A Correct answer: G

CTG is reactive. The cervix is partially effaced and 1cm dilated. There are no uterine contractions.

| EXPLANATION | Options for Questions 3-4 A Uterine perforation

B Laparotomy

C Bladder injury

D Bleeding requiring transfusion

E Continuing pregnancy G Wound infection

F Infertility H Failure to identify any pathology

I

J

Failure to gain access to abdominal cavity

Risk of failure of the procedure

Instructions:For each of the case histories described below, choose the singlemost relevant complication that you must discuss with the patient when taking consent prior to surgery from the above list of options. Each option may be used once, more than once, or not at all.

Question 3

Question 4

A 34 year old woman with a previous left salpingectomy for ectopic pregnancy presents with abdominal pain and vaginal bleeding and is found to have an empty uterus with a right adnexal mass consistent with an ectopic pregnancy. A 35 year old woman with a 10 week missed miscarriage is undergoing evacuation of retained products of conception under general anaesthesia

Your answer: A Correct answer: F

Your answer: A Correct answer: A

| EXPLANATION | Options for Questions 5-6 A Emergency caesarean section

B Ultrasound scan for placental site

C Induction of labour with prostaglandins

D Induction of labour by amniotomy

E Oxytocin augmentation of labour

F Fetal scalp blood sampling

G Vaginal operative delivery

H Maternal blood transfusion

Transfer to high dependency unit

J

Expectant management

K Arrange antenatal clinic follow-up

I

L

Treatment with tocolytics

M Umbilical artery Doppler

N Perform Kleihauer test

Instructions:For each of the case histories described below, choose the singlemost appropriate management from the above list of options. Each option may be used once, more than once, or not at all.

Question 5

Question 6

A 35 year old woman presents at 41 weeks gestation with a history of fresh vaginal bleeding and passing several clots. The uterus is soft and non-tender with no contractions. The fetal heart is normal and the woman’s pulse and BP are normal. The placenta is not low and the cervix is partially effaced, 2cm dilated with bulging membranes A 20 year old woman presents at 28 weeks gestation with fresh vaginal bleeding. The fetal heart is normal and the woman’s pulse and BP are normal. The placenta is fundal and the cervix is 1cm long with a closed internal os. There are uterine contractions occurring every 3 minutes.

Your answer: A Correct answer: D

Your answer: A Correct answer: J

| EXPLANATION | Options for Questions 7-8 A Serial beta-HCG assay

B Laparoscopy

C Repeat trans-vaginal scan in 24h

D Repeat trans-vaginal scan in 7 days

E Counsel and discharge

F

G Offer medical treatment with methotrexate

H Laparotomy

I

Serum progesterone assay

K Refer to molar pregnancy centre

Offer surgical evacuation of products of conception

J

Serum AFP assay

L

Refer to other specialty

Instructions:Instructions: For each patient described below choose the singlemost appropriate initial management option from the list. Each option may be used once, more than once, or not at all.

Question 7

Question 8

An asymptomatic 30 year old woman with a previous ectopic pregnancy presents at 6 weeks amenorrhoea for transvaginal scan to exclude a repeat ectopic pregnancy. The scan is reported to show a 14mm endometrium with no gestation sac. There are no adnexal masses and no free fluid in the pouch of Douglas. HCG concentration is 500IU and 1600IU 48h later An asymptomatic 30 year old woman with a previous ectopic pregnancy presents at 6 weeks amenorrhoea for transvaginal scan to exclude a repeat ectopic pregnancy. Her pregnancy test had been positive 2 weeks earlier. The scan is reported to show a 14mm endometrium with no gestation sac.

Your answer: A Correct answer: D

Your answer: A Correct answer: B

There is a 4cm structure in thebut right suggestive of ancystic ectopic pregnancy noadnexum free fluid in the pouch of Douglas

| EXPLANATION | Options for Questions 9-10 A Serial beta-HCG assays

B Laparoscopy

C Repeat trans-vaginal scan in 24h

D Repeat trans-vaginal scan in 7 days

E Counsel and discharge

F

G Offer medical treatment with methotrexate

H Laparotomy

I

Serum progesterone assay

K Refer to molar pregnancy centre

Offer surgical evacuation of products of conception

J

Serum AFP assay

L

Refer to other specialty

Instructions:For each patient described below choose the singlemost appropriate initial management option from the list. Each option may be used once, more than once, or not at all.

Question 9

Question 10

A 42 year old woman presents with 8 weeks of amenorrhoea, a positive pregnancy test and fresh vaginal bleeding. Transvaginal scan shows an enlarged uterus with a snow-storm appearance consistent with a complete molar pregnancy A 24 year old woman with a regular 28 day cycle had attended 2 weeks earlier with abdominal pain and fresh vaginal bleeding. Trans-vaginal scan showed a viable 6 week intra-uterine pregnancy. She now re-attends with persistent vaginal bleeding which has settled over the last 24h. Transvaginal scan shows an empty uterus with a 3cm haemorrhagic left ovarian cyst and a small amount of free

Your answer: A Correct answer: F

Your answer: A Correct answer: E

fluid in the pouch of Douglas.

| EXPLANATION | Options for Questions 11-12 A Counsel and offer evacuation of retained products B Counsel and offer support group of conception C Counsel and offer laparoscopy & salpingectomy

D

Counsel and offer investigation for recurrent miscarriage

E Counsel and offer treatment with methotrexate

F

Counsel and offer laparotomy & salpingectomy

G Counsel and offer termination of pregnancy

H Perform salpingectomy

I

J

Serial HCG assay

L

Repeat trans-vaginal scan 7 days later

Proceed to laparotomy

K Counsel and refer to early pregnancy assessment unit

Instructions:For each patient described below choose the singlemost appropriate initial management option from the list. Each option may be used once, more than once, or not at all.

Question 11

Question 12

A 25 year old woman had a transvaginal scan following IVF treatment during which 2 embryos were replaced and was found to have a 6 week singleton intra-uterine pregnancy. She presents 2 weeks later with abdominal pain and vaginal bleeding and is found to have an empty uterus with a small amount of free fluid in the pouch of Douglas. A 25 year old woman had a transvaginal scan following IVF treatment during which 2 embryos were replaced and is found to have a 6 week singleton intra-uterine pregnancy with a 5cm cystic structure adjacent to the right ovary with internal echoes consistent with an ectopic pregnancy.

Your answer: A Correct answer: B

Your answer: A Correct answer: C

| EXPLANATION | Options for Questions 13-14 A Serial beta-HCG assays

B Laparoscopy

C Repeat trans-vaginal scan in 24h

D Repeat trans-vaginal scan in 7 days

E Reassure

F

G Offer medical treatment with methotrexate

H Laparotomy

I

Serum progesterone assay

K

J

Offer surgical evacuation of products of conception Serum AFP assay

L Refer to molar pregnancy centre

Refer to other specialty

Instructions:For each patient described below choose the singlemost appropriate initial management option from the list. Each option may be used once, more than once, or not at all.

Question 13

Question 14

An asymptomatic 24 year old woman with a regular 28 day cycle presents with 7 weeks amenorrhoea. Her pregnancy test is positive. Trans-vaginal scan shows a gestation sac of 20mm diameter with a 12mm (~7 weeks size) fetal pole but no fetal heart activity. A 24 year old woman with an irregular menstrual cycle presents with 7 weeks amenorrhoea and fresh vaginal bleeding. Her pregnancy test is positive. Trans-vaginal scan

Your answer: A Correct answer: F

Your answer: A

shows a gestation sac of 12mm diameter (~5 weeks) with a yolk sac but no fetal pole

Correct answer: D

| EXPLANATION | Options for Questions 15-16 A Emergency caesarean section

B Ultrasound scan for placental site

C Induction of labour with prostaglandins

D Induction of labour by amniotomy

E Oxytocin augmentation of labour G Vaginal operative delivery

F Fetal scalp blood sampling H Maternal blood transfusion

Transfer to high dependency unit

J

Expectant management

K Arrange antenatal clinic follow-up

I

L

Treatment with tocolytics

M Umbilical artery Doppler

N Perform Kleihauer test

Instructions:For each of the case histories described below, choose the singlemost appropriate management from the above list of options. Each option may be used once, more than once, or not at all.

Question 15

Question 16

A 34 year old woman is brought in by ambulance because of severe constant abdominal pain and vaginal bleeding at 35 weeks gestation. On admission, her pulse is 120bpm, BP 80/40 and she is unresponsive. The fetal heart is 80bpm A 25 year old woman with major placenta previa presents at 32 weeks gestation with fresh vaginal bleeding. Her pulse is 70bpm and BP 120/80. The fetal heart rate is normal.

Your answer: A Correct answer: H

Your answer: A Correct answer: J

| EXPLANATION | Options for Questions 17-18 A Serial beta-HCG assays

B Laparoscopy

C Repeat trans-vaginal scan in 24h

D Repeat trans-vaginal scan in 7 days

E Reassure

F

G Offer medical treatment with methotrexate

H Laparotomy

I

Serum progesterone assay

K Refer to molar pregnancy centre

Offer surgical evacuation of products of conception

J

Serum AFP assay

L

Refer to other specialty

Instructions:Instructions: For each patient described below choose the singlemost appropriate initial management option from the list. Each option may be used once, more than once, or not at all.

Question 17

Question 18

An asymptomatic 24 year old woman with a regular 28 day cycle presents with 7 weeks amenorrhoea. Her pregnancy test was positive 2 weeks earlier and she has been referred because of a history of previous ectopic pregnancy. Transvaginal scan shows an empty uterus with no adnexal masses and no free fluid in the pouch of Douglas. A 24 year old woman with one previous ectopic pregnancy and a regular 28 day cycle presents with 7 weeks amenorrhoea and a 6h history of constant abdominal pain. Her pregnancy test was positive 2 weeks earlier. Transvaginal scan shows an empty uterus with no adnexal masses and some free fluid in the pouch of Douglas. She collapses

Your answer: A Correct answer: A

Your answer: A Correct answer: H

shortly after the ultrasound scan.

| EXPLANATION | Options for Questions 19-20 A Counsel and offer evacuation of retained products B Counsel and offer support group of conception C Counsel and offer laparoscopy & salpingectomy

D

Counsel and offer investigation for recurrent miscarriage

E Counsel and offer treatment with methotrexate

F

Counsel and offer laparotomy & salpingectomy

G Counsel and offer termination of pregnancy

H Perform salpingectomy

I

J

Serial HCG assay

L

Repeat trans-vaginal scan 7 days later

Proceed to laparotomy

K Counsel and refer to early pregnancy assessment unit

Instructions:For each patient described below choose the singlemost appropriate initial management option from the list. Each option may be used once, more than once, or not at all.

Question 19

Question 20

A 25 year old woman has a transvaginal scan following IVF treatment during which 2 embryos were replaced and is found to have a 6 week singleton intra-uterine pregnancy with no fetal heart activity. A 25 year old nulliparous woman has had 2 previous first trimester miscarriages presents at 8 weeks gestation for an early pregnancy scan. She is found to have a 7 week missed miscarriage.

Your answer: A Correct answer: A

Your answer: A Correct answer: A

| EXPLANATION | Options for Questions 21-22 A Serial beta-HCG assay

B Laparoscopy

C Repeat trans-vaginal scan in 24h

D Repeat trans-vaginal scan in 7 days

E Counsel and discharge

F

G Offer medical treatment with methotrexate

H Laparotomy

I

Serum progesterone assay

K Refer to molar pregnancy centre

Offer surgical evacuation of products of conception

J

Serum AFP assay

L

Refer to other specialty

Instructions:Instructions: For each patient described below choose the singlemost appropriate initial management option from the list. Each option may be used once, more than once, or not at all.

Question 21

Question 22

An asymptomatic 30 year old woman with a previous ectopic pregnancy presents at 6 weeks amenorrhoea for transvaginal scan to exclude a repeat ectopic pregnancy. The scan is reported to show a 14mm endometrium with no gestation sac. There are no adnexal masses and no free fluid in the pouch of Douglas. HCG concentration is 500IU and 1600IU 48h later An asymptomatic 30 year old woman with a previous ectopic pregnancy presents at 6 weeks amenorrhoea for transvaginal scan to exclude a repeat ectopic pregnancy. Her pregnancy test had been positive 2 weeks earlier. The scan is

Your answer: A Correct answer: D

Your answer: A Correct answer: B

reported to show a 14mm endometrium with no gestation sac. There is a 4cm cystic structure in the right adnexum suggestive of an ectopic pregnancy but no free fluid in the pouch of Douglas

| EXPLANATION | Options for Questions 23-24 A Emergency caesarean section

B Ultrasound scan for placental site

C Induction of labour with prostaglandins

D Induction of labour by amniotomy

E Oxytocin augmentation of labour

F Fetal scalp blood sampling

G Vaginal operative delivery

H Maternal blood transfusion

Transfer to high dependency unit

J

Expectant management

K Arrange antenatal clinic follow-up

I

L

Treatment with tocolytics

M Umbilical artery Doppler

N Perform Kleihauer test

Instructions:For each of the case histories described below, choose the singlemost appropriate management from the above list of options. Each option may be used once, more than once, or not at all.

Question 23

Question 24

A 42 year old woman had an elective caesarean section at 37 weeks gestation for major placenta previa. She returned to theatre 6h later because of suspected intra-abdominal bleeding and has undergone a total abdominal hysterectomy

A 35 year old Rhesus positive woman presented with fresh vaginal bleeding and intermittent abdominal pain at 30 weeks gestation. Maternal pulse on admission was 90bpm with BP 120/80. The fetal heart rate was normal. She has been in hospital for 48h and complains of a slight brown discharge but no other symptoms and has had no further bleeding.

Your answer: A Correct answer: I

Your answer: A Correct answer: K

| EXPLANATION | Options for Questions 25-26 A Emergency caesarean section

B Ultrasound scan for placental site

C Induction of labour with prostaglandins

D Elective caesarean section at 37 weeks

E Oxytocin augmentation of labour

F Fetal scalp blood sampling

G Induction of labour with oxytocin

H Umbilical artery Doppler

I

J

Elective caesarean section at 39 weeks

K Expectant management M Maternal blood transfusion

Arrange antenatal clinic follow-up

L Transfer to high dependency unit N Vaginal operative delivery

Instructions:For each of the case histories described below, choose the singlemost appropriate management from the above list of options. Each option may be used once, more than once, or not at all.

Question 25

Question 26

A 35 year old woman with major placenta previa has been in hospital since 32 weeks gestation when she presented with a small antepartum haemorrhage. She has had no further bleeding and is now 35 weeks pregnant A 36 year old woman presents in spontaneous labour at 39

Your answer: A Correct answer: D

Your answer: A

weeks gestation and progresses rapidly to full dilatation with a direct occipito-anterior position 1cm below the ischial spines. She complains of sudden onset constant abdominal pain and fresh vaginal bleeding and the CTG shows deep decelerations

Home | Signup | Subscribe| Contact us | Medical forums

Correct answer: N

View more...

Comments

Copyright ©2017 KUPDF Inc.
SUPPORT KUPDF